Sie sind auf Seite 1von 298

Design of Foundation Structures

and Retaining Walls

PROBLEMS
Authors Jutila, A., Syrjä, R.
Publisher Aalto University
School of Science and Technology
Faculty of Engineering and Architecture
Department of Structural Engineering and Building
Classification 66
Place Espoo, Finland
Year 2012
3

FOREWORD

This educational material includes problems of the course named Design of


Foundation Structures and Retaining Walls -course (Rak-11.2107). The course is
part of the degree programme of Structural Engineering and Building Technology
in Aalto University.

The problems are grouped to the subject matters. Each problem has a description
of the problem and the model solution.

This material has prepared since the end of 1990 decade. Two design practices,
Eurocode and Finnish standard, so-called B-set preceding Eurocodes, are
represented on the material.

This material is also published in Finnish (Perustusten ja tukimuurien


rakennustekninen suunnittelu - Esimerkkitehtävät ).

Otaniemi, 29th of October 2012


Authors
5

CONTENTS

Page
I Loads of Structures
1. Crane frame: Force and Displacement Methods 7
2. Multi-Storied Building: Combination of Actions, Eurocode 17
3. Multi-Storied Building: Combination of Actions, RIL 144 25
4. Dam: Water Pressure, Tilting and Sliding Stability 31

II Loads of Bridges
5. Elastic Intermediate Support 37
6. Simple Beam: Differential Equation of Deflection 43
7. Influence line: Reaction Force 49

III Earth Pressure


8. Cellar Wall: Neutral Pressure and Compressed Filling 55
9. Caisson: Horizontal Component of the Active Pressure 63
10. Retaining Wall: Active Pressure and Pressure in Rest 69

IV Core Figure of the Cross-Section


11. Homogenous Triangle 77
12. Elliptical Two-Component Foundation 81
13. Pile Group 89
14. Core Figure of a Two-Component Foundation 97
15. Core Figure of a Multi-Component Bar Having an Elliptical
Cross-Section 103

V Rock Foundation
16. Base Slab: Pressure Under the Slab 111
17. Base Slab: Pressure Under the Slab, Neutral Axis,
Prestressing 117
18. Retaining Wall: Safety Against Over-Turning and Sliding 125
Contents 6

VI Carrying Capacity of a Base Slab and Wall


19. Ultimate Limit State (ULS) Method by Eurocode 131
20. ULS Method by Eurocode and Finnish national annex 141
21. ULS Method by Eurocode and Finnish national annex,
Design Approach 2* 151
22. ULS Method by old Finnish Standards
23. Total Safety Method 169
24. Method of the Allowed Bearing Resistance 177

VII Pile Group


25. Variable Pile Length 183
26. Constant Stiffness, Vertical and Inclined Piles 185
27. Variable Stiffness and Slope 193
28. Column Foundation Consisting of a Pile Group 201
29. Core Figure of the Pile Group 209
30. Space Pile Group 215

VIII Large Diameter Pile


31. Finite Element Method (FEM) 221
32. Column Foundation: FEM 229
33. Bridge Intermediate Support: Force Method 237
34. Bridge Intermediate Support: Deflection Method 243
35. Bridge Intermediate Support: Mohr's Method 247
36. Bridge Intermediate Support: FEM 251

IX Support on Elastic Foundation


37. Base Slab: Winkler Support 257
38. Floor: Three-Layer Foundation 263
39. Foundation Wall: Pressure as Function of the Foundation
Modulus 271

X Bridge Abutment
40. Carrying Capacity 277
20111205 7
1. PROBLEM

Determine the horizontal component of the support reaction force of the crane
frame (Figure 1) by using force, slope-deflection and moment methods,
respectively!

3L/8 2L/8 3L/8

2F

D = EI L

Figure 1.
1. Problem, Solution 8

FORCE METHOD

At the statically determinate basic form the sum of the deflection (d 10) from the
external load (2F ) and the deflection (X 1d 11) from the unknown horizontal force
(X 1) is zero. Here d 11 is the displacement from force 1. See Figure 2.
d 10+X 1d 11 = 0 (1)
F F

1 X1
d10
3L/8 2L/8 3L/8 d11 X1d11

Figure 2.

Bending moment distribution curve due to external load is shown in Figure 3a


and due to force 1 in Figure 3b.

Bending moment in the horizontal beam due to external load


  3L 
 Fx, x  0, 8 
  
 3FL  3L 5L  (2a...c)
M0   , x ,
 8  8 8 
  5L 
 F ( L  x ), x   , L
  8 
There is no moment at the vertical columns.

Bending moment in the horizontal beam due to force F = 1 (Fig. 3b)


 L, palkki (3a, b)
M1  
 y, pilari
1. Problem, Solution 9

L
+ + +
3FL/8
L
y

x
3L/8 3L/8 L

a) b)
Figure 3.

Displacements
 M1 M 0
d 10   D ds
 M M
d 11   1 1 ds
 D (4a, b)

When inserting the bending moments we get


 15FL3 (5a, b)
d
 10 
 64 D

 5 L3
d 
 11 3 D

ANSWER:
Horizontal force
 d 10
X1 
d 11 (6)
 9F

64 (7)
Negative sign means that the force direction is opposite to the supposed
original one.
1. Problem, Solution 10

DISPLACEMENT METHOD (especially slope-deflection method)

Corner moments (Figure 4)


 M12  0

 M 21  a21f21  c21 21  M K 21
o o o
M  a f  b f  c   M
 23 23 23 23 32 23 23 K 23
 (8a…f)
 M 32  a32f32  b32f23  c32 32  M K 32

 M 34  a34f34  c34 34  M K 34
o o o

 M 43  0

2 3
Mij Dij = EijIij Mji x
i j

vi
ij fji
ji vj
y fij
1 4 lij
a) b)
Figure 4.

Compatibility conditions
f 21 = f 23 = f 2 (9)
f 32 = f 34 = f 3 = - f 2 (10)

Equilibrium conditions
0  M 21  M 23  M 23   M 21  M 2  M

0  M 32  M 34  M 34   M 32  M 3  M 2  M (11a, b)
1. Problem, Solution 11

Slope rotation in basic form (Formulary )


v2  v1 (12)

l

Member coefficients in basic form


4 EI 4 D (13)
a 
l l
2 EI 2 D (14)
b 
l l
6 EI 6 D (15)
c 
l l
3 EI 3 D (16)
ao  c o  
l l

Corner moments caused by loading (Figure 5) in basic form


 Fa1b12  Fa2b22 (17)
M Ki   M Kj  
l2 l2

a1 b1
a2 b2

MKi F F MKj
l

Figure 5.

Slope rotations in this case


 21   34   (18)
 23   32  0 (19)
1. Problem, Solution 12

Member coefficients in this case


4 EI 4 D (20)
a21  a23  a32  a34   a
L L
2 EI 2 D (21)
b23  b32   b
L L
6 EI 6 D (22)
c 23  c 32   c
L L
o o o o 3 EI 3 D (23)
a21  a34  c 21  c 34    ao
L L

Corner moments caused by loading in this case


 15FL
M K 23   M K 32 
64 (24)

Corner moments after simplification


 –M 2 = a f –a 
o o


 M 2 = a f –b f M K

 –M 3 = –a f +b f – M K (25a…d)
 M 3 = –a o f –a o 

From Equations 11
 M   (a o  a  b)f  a o
K

 M K  (a o  a  b)f  a o (26a, b)

Slope
 =0 (27)

From Equations 26a and 27


M
f o K
a ab (28)
3FL2
f 
64 D (29)
1. Problem, Solution 13

From Equation 25a


M  a of (30)
 9FL
M
64 (31)

Horizontal force at the support (Figure 6)


HL = –M 34 (32)

H
M34 = M
L
M43 = 0 H

Figure 6.

ANSWER: Horizontal force


9F
H
64 (33)
1. Problem, Solution 14

FORCE METHOD (especially moment method)

Rotations at corners (Figure 4)


f   M   M     o
 21 21 21 21 12 21 21
f23   23 M 23   23 M 32   23   23
o
 (34a…d)
f32   32 M 32   32 M 23   32   32
o

f34   34 M 34   34 M 43   34   34
o

Compatibility conditions
f 21 = f 23 (35)
f 32 = f 34 (36)

Corner moments
0  M 21  M 23  M 23   M 21  M 2  M

0  M 32  M 34  M 34   M 32  M 3  M 2  M

0  M12 (37a…d)
0  M 43

Slope rotation in basic form


v v (38)
 2 1
l

Member coefficients in basic form


l l (39)
 
3 EI 3 D
l l (40)
 
6 EI 6 D
1. Problem, Solution 15

Rotations caused by loading (Figure 7) in basic form

 io  1 1 b1  l   2 2 b2  l 
Fa b Fa b (41)
6 Dl 6 Dl
 Fa1b1
 oj  a1  l    Fa2b2 a2  l  (42)
6 Dl 6 Dl

a1 b1
a2 b2
F F
io l - jo
Figure 7.

Slope rotations in this case


 21   34   (43)
 23   32  0 (44)

Member coefficients in this case


L (45)
 21   23   32   34  
3D
L (46)
 21   23   32   34  
6D

Rotations caused by loading in this case


 21
o
  34
o
0 (47)
15FL2
 23
o
  32
o
 o
128 D (48)
1. Problem, Solution 16

Rotations after simplification


f21  M  

f23  (   ) M  
o
 (49a…d)
f32  (   ) M   o

f34  M  

From Equations 35 and 36


  o  ( 2   ) M   (50)
  o  ( 2   ) M   (51)

Slope
 =0 (52)

From Equations 50 and 52


o
M
2   (53)

When inserting member coefficients we get


 9FL
M
64 (54)

Horizontal force at the support (Figure 6)


HL = –M 34 (55)

ANSWER: Horizontal force


9F
H
64 (56)
20111205 17
2. PROBLEM

A four-storey, steel-framed residential building with flat roof is shown in Figure 1.


Determine all load combinations acting on the base slab of the outside column (grey
colour)! Use ultimate limit state (STR/GEO) according to Eurocode and National
Annex of Finland [NA SFS-EN 1990 , Table A1.2(B)].
What is the maximum overturning moment at the top of the base slab, when wind
is acting in the direction of the shorter side? The wind load is assumed to divide
equally to all piles. The building is situated in Kemijärvi (Lapland).

Number of columns
n1 = 3
n2 = 5
Dimension
a = 5m
Number of stories and storey height
nk = 4
hi = 3m
The vertical forces are caused by the are shown by the dotted lines.
Self weight of roof and floors
p = 0,005 MN/m2
Self weight of the main wall and column
v = 0,020 MN/m

A A AA
h4
h3 a
h L1 = 2a(n1-1)
h2 a
h1
a a a a
a a a a L2 = 2a(n2-1)

Figure 1.
2. Problem, Solution 18

COMBINATIONS OF ACTIONS

Design equation (NA* SFS-EN 1990 , Table A1.2(B))


x gGj ,sup  g Q1  g Qi 
Fd   K FI Gkj ,sup  g Gj ,inf Gkj ,inf   K FI Qk1   K FI y 0 i Qki (1)
g Gj ,sup  0  0 

where G kj, sup is upper characteristic value of permanent action j and


corresponding partial factor is
g Gj, sup = 1,35
G kj, inf is lower characteristic value of permanent action j and corresponding
partial factor is
g Gj, inf = 0,9
Q k 1 is characteristic value of the leading variable action 1 and corresponding
partial factor is
gQ1 = 1,5
Q ki is characteristic value of accompanying variable action i and corresponding
partial factor is
g Qi = 1,5
reduction factor is
x = 0,85
Four-storey building belongs to reliability class RC2 (NA SIS-EN 1990 , Table
B1), thus factor of actions (SFS-EN 1990 , Table B3)
K FI = 1,0
Factor for combination value of a variable actions: imposed, snow and wind loads
(NA SFS-EN 1990 , Table A1.1, Class A)
y 0,imposed = 0,7
y 0,snow = 0,7
y 0,wind = 0,6
Products
xg Gj, sup  1,15
g Qi K FI y 0,imposed = 1,05
g Qi K FI y 0,snow = 1,05
g Qi K FI y 0,wind = 0,90
2. Problem, Solution 19

HORIZONTAL FORCE DUE TO SWAY IMPERFECTIONS

Basic value of sway imperfection (SFS-EN 1993-1-1 , Chapter 5.3.2)


f0 = 0,005
Reduction factor for height h applicable to columns
  2
 
max  h0 h
a h  min  2 (2)
  3

1
where
h0 = 1 m-1
It is obtained
ah = 0,667
Number of columns in a row
m = ni-1 (3)
= 2
Reduction factor for the number of column in a row
1 1
am  1  
2 m (4)
= 0,866
Global initial sway imperfection (SFS-EN 1993-1-1 , Equation 5.5)
f = f 0a h a m (5)
= 0,002887

Horizontal force is acting in the level of storey floor and is


H = fN (6)
where N is the corresponding axial force (SFS-EN 1993-1-1 , Figure 5.4).
2. Problem, Solution 20

SELF WEIGHT OF STRUCTURE

Roof or floor
G pi = 2a 2p (7)
= 0,250 MN
Wall and column
G vi = 2a v (8)
= 0,200 MN
Characteristic value of vertical force at one floor
G i = G pi +G vi (9)
= 0,450 MN
Height of the building
h = nkhi (10)
= 12,000 m
Moment due to self weigh of roof and sway imperfection
M G 1 = f G pi h
= 0,009 MNm (11)
Moment due to self weigh of three upper floors and sway imperfection
M G 2 = 3f G i h /2 (12)
= 0,023 MNm
Sum of moments
M G = M G 1+M G 2 (13)
= 0,032 MNm

IMPOSED LOAD

Building belongs to category A (NA SFS-EN 1990 , Table A1.1).


Load per unit area (NA SFS-EN 1991-1-1 , Table 6.2)
q ki = 0,002 MN/m2
Characteristic value of vertical force
Q ii = 2a 2q ki (14)
= 0,100 MN
2. Problem, Solution 21

Number of stories above the second floor


n = nk-1 (15)
= 3
Reduction factor (NA SFS-EN 1991-1-1 , Chapter 6.3.1.2(11))
2  ( n  2)y 0, imposed
an 
n (16)
= 0,900
Moment due to imposed floor loads and sway imperfection
M i = f (a n h i +2h i +3h i )Q ii (17)
= 0,005 MNm

SNOW LOAD

Characteristic value of snow load on the ground in Kemijärvi (NA SFS-EN


1991-1-3 , Figure 4.1)
s k = 0,002750 MN/m2
Snow load shape coefficient for the flat roof (SFS-EN 1991-1-3 , Table 5.2)
mi = 0,8
Exposure coefficient C e (normal topography) and thermal coefficient C t (SFS-EN
1991-1-3 , Chapter 5.2(7) and 5.2(8))
Ce = 1,0
Ct = 1,0
Snow load (SFS-EN 1991-1-3 , Equation 5.1)
s = miCeCtsk (18)
2
= 0,002200 MN/m
Characteristic vertical force
Q ks = 2a 2s (19)
= 0,110 MN
Moment due to snow load and sway imperfection
M s = f Q ks h (20)
= 0,004 MNm
2. Problem, Solution 22

WIND LOAD

Shorter side length


L 1 = 2a (n 1- 1) (21)
= 20 m
Longer side length
L 2 = 2a (n 2- 1) (22)
= 40 m
Reference area
A ref = hL 2 (23)
2
= 480 m
Fundamental value of the basic wind velocity (NA SFS-EN 1994-1-4 , Chapter 4.2)
v b, 0 = 21 m/s
Directional factor (SFS-EN 1991-1-4 , Chapter 4.2)
c dir = 1,0
Season factor (SFS-EN 1991-1-4 , Chapter 4.2)
c season = 1,0
Basic wind velocity (SFS-EN 1991-1-4 , Equation 4.1)
v b = c dir c season v b, 0 (24)
= 21 m/s
Air density (SFS-EN 1991-1-4 , Chapter 4.5)
r = 1,250 kg/m3
Basic velocity pressure (SFS-EN 1991-1-4 , Equation 4.10)
1
qb  rvb2
2 (25)
2
= 276 N/m

Terrain category is selected to be II (SFS-EN 1991-1-4 , Appendix A).


Exposure factory at the top of the building (SFS-EN 1991-1-4 , Figure 4.2)
C e,wind = 2,5
Peak velocity pressure (SFS-EN 1991-1-4 , Equation 4.8)
q p = C e,wind q b (26)
= 689 N/m2
2. Problem, Solution 23

Because the height h is less than the shorter side length, the wind pressure should
be considered to be one part (SFS-EN 1991-1-4 , Chapter 7.2.2, Figure 7.4).
Ratio (SFS-EN 1991-1-4 , Figure 7.5)
k = h/d = L 1/h (27)
= 0,600
Pressure coefficient for the external pressure for zones D and E, when the loaded
2
area is more than 10 m , is obtained by linear interpolation (SFS-EN 1991-1-4 ,
Table 7.1)
0, 8  0, 7
C pe ,10, D  (k  0, 25 )  0,7
1  0, 25 (28)
= 0,747
 0,5  0, 3 
C pe,10, E    (k  0, 25)  0, 3
 1  0, 25  (29)
= -0,393
Wind pressure acting on the external surfaces (SFS-EN 1991-1-4 , Equation 5.1)
w e,i = q p C pe, 10, i (30)
2
w e,D = 515 N/m
w e,E = -271 N/m2
Structural factor (SFS-EN 1991-1-4 , Chapter 6.2)
cscd = 1
Wind force (SFS-EN 1991-1-4 , Equation 5.5)
F w,e = c s c d (w e,D -w e,E )A ref (31)
= 0,377 MN
For one column
Fw,e
Fw 
n1n2 (32)
= 0,025 MN
Moment
M w = F w h /2 (33)
= 0,151 MNm
2. Problem, Solution 24

ANSWER:

Combinations of actions
1,15  1,5  1,05  0, 9  (34a)
1,15  1,5  1,05  0 
    (34b)
1,15  1,5  0  0, 9  (34c)
   
1,15  1,5  0  0  (34d)
1,15  1,05  1,5  0, 9  (34e)
   
1,15  1,05  1,5  0  (34f)
   
Fd  1,15 Gk ,sup  0,9Gk ,inf  0 Qki  1,5 Qks  0,9Qkw
(34g)
1,15  0  1,5  0 
    (34h)
1,15  1,05  1,05  1,5 
(34i)
1,15  1,05  0  1,5 
    (34j)
1,15  0  1,05  1,5 
    (34k)
1,15  0  0  1,5 
(34l)
1, 35  0  0  0 
(34m)
Wind is caused the leading moment effect. Thus the unfavourable case is i.

Maximum moment
M = 1,15M G +1,05M i +1,05M s +1,5M w (35)
= 0,272 MNm
20111205 25
3. PROBLEM

A four-storey building with flat roof is shown in Figure 1. Determine the extreme
values of vertical and horizontal forces acting on the base slab of the outside
column (grey colour) - horizontal ones in the direction of the shorter side!
A load combination is determined in ultimate limit state (design of structural
member) by using Finnish standard RIL 144-2002 . The wind load is assumed to
divide equally to all piles. The building is situated in Kemijärvi (Lapland).

Number of columns
n1 = 3
n2 = 5
Dimension
a = 5m
Number of stories and storey height
nk = 4
hi = 3m

The vertical forces are caused by the are shown by the dotted lines.
Self weight of roof and floors
2
p = 0,005 MN/m
Self weight of the main wall and column
v = 0,020 MN/m

A A AA
h4
h3 a
h L1 = 2a(n1-1)
h2 a
h1
a a a a
a a a a L2 = 2a(n2-1)

Figure 1.
3. Problem, Solution 26

SELF WEIGHT OF THE STRUCTURE

Roof or floor
G pi = 2a 2p (1)
= 0,250 MN
Wall and column
G vi = 2a v (2)
= 0,200 MN
Characteristic value of vertical force at one floor
G i = G pi +G vi (3)
= 0,450 MN
Sum of loads
4
G  G pi   Gi
n 1 (4)
= 2,050 MN

SNOW LOAD

Characteristic value of snow load on the ground in Kemijärvi (RIL 144-2002 ,


Figure 4.121, p. 24)
sk = 0,002 MN/m2
Snow load shape coefficient for flat roof (RIL 144-2002 , Figure 4.122d, p. 27)
m = 1,0
Characteristic vertical force (RIL 144-2002 , Chapter 4.1, p. 23)
Q k = 2a 2m s k (5)
= 0,100 MN
3. Problem, Solution 27

IMPOSED LOAD

Load per unit area (RIL 144-2002 , Table 5.12, p. 78)


q ik = 0,001500 MN/m2
Characteristic value of vertical force
Q ii = 2a 2q ik (6)
= 0,075 MN
Reduction factor (RIL 144-2002 , Table 8.21b, p. 150)
k = 0,75
Characteristic value of vertical force
4
Qi  k  Qii
i 1
(7)
= 0,225 MN

HORIZONTAL FORCE DUE TO SWAY IMPERFECTIONS

Shorter side length


L 1 = 2a (n 1-1) (8)
= 20 m
Longer side length
L 2 = 2a (n 2-1) (9)
= 40 m
Horizontal force is acting in the level of storey floor and is
H = fN (10)
where f is in the direction of the shorter side (RIL 144-2002 , Chapter 6.5, p. 134)
f = 1/150
= 0,006667
and N is the corresponding axial force.
3. Problem, Solution 28

WIND LOAD

Height of the building


4
h   hi
n 1 (11)
= 12 m
Terrain category is selected to be II.
Wind pressure (RIL 144-2002 , Table 4.22a, p. 31)
0, 24
 h
qwk  q0  
 h0  (12)
where
q 0 = 0,000650 MN/m2
h0 = 10 m
Thus
q wk = 0,000679 MN/m2
Pressure coefficient for the windward external pressure (RIL 144-2002 ,
Table 4.231a, p. 38)
C p,t = 0,7
Ratio between the side lengths, when wind acting in the direction of the shorter
side
L 1/L 2 = 0,500
Pressure coefficient for the leeward external pressure
C p,s = -0,5
Wind load for one column (RIL 144-2002 , Equation 4.212b, p. 30)
hL2 (C p , t  C p , s )qwk
Qw 
n1 n2 (13)
= 0,026 MN
3. Problem, Solution 29

COMBINATION OF ACTIONS

Design equation (RIL 144-2002 , Chapter 8.2, p. 149)


m n
qd   g gi gi g q1qk1  g q 2qk 2   g qi qki
i 1 i 3 (14)
can be written
1,2  1,6 1,6  0,8 (1)

1,2  
1,6 
1,6  0  ( 2)
1,2  1,6 0,8 1,6  ( 3)
   
1,2  1,6 0 1,6  ( 4)
1,2  1,6 0  0  ( 5)
qd   g  qi  qk  qw
0,9 0 1,6  0,8 ( 6) (15a…j)
0,9 0 1,6  0 (7)
   
0,9 0 0,8 1,6  ( 8)
0,9 0 0 1,6  ( 9)
   
0,9 0  0  0  (10)

Sub index d refers to design value.


Below the sub indexes are referred to the load combination cases (row number).
(Amount of load cases is double, if g q 1 = 0 when g g = 1,2, and g q 1 = 1,6 when
g g = 0,9.)

Horizontal forces
Maximum snow load
V d,1,2 = 1,2G +1,6Q i +1,6Q k (16)
= 2,980 MN
Maximum wind load
V d,3 = 1,2G +1,6Q i +0,8Q k (17)
= 2,900 MN
Minimum self weight
V d,10 = 0,9G (18)
= 1,845 MN
3. Problem, Solution 30

Horizontal forces and corresponding distances from the base slab


Maximum snow load
 H1  1,2f (G p5  4Gi )  1,6f  4Qii  1,6fQk  0,8Qw (19)
= 0,042 MN
 3  3 h
1, 2f  G p5 h  4Gi h   1,6f  4Qii h  1,6fQk h  0,8Qw
e1   8  8 2 (20)
H1
= 5,807 m
Maximum wind load
 H 3  1,2f (G p5  4Gi )  1,6f  4Qii  0,8fQk  1,6Qw (21)
= 0,062 MN
 3  3 h
1, 2f  G p5 h  4Gi h  1,6f  4Qii h  0,8fQk h  1,6Qw
e3   8  8 2 (22)
H3
= 5,819 m
Minimum loads
H 10 = 0 MN

ANSWER:

Vertical forces
V d,max = 2,980 MN
V d,min = 1,845 MN
Horizontal forces and corresponding distances from the base slab
H d,max = 0,062 MN
e d,max = 5,819 m
H d,min = 0,000 MN
20111205 31
4. PROBLEM

A massive reinforced concrete dam with a inspection manway is shown in Figure 1.


Determine water pressures and buoyancy (uplift force) resultants and
corresponding locations in x-y- coordinate system, when the water deep of the lower
pond is
h a ,1 = 3,000 m

Determine stability against tilting and sliding, when the water deep of the lower
pond is
h a ,2 = 0m

Use the equations given in Finnish standard Vesirakenteiden suunnittelu


RIL 123-1979 [Design of Waterway Structures ].

Friction coefficient between rock and concrete (slightly fissured and varved rock,
RIL 123-1979 , p. 64)
m = 0,700

d
w

Water deep of the upper pond


hy = 9,000 m
s
a
Dimensions
h = 9,500 m
ht = 2,000 m hy h
a = 1,000 m w
d = 2,500 m
c = 0,500 m ht r ha,1
a = 30 °
O
x
c a
y
Figure 1.
4. Problem, Solution 32

Unit weight of reinforced concrete


gc = 25 000 N/m3
Unit weight of water
gw = 9 810 N/m3

Width of the dam as function of "deep" t (Figure 2)


t (1)
b( t )  d 
3
Width at bottom surface
b p = b (t = h ) (2)
= 7,985 m

w
d
t
gw a

b(t)
hy
F1 w
gw
F2,1
e1
a ha,1
bp
O hagw
e2,1
hygw

Figure 2.

WATER PRESSURE OF THE UPPER POND

Resultant (Figure 2)
1 (3)
F1  g w h2y
2
= 397 305 N/m
4. Problem, Solution 33

and corresponding distance from the bottom surface


e 1 = h y /3 (4)
= 3,000 m

WATER PRESSURE OF THE LOWER POND

Horizontal component of the resultant (Figure 2)


F 2,1h = g w h a, 12cosa /2 (5)
= 38 231 N/m
and corresponding distance from point O
e 2,1h = h a, 1/3 (6)
= 1,000 m

Vertical component of the resultant


F 2,1v = g w h a, 12sina /2 (7)
= 22 073 N/m
and corresponding distance from point O
e 2,1v = e 2,1h tana (8)
= 0,577 m

BUOYANCY AT THE SIDE OF THE UPPER POND

Resultant (RIL 123-1979. Chapter 2. Figure 7.) (Figure 3)



    

F3, i  0,4 h y  ha , i  h y  0,4 h y  ha , i g w a  c 
1
 2  (9)
F 3,1 = 83 876 N/m
F 3,2 = 92 705 N/m
4. Problem, Solution 34

Corresponding distance from the surface of upper pond water


0,4 h y  ha ,i a 2 c  12 hy  0,4hy  ha,i a 3 c
e 3, i  c
0,4h y  ha ,i   h y  0,4h y  ha ,i 
1 (10)
2
e 3,1 = 0,105 m
e 3,2 = 0,143 m

x -coordinate
x 3,i = e 3,I -b p (11)
x 3,1 = -7,880 m
a

a w

c gw r ha,1
e3,i

F4,i hagw
hygw e4,i
F3,i 0,4(hy-ha)g w

Figure 3.

BUOYANCY AT THE SIDE OF THE LOWER POND

Resultant (RIL 123-1979. Chapter 2. Figure 7.) (Figure 3)


 1
   
 
F4, i  ha , i  0,4 h y  ha , i  ha , i g w b p  d 
 2  (12)
F 4,1 = 145 277 N/m
F 4,2 = 96 851 N/m
4. Problem, Solution 35

Corresponding distance from the point O


bp  d
ha , i
2

1
2
   2
 
0,4 h y  ha , i  ha , i b p  d
3

e4, i 
1
 
ha , i  0,4 h y  ha , i  ha , i
2
  (13)

e 4,1 = 2,641 m
e 4,2 = 3,657 m

SELF WEIGHT OF THE DAM

Resultant (Figure 4)
G = [hd +h (b p -d )/2]g c (14)
= 1 245 073 N/m

Corresponding distance from the point O


g   d 1 2 
eG  c hd  b p    h b p  d b p  d 
G  2 2
   
3  (15)
= 5,125 m

eG
gc
G h
bp
F1 bp-e3,2
e1 e4,2
F3,2
F4,2 O

Figure 4.
4. Problem, Solution 36

ANSWER: Resultants and x - and y -coordinates are given in Table 1.

Table 1.
F x [N/m] y [m] F y [N/m] x [m]
Water pressure Upper pond 397 305 -3,000
Lower pond -38 231 1,000 22 073 -0,577
Buoyancy Upper pond side -83 876 -7,880
Lower pond side -145 277 -2,641

Safety against sliding (RIL 123-1979 , Chapter 2.42, p. 63)


Mp GeG
n 
 M k F1e1  F3,2 (b p  e3,2 )  F4,2e4,2 (16)
= 2,807 > 1,5 O.K.

Safety against tilting (sill of dam is ignored)


m  V  N 
n
H (17)

n
 
m G  ( F3,2  F4,2 )
F1 (18)
= 1,860 < 2 Sliding!
20121105 37
5. PROBLEM

The end supports of the bridge shown in Figure 1 are founded on rock and the
middle support on sand. Due to loading, the middle support sinks
d = 0,010 m.

Determine the bending moment and shear force distribution curve of the bridge
due to dead load before and after sinking.

Total length of the bridge


L = 16 m

Bending stiffness of the bridge


2
D = 120 MNm

Dead load (self weight)


g = 0,010 MN/m

D = EI
L/2 L/2

Figure 1.
5. Problem, Solution 38

In the statically determinate basic structure, the sum of the deflection (d 10) due to
the external load (g ) and deflection (-X 1d 11) due to the unknown reaction force (X 1)
has to be equal to the total deflection at the middle point. Here, d 11 is deflection due
to force 1 acting at the middle point. See Figure 2.
d 10-X 1d 11 = d (1)

1
g
+ =
D D D
X1 X1
L/2 L/2 L/2 L/2 L/2 L/2

x X1d11 x x
+ =
d10 d11
v v v d

Figure 2.

Bending moment due to force 1 acting at the middle point is determined in


Figure 3.
1

L/2 L/2

x  {0...L/2}

M(x) = Ax
A = 1/2

x  {L/2...L}

1
L/2 M(x) = Ax-(x-L/2)
A = 1/2

Figure 3.
5. Problem, Solution 39

Bending moment is (Figure 4)


x  L
 2 , x  0, 2 
  
M1  
 L  x , x   L , L (2a, b)
 2 2 
 
0 2 4 6 8 10 12 14 16 18
0
1 x [m]
2
3
4
5 M [m]

Figure 4.

Bending moment due to the dead load is determined in Figure 5.

L/2 L/2

x  {0...L/2}

g
M(x) = Ax-gx2/2
A = gL/2

Figure 5.

Bending moment is (Figure 6)


g
M 0  Lx  x 2 ,
2
  x  0,L
(3)
5. Problem, Solution 40

0 2 4 6 8 10 12 14 16 18
0,0
0,1 x [m]

0,2
0,3
0,4 M [MNm]

Figure 6.

Deflections
 L
M M
d 10   1 0 dx
 0
D
 L
 M1 M1
d 11   D dx
(4a, b)
 0
When inserting the bending moments we get
 5 gL4
d 10 
 384 D
 (5a, b)
 L3
d 
 11 48 D
d 10 = 0,071 m

d 11 = 0,711 m/MN

Reaction force
d  di
X 1,i  10
d 11 (6)
Before sinking (i = b )
db = 0,000 m
X 1,b = 0,100 MN
After sinking (i = a )
X 1,a = 0,086 MN
5. Problem, Solution 41

Bending moment
M i ( x )  M 0  X 1,i M1 (7)
g

 2
 x  L
 2 Lx  x  X 1,i 2 , x  0, 2 
 


 2
 
 g Lx  x 2  X L  x , x   L ,L
1, i
2 2 
 
(8a, b)

Shear force
dM
Qi ( x ) 
dx (9)
g X1,i  L
 L  2 x   x  0, 
,
2 2  2

 g L  2 x   X1,i L  (10a, b)
, x   , L
 2 2 2 

ANSWER:

The bending moment distribution curve before (b ) and after (a ) sinking,


respectively, is shown in Figure 7.

-0,10 Mb
Ma
-0,05
0 2 4 6 8 10 12 14 16 18
0,00
x [m]
0,05

0,10 Mi [MNm]

Figure 7.
5. Problem, Solution 42

The shear force distribution curve before (b ) and after (a ) sinking, respectively, is
shown in Figure 8.

-0,06
-0,04
-0,02
0 2 4 6 8 10 12 14 16 18
0,00
0,02 x [m]

0,04 Qb

0,06 Qi [MN] Qa

Figure 8.
20111205 43
6. PROBLEM

Determine the deflection curve of the simply supported bridge due to the train load
shown in Figure 1! Compare Eurocode: EN 1991-2 Traffic Loads on Bridges ,
Chapter 6.3.2, Load Model 71, when factor
a = 1,46

Distance
a0 = 3m

Length of the bridge


L = 16 m

Bending stiffness of the bridge


2
D = 12 000 MNm

Axel load of the engine


F = 0,350 MN

Cars (arbitrary length)


q = 0,120 MN/m
Distance
b = 0,800 m

4F
a0 q

8b
L

Figure 1.
6. Problem, Solution 44

When integrated, the differential equation of the deflection is


 M ( x)
v'' ( x ) 
EI (1)
1 x
 v' ( x ) 
D 0
 M ( x )dx  A   ( x )
(2)
1 xx
 v( x )    M ( x )(dx )2  Ax  B (3)
D 00

POINT LOAD (P = 1) ACTING AT POINT a

a 1

x  {0...a}

M(x) = Ax
A = (L-a)/L

x  {a...L}

a 1
M(x) = Ax-(x-a)
A = (L-a)/L
Figure 2.

Bending moment (Figure 2)


x
 L  L  a , x  0... a
M ( x)  
 a  L  x , x  a ... L (4a, b)
 L
6. Problem, Solution 45

Deflection as a function of x
v1 ( x ), x  0...a
v( x )  
v2 ( x ), x  a ...L (5a, b)
 x2
 a  L  A1 , x  0...a
 2 DL
 ax  x  2 L  A , x  a ...L (6a, b)
 2 DL 2

v1 ( x ), x  0...a
 v( x )  
v2 ( x ), x  a ...L (7a, b)
 x3
 a  L  A1 x  B1 , x  0...a
 6 DL
 
 a x 3  3 Lx2  A x  B , x  a ...L
 6 DL 2 2
(8a, b)

Boundary conditions
v 1' (a ) = v 2' (a ) (9)
v 1(0) = 0 (10)
v 2(L ) = 0 (11)
v 1(a ) = v 2(a ) (12)
Integration constants from boundary conditions 9, 10, 11 and 12
 a
 2
 A1  6 DL 2 L  3 La  a
2
 (13)

 B1  0 (14)


 A  a 2 L2  a 2
 2 6 DL 
 (15)
  a3
 B2  6 D
(16)
Deflection curve from point load F acting at point x = a
 F
 
 6 DL a  L x  a ( 2 L  3 La  a ) x , x  0... a
3 2 2

vF ( x )  
 
 F ax 3  3 Lax 2  a ( 2 L2  a 2 ) x  La 3 , x  a ... L
 6 DL
(17a, b)
6. Problem, Solution 46

UNIFORMLY DISTRIBUTED LOAD (q = 1) BETWEEN x = a AND x = L

a
x 1 1 ...

(L-a)/2
R = L-a

A = (L-a)(L-a)/2L B

x = [0, a]

M(x) = Ax
A = (L2-2La+a2)/2L

x = [a, L]
x-a
a

A M(x) = Ax-(x-a)2/2

Figure 3.

Bending moment (Figure 3)


 x 2
 
 2 L L  2 La  a , x  0... a
2
(18a, b)
M ( x)  
  
 1  Lx 2  L2  a 2 x  La 2 , x  a ... L
 2 L 
6. Problem, Solution 47

Deflection as a function of x
v1 ( x ), x  0...a
v ( x )  
v2 ( x ), x  a... L (5a, b)
 x2
  
 L2  2 La  a 2  A1 , x  0...a
  4 DL
  
 1 2 Lx 3  3 L2  a 2 x 2  6 La 2 x  A ,
 12 DL 2  x  a... L
(19a, b)

v1 ( x ), x  0...a
 v( x )  
v2 ( x ), x  a... L (7a, b)
 x3
  
 L2  2 La  a 2  A1 x  B1 , x  0...a
 12 DL
 (20a, b)

  
 1 Lx 4  2 L2  a 2 x 3  6 La 2 x 2  A x  B ,
 24 DL 2 2 

 x  a... L
Integration constants from boundary conditions 9, 10, 11 and 12
 1
 A1  24 DL L4  4L2a 2  4La 3  a4  (21)


 B1  0 (22)


 A2 
1
24 DL
L4  4L2a 2  a4  (23)

 a4
 B2  24 D
(24)
Deflection curve due to uniformly distributed load q
 q

2 2 3
 24 DL 2  L  2 La  a x  


 L4  4 L2a 2  4 La 3  a 4 x , x  0... a


vq ( x )   (25a, b)
 q

 24 DL
  
Lx 4  2 L2  a 2 x 3  6 La 2 x 2 

 
 L4  4 L2a 2  a 4 x  La 4 , x  a ... L
 
6. Problem, Solution 48

SUPERPOSITION

Table 1. Location of the point loads and the uniformly distributed load,
respectively.
ai
m
Point load F 1 a 1 = a 0+b = 3,800
Point load F 2 a 2 = a 0+3b = 5,400
Point load F 3 a 3 = a 0+5b = 7,000
Point load F 4 a 4 = a 0+7b = 8,600
Uniform load q a 5 = a 0+8b = 9,400

Inserting a 1…a 4 (Table 1) in Equation 17 and a 5 in Equation 25 and by using


superposition principle, the total deflection curve is obtained (Figure 4).

ANSWER: Deflection curve is shown in Figure 4.

-0,002
0 2 4 6 8 10 12 14 16
0,000

0,002
x [m]
0,004

0,006

0,008

0,010
F1 F2
0,012
F3 F4
0,014 v [m] q S

Figure 4. Deflection curve.


20111205 49
7. PROBLEM

Determine the position of the service vehicle (distance a ) so that the mid-support
reaction force of the double-span bridge, shown in Figure 1, is maximized!

Traffic load representing service vehicle is accidental loading model for footbridges
given in Eurocode (EN 1991-2 § 5.6.3).

Length of the bridge


L = 16 m

Bending stiffness of the bridge


2
D = 120 MNm

Uniformly distributed load on the whole bridge due to dead weight of bridge
p = 0,009 MN/m

Axel loads of the service vehicle


F1 = 0,080 MN
F2 = 0,040 MN

Distance between axels


b = 3,000 m

F1 F2 F3
a b c
p

D = EI
L/2 L/2

Figure 1.
7. Problem, Solution 50

By removing the middle support, the statically determinate basic form is obtained
(Figure 2).
1 2 P=1 3
n
X2
L
P=1

d2n

d22

X2d22
X2

dn2

Figure 2.

At the mid-point (point 2), the sum of deflection d 2n caused by load P = 1 at point n
(value of x -coordinate) and deflection X 2d 22 caused by point load X 2 at point 2 has
to be equal to zero. Here, d 22 is the mid-point deflection due to point load P = 1
acting at the same point. Thus,
0  d 2n  X 2d 22 (1)
d
 X 2  2n
d 22 (2)
Minus-sign in Equation 1 means, that the reaction force is acting upwards.
According to Equation 2, reaction force X 2 is maximized, when deflection d 2n is
maximized.

Following from Maxwell's rule: mid-point deflection d 2n caused by point load P = 1


at point n is equal to deflection d n 2 at point n caused by point load P = 1 at the
mid-point. Thus,
d 2n = d n 2 (3)
7. Problem, Solution 51

Influence line of the reaction force (due to point load P = 1 moving over the bridge)
is obtained by using deflection curve d n 2 (due to point load P = 1 acting at the mid-
point).
d
X 2  n2
d 22 (4)

Deflection d n 2 = v (x ) is determined by Mohr's method.

Dividing bending moment by bending stiffness we get (Figure 3)


 x
M  2 D , x  0... L / 2 (5a, b)
( x)  
D  L  x , x  L / 2... L
 2 D
1 D

L/2 L/2

x = [0, L/2]
M(x)/D = Ax/D
A = 1/2

x = [L/2, L]
1
L/2 M(x)/D = [Ax-(x-L/2)]/D
A = 1/2

Figure 3.

Deflection curve is equal to the bending moment distribution curve M M/D caused by
load M/D (Figure 4)
v (x ) = M M/D (x ) (6)
7. Problem, Solution 52

L2
T
16D M/D
L/4D
x
e1 
3
x2
R1  x = [0, L/2]
4D
L x/2D
e2  x  R1
3 MM/D(x)
L2
R2  T e1
16D
2 L x = [L/2, L]
e3   x  
3 2
2x  L  L L/4D
(2x-L)/4D
R3  x  R3 (L-x)/2D
8D  2
R4
1 L MM/D(x)
e4   x  
2 2 R2 e4
T
L x L e3
R4  x 
2D  2 e2

Figure 4.

Deflection (as function of x ) due to point load P = 1 acting at the mid-point


Tx  R1e1 , x  {0... L / 2}
v( x )  
Tx  R2e2  R3e3  R4e4 , x  { L / 2... L} (7a, b)
 1
 3 2

 48 D  4 x  3 L x , x  {0... L / 2}

 
 1 4 x 3  12 Lx 2  9 L2 x  L3 , x  { L / 2... L}
 48 D
(8a, b)
7. Problem, Solution 53

Deflection (as function of x and F ) due to point load F i acting at the mid-point
 F
 3 2

 48 D  4 x  3 L x , x  {0... L / 2}
v( x, F )  
 
 F 4 x 3  12 Lx 2  9 L2 x  L3 , x  { L / 2... L}
 48 D
(9a, b)

Deflection at the mid-point due to point load P = 1 acting at the same point is
obtained from Equation 8
 L
d 22  v  x  
 2 (10)
L3

48 D (11)
Influence line of reaction force is obtained by inserting Equations 8 and 11 in
Equation 4. See Figure 5.
1
 3 2

 3  4 x  3 L x , x  {0...L / 2}
X 2 ( x)   L

 L3
 
 1 4 x 3  12 Lx2  9 L2 x  L3 , x  { L / 2...L} (12a, b)

0 2 4 6 8 10 12 14 16 18
0,0
x [m]
0,5

1,0

1,5 |X2| [-]

Figure 5.

Influence line of the reaction force due to axel load F i is


 Fi
 3 2

 3  4 x  3 L x , x  {0...L / 2}
X 2, Fi ( x )   L
 
 Fi 4 x 3  12 Lx2  9 L2 x  L3 , x  { L / 2...L}
 L3
(13a, b)
7. Problem, Solution 54

Reaction force due to uniformly distributed load p is obtained by multiplying


Equation 12 by load p and integrating over the bridge.

 4 x 3  3 L2 x dx 
L/2
p
X 2, p  
3
0 L

4 x 
L (14)
p 3 2 2 3
   12 Lx  9 L x  L dx
3
L/ 2 L
5
 pL (15)
8

By using iteration the maximum reaction force and the corresponding load position
is found (Equations 13 and 15, Table 1)
a = 7,050 m

Table 1.
x i [m] X 2,i [MN]
Point load F 1 x1 = a = 7,050 0,078
Point load F 2 x 2 = a +b = 10,050 0,036
Uniform load p 0,090
S 0,205

ANSWER: Position of the service vehicle: distance


a = 7,050 m
Reaction force
X2 = 0,205 MN
20111007 55
8. PROBLEM

Determine distribution of earth pressure acting against a basement wall and the
corresponding resultant force, when the angle of earth surface with respect to
horizontal plane is
A) bA = 0°
B) bB = 20 °

The filling is compressed followingly:


Compression machine Vibrating slab, 400 kg
Compression times 4
Layer height 0,350 m

Height of the wall


h = 2,500 m

Angle of internal friction of the earth


j = 34 °

Effective unit weight of soil


g =
3
0,019 MN/m

bi

j
h

Figure 1.
8. Problem, Solution 56

Table 1. Permanent earth pressure caused by filling behind the non-transferable


1
supporting structure (Figure 2) .

Compressing Weight Number Layer Fold Earth


machine of of com- height depth pressure
machine pressing h z p
times
[kg] [m] [m] [MN/m2]
Vibrating roller 3 000 6 0,400 0,500 0,019
Vibrating slab 400 4 0,350 0,500 0,016
Vibrating slab 100 4 0,200 0,500 0,012

p
z

Additional pressure

Earth pressure at rest

Figure 2. Earth pressure distribution curve.

1) Finnish planning instructions for Eurocode EN 1997-1, Geotekninen


suunnittelu RIL 207-2009 , Chapter 9.5.5.
8. Problem, Solution 57

Earth pressure acting on the non-transferable structure corresponds to pressure at


rest. It acts in horizontal direction.

A) Angle of earth surface is zero

Coefficient of earth pressure at rest


K o = 1-sinj (1)
= 0,441

Pressure at the bottom level of the wall without compressing (Figure 3)


po = Kogh (2)
2
= 0,021 MN/m

Resultant of earth pressure

1 (3)
Po  poh
= 2 0,026 MN/m

Distance of the resultant measured from the bottom of the wall


e o = h /3 (4)
= 0,833 m

p
P1 z
p1
P2 zo
h
R
e1
Po
e2
e
eo

po = Kogh
Figure 3.
8. Problem, Solution 58

Earth pressure at depth z after compressing (Figures 2 and 3)


2
p = 0,016 MN/m

Depth of the fold of the earth pressure distribution curve


z = 0,500 m

Depth of the lower edge of the additional pressure distribution curve (compare
with Equation 2)

(5)
p

zo = 1,910 m
K og
<h

Additional earth pressure at the fold, i.e. at depth z ,


p l = p -K o g z (6)
2
= 0,012 MN/m

Resultant force due to additional pressure above z


P 1 = p l z /2 (7)
R  P o + P 1 + P 2
= 0,003 MN/m

Distance of the resultant measured from the bottom of the wall

(8)
= h  2 2,167
e1  z m
3
Resultant force due to additional pressure under z
P 2 = p l (z o -z )/2 (9)
= 0,008 MN/m
8. Problem, Solution 59

Distance of the resultant measured from the bottom of the wall

(10)
= 1 m
1,530
e2  h  z  ( zo  z )
3
ANSWER: Earth pressure distribution curve and its resultant is shown in Figure 4.

0,00 0,01 0,02 0,03 0,04


0,0 p [MN/m2]…
0,5
1,0
1,5
2,0
2,5 p
3,0 h [m] R

Figure 4.

Resultant of the total earth pressure


R = P o+P 1+P 2 (11)
= 0,0375 MN/m

Distance of the resultant measured from the bottom of the wall

(12)
= P e +1,093 + P2e2
P1e1 m
e o o
R
8. Problem, Solution 60

B) Angle of earth surface differs from zero

Coefficient of earth pressure at rest (compare with Equation 1)


K oB = (1-sinj )(1+sinb ) (13)
= 0,592

Earth pressure at the base level (Figure 5)


p oB = K oB g h (14)
= 0,028 MN/m2

Resultant of earth pressure


(15)

= 0,035 MN/m
1
PoB  poB h
2
Distance of the resultant measured from the bottom of the wall
e oB = h /3 (16)
= 0,833 m

b
p
P1 z
p1
P2 zo
h
R
e1
P
e2
o e
eo
Figure 5.
po = Kogh
8. Problem, Solution 61

Depth of the lower edge of the additional pressure distribution curve (compare
with Equation 2)
(17)

= 1,424 m
p
zoB <
 h
K oBg
Additional earth pressure at the fold, i.e. at depth z ,
p lB = p -K oB g z (18)
2
= 0,010 MN/m

Resultant force due to additional pressure above z


P 1B = p lB z /2 (19)
= 0,003 MN/m

Distance of the resultant measured from the bottom of the wall


(20)

= 2,167 m
2
e1B  h  z
Resultant force due to additional
3 pressure under z
P 2B = p lB (z oB -z )/2 (21)
= 0,005 MN/m

Distance of the resultant measured from the bottom of the wall


(22)

= 1,692 m
1
e2 B  h  z  ( zoB  z )
3
8. Problem, Solution 62

ANSWER: Earth pressure distribution curve and its resultant is shown in Figure 6.

0,00 0,01 0,02 0,03 0,04 0,05


0,0
p [MN/m2]…
0,5
1,0
1,5
2,0
2,5 p
3,0 h [m] R

Figure 6.

Resultant of the total earth pressure


R = P oB +P 1B +P 2B (23)
= 0,0425 MN/m

Distance of the resultant measured from the bottom of the wall


(24)

= 1,012 m

P e + P e + P2 B e2 B
e  oB oB 1B 1B
R
20111205 63
9. PROBLEM

Determine distribution of active earth pressure (horizontal component) and the


corresponding resultant acting on the concrete caisson shown in Figure 1, when the
filling is see-sand!

Dimensions
a = 1 m
h1 = 2 m
h2 = 1 m
h3 = 6 m
h4 = 1 m

Inverse of tangent of the angle of earth surface respective to the horizontal plane
k = 10

Angle of internal friction of the earth


j = 38 °

Unit weight of soil


g =
3
0,018 MN/m
a 1:k
c
h1
w
h2
h2
g
j
h3

h4

Figure 1.

Direction: To calculate earth pressure, use three layers separated by the dashed
lines shown in Figure 1.
9. Problem, Solution 64

Angle of earth surface with respect to the horizontal plane (Figure 2)


1
 = arctan  
k (1)
2=
d j( )
3

= 5,711 °
Angle of structure at any layer with respect to the vertical plane (Figure 2)
a
a1 = arctan 
 h1  (2)
= 26,565 °
a2 = 0,000 °
a3 = 0,000 °

+

-

+d
a 1:k
 -d
+a -a
w a1 h1
a2 = 0
+a
2

g Pa
a3 = 0 Pav
Pah d -a
3 j
d +a
h4
a) b)
Figure 2.

Angle of wall friction, when the sliding surface is between the concrete strucure
and soil
3
d= j
4 (3)
= 28,500 °
9. Problem, Solution 65

Coefficient of the horizontal component of active earth pressure

K ahi = cos2 (j a i )
2
 sin(j  d ) sin(j   ) 
cos2a i  1 (4)
 cos(a i d ) cos(a i   ) 
K ah 1 = 0,074
K ah 2 = 0,203
K ah 3 = 0,203

Unit weight of soil above the ground water level


g1=g2 =
3
0,018 MN/m
Unit weight of soil under the ground water level (Table 1 at the end of this
problem)
g3 =
3
0,012 MN/m

Active earth pressure at the top (y ) and bottom (a ) of layer n , respectively,


  0, n = 1
  n1
 pny = 
  K n  (g i hi ), n  2
  i =1 , n N
 n
(5a, b)
p = K
 na n  (g i hi ), n  1
 i =1
2
p 1y = 0 MN/m
p 1a = K ah 1(g 1h 1) (6)
2
= 0,002675 MN/m
p 2y = K ah 2(g 1h 1) (7)
2
= 0,007312 MN/m
p 2a = K ah 2(g 1h 1+g 2 h 2) (8)
2
0,010968 MN/m
p 3y = K ah 3(g 1h 1+g 2 h 2) (9)
2
0,010968 MN/m
p 3a = K ah 3(g 1h 1+g 2 h 2+g 3h 3) (10)
2
= 0,025593 MN/m
9. Problem, Solution 66

h1 P1
h2 P2

e1
P e2
h3
e P3
e3

Figure 3.

Resultant of active earth pressure at layer n (Figure 3)


1
Pn = pny hn  ( pna  pny )hn
2 (11)
P1 = 0,003 MN/m
P2 = 0,009 MN/m
P3 = 0,110 MN/m

and corresponding distances from the base of the caisson

2 2
( h

n
3   i
)
pny n  pna  pny n  maxh , n = [1, n
h 1
max ]
en =  = 
( )
1 i n 1
pny  pna  pny  (12)
2 0, n = nmax
e1 = 7,667 m
e2 = 6,467 m
e3 = 2,600 m

ANSWER:

Resultant of active earth pressure


P = P 1+P 2+P 3 (13)
= 0,121 MN/m
9. Problem, Solution 67

Distance from the bottom of the caisson


nmax
 Pnen
e = =1
n
nmax (14)
 Pn
n =1
= 3,002 m

Earth pressure distribution curve and resultant is shown in Figure 4.

0,00 0,05 0,10 0,15


0
p [MN/m2]
2 P [MN/m]

8
p
10 h [m] P

Figure 4.
9. Problem, Solution 68

1)
Table 1. Estimation of coarse soils based on grain size.

Unit weight of soil Internal


Soil above under friction
the ground water angle
level
[kN/m ] [kN/m3]
3
f [°]
Fine sand Loose 15…17 9… 30
d 10 0,06 Normal 33
Tight 16…18 11 36
Sand Loose 16…18 10… 32
d 10 > 0,06 Normal 35
Tight 17…19 12 38
Gravel Loose 17…19 10… 34
Normal 37
Tight 18…20 12 40
Moraine Very loose 16…19 10…12 …34
Loose 17…20 10…12 …36
Normal 18…21 11…13 …38
Tight 19…23 11…14 …40
Compressed Blast stones 15…18 9…11 45
filling under the Crushed stone 19…22 11…13 42
foundation2) Gravel 18…21 11…13 40

1) Finnish Road Administration: Finnish standard of bridge foundation


engineering. TIEL 2172068-99. Helsinki 1999. 71 s. ISBN 951-726-583-2.
Table 1. p. 9.

2) To use these values it is required, that the work and materials fulfil the
requirements of the Finnish specifications for bridges (2.7.1.2/24/).
20111205 69
10. PROBLEM

Determine distribution of earth pressure against the concrete retaining wall shown
in Figure 1 and the corresponding resultant in two cases: active earth pressure and
earth pressure at rest, respectively. The filling material is sand.

Dimensions
a = 2,200 m
b = 5,200 m
c = 1,000 m
d = 1,000 m
h1 = 6,200 m
h2 = 2,100 m
Loading
2
q = 0,012 MN/m
Effective unit weight of soil
g =
3
0,018 MN/m
Angle of internal friction of the earth
j1 = 32 °
j2 = 34 °

a c
q

h1
j1

w
h2 d
j2
b

Figure 1.
10. Problem, Solution 70

Unit weight of soil above the ground water level


g1 =
3
0,018 MN/m

Unit weight of soil under the ground water level (Table 1 of Problem 9)
g2 =
3
0,011 MN/m

EARTH PRESSURE AT REST

Coefficient of earth pressure at rest


K oi = 1-sinj i (1)
Ko1 = 0,470
Ko2 = 0,441

Earth pressure at rest at the top (y ) and bottom (a ) of layer n , respectively,


(Figure 2)
  K onq , n  1
 
 pony    n 1 
  on   g i hi  q  , n  2
K
   i 1  , n N
 (2a, b)
p  n 
 K   g i hi  q  , n  1
 ona on  
  i 1 
p o1y = K o1q (3)
2
= 0,005641 MN/m
p o1a = K o1(g 1h 1+q ) (4)
2
= 0,058102 MN/m
p o2y = K o2(g 1h 1+q ) (5)
2
= 0,054484 MN/m
p o2a = K o2(g 1h 1+g 2h 2+q ) (6)
2
= 0,064666 MN/m
10. Problem, Solution 71

q
po1y

h1

po1a
po2y
h2
po2a

Figure 2.

Resultant of earth pressure at rest at layer n and the total sum of resultants
(Figure 3)
1
Pon  pony hn  ( pona  pony )hn
2 (7)
Po1 = 0,198 MN/m
Po2 = 0,125 MN/m
P o = P o 1+P o 2 (8)
= 0,323 MN/m
q
a c

h1
Po1
Po
eo eo1
Po2
h2 eo2

Figure 3.
10. Problem, Solution 72

Corresponding distances from the bottom of the retaining wall (Figure 3)


h 1
pony n  pona 
2 2
 pony  h3n  n hi , n  [1,
max
nmax  1]
eon    i  n 1
1

pony  pona 
2
pony  
0, n  nmax
(9)

eo1 = 4,350 m
eo2 = 1,020 m

nmax
 Poneon
eo  1
n (10)
nmax
 Pon
n1
= 3,059 m
10. Problem, Solution 73

ACTIVE EARTH PRESSURE

Angle of earth surface with respect to the horizontal plane (Figure 4)


b = 0°

Angle of sliding surface with respect to the vertical plane (Figure 4)


bac
a1   arctan 
 h1  (11)
= -17,879 °
a2 = 0,0000 °

+b

a c -b
q
+d

a1 -d
h1 +a -a

+a
Pa
Pav
h2 a2 = 0 Pah d -a

b +a

a) b)
Figure 4.

Angle of wall friction, when the sliding surface is not between the steel structure
and soil
d1 = j1 (12)
= 32,000 °
10. Problem, Solution 74

Angle of wall friction, when the sliding surface is between the concrete structure
and soil
3
d 2  j2
4 (13)
= 25,500 °

Coefficient of the horizontal component of the active earth pressure

K ahi  cos2j i a i 
2
 sin(j i d i ) sin(j i  b ) 
cos2a i  1 (14)
 cos(a i d i ) cos(a i  b ) 
K ah 1 = 0,293
K ah 2 = 0,229

Active earth pressure at the top (y ) and bottom (a ) of layer n , respectively,


  K nq , n  1
 
 pny    n 1 
, n  2
 K
  n  g i hi  q 
   i 1  , n N
 (15a, b)
 p  K  n g h  q , n  1
 na n  i i 
  i 1 
p 1y = K ah 1q (16)
2
= 0,003521 MN/m
p 1a = K ah 1(g 1h 1+q ) (17)
2
= 0,036263 MN/m
p 2y = K ah 2(g 1h 1+q ) (18)
2
= 0,028363 MN/m
p 2a = K ah 2(g 1h 1+g 2 h 2+q ) (19)
2
= 0,033664 MN/m
10. Problem, Solution 75

Resultant of active earth pressure at layer n and the total sum of resultants
(Figure 5)
1
Pn  pny hn  ( pna  pny )hn
2 (20)
P1 = 0,123 MN/m
P2 = 0,065 MN/m
P = P 1+P 2 (21)
= 0,188 MN/m
q
a c

h1
P1
P
e e1
P2
h2 e2

Figure 5.

Corresponding distances from the bottom of the retaining wall (Figure 5)

2 2
 h n
3   i

pny n  pna  pny n  maxh , n  [1, n
h 1
max ]
en    
 
1 i n 1
pny  pna  pny  (22)
2 0, n  nmax
e1 = 4,350 m
e2 = 1,020 m
nmax
 Pnen
e  n1 (23)
nmax
 Pn
n 1

= 3,199 m
10. Problem, Solution 76

ANSWER:

Earth pressure resultant and the corresponding distance from the bottom of the
retaining wall
Earth pressure at rest
Po = 0,323 MN/m
eo = 3,059 m
Active earth pressure
P = 0,188 MN/m
e = 3,199 m

Earth pressure distribution curve is shown in Figure 6.

0,00 0,05 0,10 0,15 0,20 0,25 0,30 0,35


0
p [MN/m2]
2 R [MN/m]

6 po
p
8
Po
10 h [m]
P

Figure 6.
20111205 77
11. PROBLEM

Determine the core figure of the homogenous isosceles triangle shown in Figure 1.

Dimensions
b = 2m
h = 3m
y
B

h
2r x
A
C
b

Figure 1.
11. Problem, Solution 78

The main coordinate system and the principal coordinate system directions are
combined.

Cross-sectional area
A = bh /2 (1)
2
= 3,000 m
Moment of inertia with respect to x - and y -axis
bh3
Ix 
36 (2)
4
= 1,500 m
hb 3
Iy 
48 (3)
4
= 0,500 m
Radius of gyration with respect to s -axis for homogenous surface area
Is
is 
A (4)
Thus
I
i x2  x
A (5)
2
= 0,500 m
Iy
i 2y 
A (6)
2
= 0,167 m
Coordinates of "convex corner points"
x Ap = -b /2 (7)
= -1,000 m
y Ap = -h /3 (8)
= -1,000 m
x Bp = 0,000 m
y Bp = 2h /3 (9)
= 2,000 m
11. Problem, Solution 79

Boundary line of the core figure


xip yip
0 1 x y (10)
i 2y i x2
 y ( x )  ax  b
(11)
where
xip i x2
a 
yip i 2y (12)

i x2
b  (13)
yip

Hence,
ai bi
y A(x ) = -3,000 x + 0,500 [m] [m]
y B(x ) = 0,000 x + -0,250 [m] [m]

The x -coordinate of the intersection point of lines i and j


bi  b j
xij 
a j  ai (14)

Numerical values of the intersection points are presented in Table 1.

Table 1.
ij x ij y ij
m m
AB 4,000 -1,000
Ax 6,000 0,000
Ay 0,000 -3,000
11. Problem, Solution 80

ANSWER:

Boundary lines of the core figure are shown in Figure 2.

4 y [m]

A
2 Ay
B
c-s

1 c-s
c-s

x [m]
0
-1,5 -1,0 -0,5 0,0 0,5 1,0 1,5

-1

-2

-3
Figure 2.
20111205 81
12. PROBLEM

A composite base slab lying on the rock consists of a steel band, which inner and
outer edges are elliptical, and casted concrete inside the steel band (Figure 1).
Determine the core figure of the slab before casting and after hardening!

Dimensions
a = pm
 3,142 m
b = p/2 m
 1,571 m
t = 1/pp m
 0,027 m
Modulus of elasticity of concrete (c ) and steel (s )
Ec = 30 000 MN/m2
E s = 210 000 MN/m2
Advice: Estimate the shape of the core figure by determine the core figure
boundary lines corresponding to the points A, B and C shown in Figure 1. Angle
a = p/8

y
a a
A t
B
b
a b-t x
C
L L
b b-t
t a-t a-t t t

L-L

Figure 1.
12. Problem, Solution 82

Area
Concrete
A c = p(a-t )(b-t ) (1)
= 15,099 m2
Steel
A s = pab-A c (2)
= 0,404 m2
Composite slab
A = A c +A s (3)
= 15,503 m2

Moment of inertia with respect to x- and y- axis


Concrete
I cx = p(a-t )(b-t )3/4 (4)
= 8,992 m4
I cy = p(b-t )(a-t )3/4 (5)
= 36,609 m4
Steel
I sx = pab 3/4-I cx (6)
= 0,571 m4
I sy = pba 3/4-I cy (7)
= 1,644 m4

STEEL BAND

Radius of gyration with respect x -axis and y- axis power two


i sx 2 = I sx /A s (8)
2
= 1,416 m
2
i sy = I sy /A s (9)
= 4,072 m2
12. Problem, Solution 83

Slope
k = tan(a ) (10)
 0,414

Coordinates of "convex corner points"


x Ap = 0,000 m
y Ap = b
= 1,571 m
 x Bp
2 2
yBp
  1 (11a, b)
 a2 b2
 yBp  kx Bp

 a 2b 2
 x Bp  (12)
(  ) k 2a 2  b 2

= 2,419 m
y Bp = 1,002 m
x Cp =a
= 3,142 m
y Cp = 0,000 m

Boundary line of the core figure


xip yip
0 1 x y (13)
i 2y i x2
 y ( x )  ax   (14)
where
xip i x2
a  (15)
yip i 2y

i x2
  (16)
yip
12. Problem, Solution 84

Hence,
ai i
y As(x ) = 0,000 x + -0,901 [m] [m]
y Bs(x ) = -0,839 x + -1,413 [m] [m]
When y ip = 0,
x = -i y 2/x ip (17)
x Cs = -1,296 m

The x -coordinate of the intersection point of lines i and j


i   j
xij 
a j  ai (18)
Numerical values of the intersection points are presented in Table 1.

Table 1.
ij x ij y ij
m m
AB -1,641 1,479
BC -1,296 0,992

Let's assume, that the core figure is an ellipse, which half rays are
n s = -x Cs
= 1,296 m
ms = -  As
= 0,901 m

An intersection of the core figure boundary line corresponding to point B and the
ellipse mentioned above
 x 2 y2
 o2  o2  1
 ns m s
 (19a, b)
 yo  a B xo   B
 ns2a B  B  ns ms ns2a B2  m s2   B2
 xo 
m s2  ns2a B2 (20)
12. Problem, Solution 85

xo = -0,998 m
yo = -0,575 m

Tangent of the ellipse at this point is


xo y
x o y1
ns2 ms2 (21)
m2 x m2
 y s o x s
ns2 yo yo (22)
= -0,839 x + -1,413 [m]
which is same as core figure boundary line y B (x ) obtained above.

ANSWER: Boundary lines of the core figure are shown in Figure 2.

3 y A
B
2
C
1 Ax
Bx
0 x
-4 -3 -2 -1 0 1 2 3 4 By
-1 Cy
Bxy
-2
pl
-3 appr.

Figure 2.

Note! The core figure is not similar with inner or outer edge of the steel band:
a /b = 2,000
(a-t )/(b-t ) = 2,018
n s /m s = 1,438
12. Problem, Solution 86

COMPOSITE SLAB

Axial stiffness
Concrete
Cc = EcAc (23)
= 452 984 MN
Steel
Cs = EsAs (24)
= 84 768 MN
Composite slab
C = C c +C s (25)
= 537 753 MN

Bending stiffness
Concrete
D cx = E c I cx (26)
2
= 269 751 MNm
D cy = E c I cy (27)
= 1 098 265 MNm2
Steel
D sx = E s I sx (28)
= 119 994 MNm2
D sy = E s I sy (29)
= 3,45E+05 MNm2
Composite slab
D x = D cx +D sx (30)
= 389 746 MNm2
D y = D cy +D sy (31)
2
= 1,44E+06 MNm

Radius of gyration with respect x -axis and y- axis power two


i x 2 = D x /C (32)
2
= 0,725 m
12. Problem, Solution 87

2
i y = D y /C (33)
= 2,684 m2

Boundary lines of the core figure


ai i
y A(x ) = 0,000 x + -0,461 [m] [m]
y B(x ) = -0,652 x + -0,723 [m] [m]
When y ip = 0,
xC = -0,854 m
Numerical values of the intersection points are presented in Table 2.

Table 2.
ij x ij y ij
m m
AB -2,489 1,149
BC -0,854 -0,034

Let's assume, that the core figure is an ellipse, which half rays are
n = -x C
= 0,854 m
m = -A
= 0,461 m

An intersection of the core figure boundary line corresponding to point B and the
ellipse mentioned above

  n 2a B  B  nm n 2a B2  m 2   B2
 xo 
 m 2  n 2a B2 (34a, b)

 yo  a B xo   B
x o = -0,658 m

y o = -0,294 m
12. Problem, Solution 88

Tangent of the ellipse at this point is


 m 2 xo m2
y x
n2 yo yo (35)
= -0,652 x + -0,723 [m]
which is same as core figure boundary line y B (x ) obtained above.

ANSWER: Boundary lines of the core figure are shown in Figure 3.

3 y A
B
2
C
1 Ax
Bx
0 x
-4 -3 -2 -1 0 1 2 3 4 By
-1 Cy
Bxy
-2
pl
-3 appr.

Figure 3.

Note! The core figure is not similar with inner or outer edge of the steel band:
a /b = 2,000
(a-t )/(b-t ) = 2,018
n /m = 1,852
20111205 89
13. PROBLEM

Determine the area, where vertical force can be acting so that all piles are
compressed (core figure)! The piles are in vertical direction and they all have
equal cross-section, material and length. See Figure 1 and Table 1.

In the answer use principal coordinate system (y , z ) or center-of-mass coordinate


system (y , z ) parallel to the given coordinate system (y 0, z 0)!

Dimensions
a = 1,5 m
d = 1,5 m

Table 1. Coordinates of the top of the piles. y0


Symbol i z 0i y 0i 1 2
Unit - m m a/2
Numerical 1 -1,500 0,750 z0
a/2
value 2 1,500 0,750
4 3
3 0,000 -0,750 d d
4 -1,500 -0,750
V

Figure 1.
13. Problem, Solution 90

It is supposed, that moments and product of inertia with respect to centroid line of
any pile are small and therefore can be neglected.
I z'i = 0 (1)
I y'i = 0 (2)
I y'z'i = 0 (3)
Cross-section area of pile i (without dimension)
Ai = 1 (4)
Cross-section area of the pile group
A = 4A i (5)
= 4
Center of gravity (centroid)
z 0p = Sz i A i /A (6)
= -0,375 m
y 0p = Sy i A i /A (7)
= 0,000 m
z -coordinate of piles in the center-of-mass coordinate system (y-z )
z i = z 0i -z 0p (8)
z1 = -1,125 m
z2 = 1,875 m
z3 = 0,375 m
z4 = -1,125 m
y -coordinate of piles in the center-of-mass coordinate system (y-z )
y i = y 0i -y 0p (9)
y1 = 0,750 m
y2 = 0,750 m
y3 = -0,750 m
y4 = -0,750 m
Modulus of inertia of the piles with respect to z -axis (Steiner's rule)
I zi = I z'i +A i y pi 2 (10)
Iz1 = 0,563 m2
Iz2 = 0,563 m2
Iz3 = 0,563 m2
Iz4 = 0,563 m2
13. Problem, Solution 91

Modulus of inertia of the piles with respect to y -axis


I yi = I y'i +A i z pi 2 (11)
Iy1 = 1,266 m2
Iy2 = 3,516 m2
Iy3 = 0,141 m2
Iy4 = 1,266 m2
Product of inertia of the piles in the center-of-mass coordinate system (y , z )
I yzi = I y'z'i +A i y pi z pi (12)
I yz 1 = -0,844 m2
I yz 2 = 1,406 m2
I yz 3 = -0,281 m2
I yz 4 = 0,844 m2
Moment and product of inertia for the pile group
I z = SI zi (13)
2
= 2,250 m
I y = SI yi (14)
2
= 6,188 m
I yz = SI yzi (15)
= 1,125 m2
Angle of principal direction
1 2 I yz
a  arctan
2 I y  Iz (16)
= 0,260 rad
= 14,872 °
Principal moments of inertia
I z = I z cos2a +I y sin2a -I yz sin(2a ) (17)
= 1,951 m2
I y = I z sin2a +I y cos2a +I yz sin(2a ) (18)
= 6,486 m2
Square of radius of gyration with respect to z - and y -axis
i z 2 = I z /A (19)
= 0,488 m2
13. Problem, Solution 92

2
i y = I y /A (20)
= 1,622 m2
z -coordinates of piles
z i = y i sina +z i cosa (21)
z1= -0,895 m
z2= 2,005 m
z3= 0,170 m
z4= -1,280 m
y -coordinates of piles
y i = y i cosa z i sina (22)
y1= 1,014 m
y2= 0,244 m
y3= -0,821 m
y4= -0,436 m
Equation of core figure boundary line in the principal coordinate system
 ip y ip
0 1  y (23)
iy2 i2
 y ( )  A  B
(24)
where
 ip i2
A
y ip iy2 (25)

i2
B (26)
y ip
By inserting numerical values
Bi Ai
y 1(z ) = 0,266 z + -0,481 [m]
y 2(z ) = -2,475 z + -2,002 [m]
y 3(z ) = 0,062 z + 0,594 [m]
y 4(z ) = -0,883 z + 1,119 [m]
13. Problem, Solution 93

z -coordinate of the intersection point of lines i and j is


Bi  B j
 ij 
A j  Ai (27)
Numerical values of the intersection points are presented in Table 2.

Table 2.
ij z ij y ij z ij y ij
m m m m
12 -0,555 -0,629 -0,375 -0,750
23 -1,023 0,530 -1,125 0,250
34 0,555 0,629 0,375 0,750
41 1,393 -0,111 1,375 0,250

Equation of core figure boundary line in the center-of-mass coordinate system


y (z ) = Cz +D (28)
where
C = tan(a +arctanA ) (29)
D = B (cosa +C sina ) (30)
By inserting numerical values
Ci Di
y 1(z ) = 0,571 z + -0,536 [m]
y 2(z ) = -1,333 z + -1,250 [m]
y 3(z ) = 0,333 z + 0,625 [m]
y 4(z ) = -0,500 z + 0,938 [m]
The z -coordinate of intersection point of lines i and j
Di  D j
zij 
C j  Ci (31)
Numerical values of intersection points are expressed in Table 2.
13. Problem, Solution 94

ANSWER: Boundary lines 1, 2, 3 and 4 of the core figure are shown in Figures
2 and 3, respectively.
3,0 y [m] 1 2
3 4
2,5
Pile Slab
2,0
1,5
1,0
0,5 z [m]
0,0
-3,0 -2,0 -1,0 0,0 1,0 2,0 3,0 4,0 5,0
-0,5
-1,0
-1,5
-2,0

Figure 2. Principal coordinate system.

3,0 y [m] 1 2
3 4
2,5
Pile Slab
2,0
1,5
1,0
0,5 z [m]
0,0
-3,0 -2,0 -1,0 0,0 1,0 2,0 3,0 4,0 5,0
-0,5
-1,0
-1,5
-2,0

Figure 3. Center-of-mass coordinate system.


13. Problem, Solution 95

ALTERNATIVE WAY (Compare Problem number 29.)

Centroid is calculated as above


z 0p = -0,375 m
y 0p = 0,000 m

Table 3.
Variable Pile Coordinates Angle Stiffness
Symbol i xi yi zi ai ki
Unit m m m ° -
Numerical 1 0 0,750 -1,125 0 1
value 2 0 0,750 1,875 0 1
3 0 -0,750 0,375 0 1
4 0 -0,750 -1,125 0 1

Variable Pile Cosine of directional angles Moment arms


Symbol i p x,i p y,i p z,i r x,i r y,i r z,i
Unit - - - m m m
Numerical 1 1,000 0,000 0,000 0,000 -1,125 -0,750
value 2 1,000 0,000 0,000 0,000 1,875 -0,750
3 1,000 0,000 0,000 0,000 0,375 0,750
4 1,000 0,000 0,000 0,000 -1,125 0,750

Variable Pile Elements of stiffness matrix of the piles


Symbol i k 11,i k 12,i k 13,i k 22,i k 23,i k 33,i
Unit 1 1,000 -1,125 -0,750 1,266 0,844 0,563
Numerical 2 1,000 1,875 -0,750 3,516 -1,406 0,563
value 3 1,000 0,375 0,750 0,141 0,281 0,563
4 1,000 -1,125 0,750 1,266 -0,844 0,563
Unit MN/m MN MN MNm MNm MNm
Numerical value 4,000 0,000 0,000 6,188 -1,125 2,250
Symbol k 11 k 12=k 21 k 13=k 31 k 22 k 23=k 32 k 33
Variable Elements of stiffness matrix of pile group
13. Problem, Solution 96

Elements of the stiffness matrix are calculated in Table 3.


 4,000 0,000 0,000 MN/m MN/m MN 
  MN/m
K    0,000 6,188 -1,125  MN/m MN 
 0,000 -1,125 2,250  MN MN MNm 
Inverse matrix
 0,250 0,000 0,000 m/N m/N 1/N 

K    0,000  m/N m/N 1/N 
1
0,178 0,089 
 0,000 0,089 0,489  1/N 1/N 1/Nm 
From equilibrium condition
{F } = [K ]{d } (32)
the displacement vector
{d } = [K ]-1{F } (33)
u  1  (x -axial vertical force [MN])
  1  
 j   K   z  (Moment with respect to y -axis [MNm]) (34)
q   y 
    (Moment with respect to z -axis [MNm])
u = 0,000 y + 0,000 z + 0,250

 j = -0,089 y + 0,178 z + 0,000
q = -0,489 y + 0,089 z +
 0,000
is obtained.
Pile force
Ni = kiDi (35)
= k i (p x,i u +r y,i j +r z,i q ) (36)
N1 = 0,467 y + -0,267 z + 0,250 =0
N2 = 0,200 y + 0,267 z + 0,250 =0
N3 = -0,400 y + 0,133 z + 0,250 =0
N4 = -0,267 y + -0,133 z + 0,250 =0
Equations of core figure boundary line in the center-of-mass coordinate system
y 1(z ) = 0,571 z + -0,536 [m]
y 2(z ) = -1,333 z + -1,250 [m]
y 3(z ) = 0,333 z + 0,625 [m]
y 4(z ) = -0,500 z + 0,938 [m]

ANSWER: As above (Figure 3).


20130917 97
14. PROBLEM

Determine the core figure of the two-component base slab shown in Figure 1.

Modulus of elasticity of concrete


E1 = 40000 MN/m2
E2 = 20000 MN/m2

Dimensions are
a = 1 m
b = 3 m
c = 1 m
d = 3 m
e = 4 m

y C D
A B

ex,1 ex,2 d/2


b/2
x
E1 E2
b/2
d/2

a
c
e

Figure 1.
14. Problem, Solution 98

Cross-sectional area of part i


A 1 = ab (1)
= 3,000 m2
A 2 = cd (2)
= 3,000 m2

Axial stiffness of part i


Ci = EiAi (3)
C 1 = 120 000 MN
C 2 = 60 000 MN

Axial stiffness for the whole cross-section


n
C = å Ci
i =1 (4)
= 180 000 MN

The main coordinate system and the principal coordinate system directions are
combined. Origin of the coordinate system is located at the center of gravity
weighted by modulus of elasticity.

Moment of inertia of a sub-section with respect to its own x '-axis going through the
the sub-section's center of gravity and being parallel with the principle axis x of the
whole cross-section
ab 3
I x ',1 =
12 (5)
= 2,250 m4
cd 3
I x ',2 =
12 (6)
= 2,250 m4
14. Problem, Solution 99

Correspondingly, with respect to y '-axis,


ba 3
I y',1 =
12 (7)
= 0,250 m4
dc 3
I y ', 2 =
12 (8)
= 0,250 m4

Steiner's rule: moment of inertia with respect to an arbitrary r -axis for sub-section
i is
I r,i = I r',i +A i e s,i 2 (9)
where I r ',i is moment of inertia with respect to r '-axis, which is parallel to the
x -axis going through the center of gravity, and e s,i is perpendicular distance
between r - and r '-axis.
Correspondingly, for a composite cross-section, the bending stiffness is
D r,i = D r',i +C i e s,i 2 (10)

Bending stiffness of the sub-sections with respect to x '- and y '-axis, respectively,
D r',i = E i I r',i (11)
D x', 1 = 2
90 000 MNm
D x', 2 = 45 000 MNm2
D y', 1 = 10 000 MNm2
D y', 2 = 5 000 MNm2

Bending moment equilibrium condition with respect to y -axis


C 1e x, 1 = C 2e x, 2 (12)
Þ C 1e x, 1 = C 2(e-e x, 1) (13)
Distances between y -axis and center of gravity of sub-section i
eC 2
e x ,1 =
C1 + C 2 (14)
= 1,333 m
e x, 2 = e-e x, 1 (15)
= 2,667 m
14. Problem, Solution 100

Bending stiffness of the sub-sections with respect to x - and y -axis, respectively, by


using Equation 11
D x, 1 = 2
90 000
MNm
D x, 2 = MNm2
45 000
D y, 1 = 2
223 333
MNm
D y, 2 = 2
431 667
MNm
Bending stiffness for the whole cross-section
n
Dr = å Dr ,i
i =1 (16)
Dx = 2
135 000 MNm
Dy = 655 000 MNm2
For a homogenous cross-section, the radius of gyration is
Is
is =
A (17)
Correspondingly, for a composite cross-section,
Ds
is =
C (18)
Thus
Dx
i x2 =
C (19)
2
= 0,750 m
Dy
i 2y =
C (20)
2
= 3,639 m
Coordinates of the "convex corner points"
x Ap = -e x, 1-a /2 (21)
= -1,833 m
y Ap = b /2 (22)
= 1,500 m
x Bp = -e x, 1+a /2 (23)
= -0,833 m
y Bp = b /2 (24)
= 1,500 m
14. Problem, Solution 101

x Cp = e x, 2-c /2 (25)
= 2,167 m
y Cp = d /2 (26)
= 1,500 m
x Dp = e x, 2+c /2 (27)
= 3,167 m
y Dp = d /2 (28)
= 1,500 m
Equation of core figure boundary line
xip yip
0 = 1+ x+ y (29)
i 2y i x2
Þ y ( x ) = ax + b (30)
where
xip i 2x
a =-
yip i 2y (31)

i x2 (32)
b =-
yip
By inserting numerical values
ai bi
y A(x ) = 0,252 x + -0,500 [m]
y B(x ) = 0,115 x + -0,500 [m]
y C(x ) = -0,298 x + -0,500 [m]
y D(x ) = -0,435 x + -0,500 [m]

The x -coordinate of the intersection point of lines i and j


bi - b j
xij =
a j - ai (33)
14. Problem, Solution 102

Numerical values of the intersection points are presented in Table 1.

Table 1.
ij x ij y ij
m m
Ax 1,985 0,000
Bx 4,367 0,000
Cx -1,679 0,000
Dx -1,149 0,000
AB 0,000 -0,500
BC 0,000 -0,500
CD 0,000 -0,500

ANSWER:

Boundary lines of the core figure are shown in Figure 2.

2,0 y [m]
A
1,5 Ax
(B)
1,0
(Bx)
0,5 x
(C)
[m]
0,0 (Cx)
-2,5 -1,5 -0,5 0,5 1,5 2,5 3,5 D
-0,5 Dx
-1,0 pl
pl
-1,5
pl
-2,0 pl

Figure 2.
20111205 103
15. PROBLEM

A multi-component bar consists of n nested pipes. At cross-section (x-y -plane)


inner and outer edges of each bands are ellipses and the principal coordinate
system directions in any part are congruent (Figure 1). The innermost area is solid.

Half ray, in direction of x - and y- axis, respectively, of the outer edge of band i is
a i > 0  r i R
b i > 0  r i R
The corresponding modulus of elasticity valid in that area is
E i  0  E i R
If E i = 0, no material exists. Here i N  i {1…n }.

Determine the core figure of the cross-section and apply the solution to a
homogenous bar and to an empty circular tube with infinitely thin wall thickness!

an

a3

bn
a1 b3
b1 x
En …Ei… E3 E2 E1 b2
a2
…bi…
…ai…

Figure 1.
15. Problem, Solution 104

Equation of the ellipse is (Figure 2)


x2 y2 (1)
 1
a2 b2
Equation of core figure boundary line, or neutral axis corresponding to load acting
at point (x p , y p ) is
xp yp
1 x y0
i 2y i x2 (2)
 y  x   (3)
where
x p i x2
 
y p i 2y
(4)
i x2
 
yp (5)
y
b

y = x+ (xp, yp)


d (xq, yq)
y = x+ x
-a -c (xo, yo) c a
-d

-b

Figure 2.

Let's assume, that the shape of the core figure of ellipse area (boundary given in
Equation 1) is ellipse
x2 y2
 1
c2 d2 (6)
15. Problem, Solution 105

Tangent of that core figure ellipse at point (x q , y q ) is


xq yq
x y1
c2 d2 (7)
 y  x   (8)
where
xq d 2
 
yq c 2 (9)
d2

yq (10)

Equation of core figure boundary line of ellipse (Equation 1) corresponding to point


(a , 0) is
a
1 x0
i 2y (11)
where from half ray of the core figure ellipse, parallel to x- axis, at point (-c, 0) is
obtained
i 2y
c
a (12)

Respectively, core figure boundary line corresponding to point (0, b ) is


b
1 y0
i x2 (13)
where from half ray of the core figure ellipse, parallel to y- axis, at point (0, -d ) is
obtained
i x2
d
b (14)

By using half ray c and d the Equations 4 and 5 can be written in form
x p bd
 
y p ac (15)
bd
 
yp (16)
15. Problem, Solution 106

Equation of core figure boundary line (Equation 3) of ellipse (Equation 1)


corresponding to point (x p , y p ) and core figure ellipse (Equation 6) have an
intersection (x o , y o )

 xo2 yo2
 2  2 1
c d
 y  x   (17a, b)
 o o
  c 2   cd q
 xo 
 d 2  c 2 2
 (18a, b)
 d 2   cd q
y
 o 
 d 2  c 2 2
where
q  d 2  c 2 2   2 (19)
2 2
2 2 
x p bd   bd 
  
y 2p
qd c  (20)
 y p ac   y p  b 2d 2
   
y 2p y 2p x 2p
 q  1
b 2d 2 b2 a2 (21)
q0 (22)

Or intersection (from Equation 18 and 22) is

  c 2 
 xo  2
 d  c 2 2
 (23a, b)
 d 2
y
 o 
 d 2  c 2 2

Because point (x q , y q ) is an arbitrary point of the core figure ellipse (Equation 6),
we can substitute
 xq  xo


 yq  yo
 (24a, b)
15. Problem, Solution 107

Gamma (from Equations 9, 24 and 23) is


 c 2 d 2  c 2 2 d 2
 
d 2  c 2 2 d 2 c2 (25)
  (26)

Delta (from Equations 10, 24b, 23b and 15) is


d 2  c 2 2
  d2
d 2 (27)
 2
d 2  c 2  x p bd  
  1 (28)
  d 2  y p ac  
 

d2  x 2p b 2 
  1   (29)
  2 2
yp a 

Because from Equation 1 we obtain


x 2p b 2 b2
1 (30)
y 2p a 2 y 2p

so (from Equation 16)


d 2 b2

 y 2p (31)
   (32)

Thus Equation of core figure boundary line (Equation 3) of ellipse (Equation 1)


corresponding to point (x p , y p ) and the tangent of core figure ellipse (Equation 8)
are congruent. So, the core figure of ellipse area is an ellipse area.

Half ray (in direction of x- axis) of the core figure ellipse of the multi-component
bar is (Equation 12)
Dy
c
anC (33)
15. Problem, Solution 108

n
 Ei I yi
 c  i 1 (34)
n
an  E i Ai
i 1

 Ei bi ai3  bi 1ai31 


n

c i 1 (35)
n
4an  E i ai bi  ai 1bi 1 
i 1
Here D y is bending stiffness with respect to y- axis and C is axial stiffness of the
whole cross-section. Respectively I yi is moment of inertia and A i area of band i .

Respectively half ray in direction of y -axis is (Equation 14)


Dx
d
bnC (36)
n
 Ei I xi
d  i 1
n (37)
bn  E i Ai
i 1

 Ei ai bi3  ai 1bi31 


n

d  i 1 (38)
n
4bn  E i ai bi  ai 1bi 1 
i 1

In the case of homogenous solid cross-section n = 1 and


a
c 1
4 (39)
b
d 1
4 (40)
15. Problem, Solution 109

Or ellipses are equiform


a1 b1
 4
c d (41)

In the case of homogenous band n = 2 and E 1 = 0 and


b2 a23  b1a13
c
4a2 a2 b2  a1b1  (42)
a2 b23  a1b13
d
4b2 a2 b2  a1b1  (43)

In the case of very thin circular band b i = a i and a 1  a 2 and


c  a2
 r
d 2 (44)
where r is radius of the core figure circle.

ANSWER:

The core figure of the multi-component bar is ellipse area, which equation in the
principal coordinate system is
2 2
n  n 
  E i ai bi  ai 1bi 1    E i ai bi  ai 1bi 1 
  2  i 1  2
16an2  i 1 x  16bn2 y 1 (45)
2 2
n
   
  
n
  E i bi ai  bi 1ai 1    E i ai bi  ai 1bi 1 
3 3 3 3
 i 1   i 1 

In the case of homogenous solid cross-section the equation of the core figure is
(Figure 3)
16 2 16 2
x  y 1
a12 b12 (46)
15. Problem, Solution 110

In the case of very thin circular band the equation of the core figure is (Figure 4)
2 2 a22
x y 
4 (47)

a1
a1
4 b1 b1
4
x

Figure 3.

y a2

a b2 = a2
r 2
2

Figure 4.
20111205 111
16. PROBLEM

Vertical force and moment


F zo = 2 MN
M xo = 0,350 MNm
act on a base slab lying on rock.

A) Find the minimum side width a so that the whole base slab is compressed!
What is the corresponding maximum stress?

B) Find the minimum side width a so that the maximum stress in rock is
(RIL 121-2004 , Chapter 5.5.3.1, p. 82)
s max = 10 MN/m2

C) Find the minimum side width a in case when the base slab is anchored by
using vertical rock anchors located as shown in Figure 1, where distance
e = 0,10 m
and the maximum stress in rock is
s max = 10 MN/m2
How big is the corresponding anchor force?

a/2 a/2
Fzo
a/2 Mxo
a/2 e z y

Figure 1.
16. Problem, Solution 112

A)

Normal stress (Figure 2)


Fz M x
s ( y)   y
A Ix (1)

x
a/2 a/2
Fzo
a/2
Mxo
a/2 y

smin = 0
z
smax

Figure 2. Stress distribution, a three-side prism.

Cross-section area
2
A =a (2)
Moment of inertia
I x = a 4/12 (3)
From Equations 1, 2 and 3
F M
s ( y )  zo2
 12 xo y
a a4 (4)
Normal stress at the edge
 a
s min  s  y  0
 2  (5)
F M
 0  zo  6 xo
2
a a3 (6)
Side width
M xo
a6
Fzo (7)
= 1,050 m
16. Problem, Solution 113

Maximum stress
æaö
s max = s ç ÷
è 2ø (8)
F M
= zo + 6 xo
a2 a3 (9)
= 3,628 MN/m2

Normal stresses at the edge as a function of side width is shown in Figure 3.

10 min
s
[MN/m2] max
5
a [m]
0
0,0 0,5 1,0 1,5 2,0 2,5 3,0
-5

-10

Figure 3.

B)

a/2 a/2
Fzo Mxo

smin = 0
smax
a/2-b/3
b/3
b

Figure 4.
16. Problem, Solution 114

Equilibrium conditions (Figure 4)


  Fz  0

 M x  0
(10a, b)

 ab
0  Fzo  2 s max
  (11a, b)
0  M xo   a  b  ab s max
  2 3 2
 2Fzo
b  as
 max

  ( ) 4 Fzo3
2
  M xo M xo  (12a, b)
a   3 s max
  Fzo

Side width a and effective width b


a = 0,720 m
b = 0,555 m

Maximum stress as a function of side width is shown in Figure 5.

35 s
30 [MN/m2]
25
20
15
10
5 a [m]
0
0,0 0,5 1,0 1,5 2,0

Figure 5.
16. Problem, Solution 115

C)

Equilibrium conditions (Figure 6)


 R  Fzo  P

 a  (13a, b)
 M xo  P  2  e 
  

a/2 a/2
Fzo Mxo
y
e
P
smax
R = smaxa2
Figure 6.

Anchor force P from Equation 13b


M
P  xo
a
e (14)
2

Anchor force P is inserted in Equation 13a

 a  2e 
M xo
s maxa 2  Fzo 
a (15)
e
2
 s maxa 2 a  2e   Fzo a  2e   2 M xo (16)
 s maxa 3  2es maxa 2  Fzo a  2eFzo  2 M xo  0 (17)

Expression at the left hand side of Equation 17 has value


0,000 MNm
when
a = 0,609 m
16. Problem, Solution 116

Anchor force from Equation 14


P = 1,711 MN

The value of the left hand side of Equation 17 as function of side width a is shown
in Figure 7.
10 f [MNm]

5
a [m]
0
-1,5 -1,0 -0,5 0,0 0,5 1,0 1,5
-5

-10

Figure 7.

ANSWER:

A) The minimum side width, when the whole slab is compressed, is


a = 1,050 m.
The corresponding maximum stress is
s max = 3,628 MN/m2.

B) The minimum side width, when the maximum stress in rock is


s max = 10 MN/m2,
is
a = 0,720 m.

C) The minimum side width, when anchors are used, is


a = 0,609 m.
The corresponding anchor force is
P = 1,711 MN
20111205 117
17. PROBLEM

Vertical force
N = 0,750 MN
acts on a base slab anchored by using prestressed vertical rock anchors
(Figure 1).

A) Determine the stress distribution, when the prestressing force is


P = 0 MN?

B) How big prestressing force is needed to obtain a uniform stress distribution


under the slab?

Dimensions of the base slab


x
a1 = 0,400 m
a2 = 0,800 m b/2 b/2
a3 = 0,600 m
e3
b = 2,000 m a3
L = 6,000 m
e1 e2 L/2

z y

2a1/2 2a2/2 L/2

a3

N
3P
y
A B C D

z
Figure 1.
17. Problem, Solution 118

Cross-section areas
A1 = a 1L (1)
2
= 2,400 m
A2 = a 2L (2)
2
= 4,800 m
A3 = a 3b (3)
2
= 1,200 m
A = A 1+A 2+2A 3 (4)
2
= 9,600 m

Origin is situated to the centroid.

Distances between x -axis and the centroid of the sub-section 1


A  0  A2 ( e1  e 2 )  2 A3 ( e1  e 3 )
e1  1
A (5)
a a  a b
A2  1  b  2   2 A3  1  
  2 2  2 2 (6)
A
= 1,600 m
Respectively
a a
e2  1  b  2  e1
2 2 (7)
= 1,000 m
a b
e3  e1  1 
2 2 (8)
= 0,400 m
17. Problem, Solution 119

A)

Moment of inertia with respect to x -axis (Steiner's rule)

La13 2 La 2
3  a b3 
Ix   A1e1   A2e2  2 3  A3e32 
2
(9)
12 12  12 
 
4
= 12,416 m

Bending moment with respect to x -axis


M x = Ne 2 (10)
= 0,750 MNm

Stress at point A
Mx N
 ( y)  y
Ix A (11)
M x   a1  N
 A    e1  
Ix  2  A (12)
2
= -0,031 MN/m < 0, tension

Tension is not possible. Stress at point B is studied. Part 1 is neglected.


A B = A 2+2A 3 (13)
2
= 7,200 m
a b
2 A3  2  
e2 B   2 2 (14)
AB
= 0,467 m
a b
e3 B  2   e2 B
2 2 (15)
= 0,933 m

La23  a b3 
I xB   A2 e 22B  2 3  A3 e 32B  (16)
12  12 
 
4
= 4,192 m
17. Problem, Solution 120

M xB = Ne 2B (17)
= 0,350 MNm
M xB   b  N
B    e3 B  
I xB  2  AB (18)
2
= -0,057 MN/m < 0, vetoa (a)

Position of the neutral axis


c n = 3a 2/2 (19)
= 1,200 m > a2 (b)

2a2/2
N
cn
y

z R max
Figure 2.

According to notes (a) and (b), the neutral axis is located between points B and C.

Stress distribution is shown in Figure 3.

x
a3
N
L V2
y
2a2/2
a3
V1

1
z, 
c

Figure 3.
17. Problem, Solution 121

Equilibrium condition of vertical forces


V  0 (20)
 N  V1  V2 (21)
1 1  c  a2 
N  Lc 1  ( L  2a3 )(c  a2 )  1
2 2  c  (22)
2 Nc
 1 
Lc  ( L  2a3 )(c  a2 )2
2 (23)

Equilibrium condition of moment with respect to point D


M  0 (24)
a c  c  a2 
 N 2  V1  V2   a2 
2 3  3  (25)
a 1 1
 N 2  Lc2 1  ( L  2a3 )(c  a2 )2 (c  2a2 ) 1
2 6 6c (26)


 N 2  1 Lc3  ( L  2a3 )(c  a2 )2 c  2a2 
a
2 6c
 (27)

Following from Equations 23 and 27


a2 Lc 3  ( L  2a3 )( c  a2 )2 c  2a2 

2 3 Lc 2  3( L  2a3 )( c  a2 )2 (28)

From an iteration process (compare to the solution of cubic equation at the end of
this problem)
a 2/2 = 0,400 m
when
c = 1,600 m

From Equation 23
1 =
2
0,195 MN/m
17. Problem, Solution 122

ANSWER:

Stress distribution is similar to Figure 3, where


c = 1,600 m
1 =
2
0,195 MN/m

B)

Stress distribution is uniform, when the force resultant is acting at the centroid
(Figure 4).
Ne2  3 Pe1 (29)
Ne2
 P
3e1 (30)
= 0,156 MN

e2
e1 N
3P
y

z
Figure 4.

ANSWER:

Prestressing force needed is


P = 0,156 MN
17. Problem, Solution 123

SOLUTION OF CUBIC EQUATION

Cubic equation is
a c 3b c 2g c d 0 (I)
Exploiting
l2 3
m 
4 27 (II)
where

3ag  b 2
 (III)
3a 2
2 b 3  9ab g  27a 2d
l (IV)
27a 3
If m < 0, the cubic equation has three real roots
   2 n b
cn  2 cos  , n  1, 2, 3
3 3 3a (Va, b, c)
where

l  27 
  arccos  (VI)
 2  3 
If m = 0, the cubic equation has two inequal real roots
l b
c1  2  3 
2 3a
c2  l b (VIIa, b)
3 
c3  2 3a
where the last one is a double root.
Furthermore, if l = 0, the equation has one real triple root.

If m > 0, the cubic equation has one real root


l l b
c3  m 3  m 
2 2 3a (VIII)
and two complex roots.
17. Problem, Solution 124

Equation 20 can be written in the form of Equation I, where


a = 4a 3 (IX)
= 2,400 m
b = –6a 2a 3 (X)
2
= -2,880 m
g =
3
0,000 m
d = a 23(2a 3-L ) (XI)
4
= -2,458 m
Hence
 =
2
-0,480 m
l =
3
-1,152 m
m =
6
0,328 m
c = 1,600 m
See Figure I.

6 f(c) [m4]

0 c [m]
-1,0 -0,5 0,0 0,5 1,0 1,5 2,0
-2

-4

-6

-8

-10

Figure I.
20111205 125
18. PROBLEM

Determine stability against tilting and sliding of the retaining wall shown below
(Figure 1). The structure is lying on rock. Use safety factory analysis!

Dimensions
a = 1,000 m
b = 2,000 m
h = 5,000 m
t = 0,500 m

Superimposed load
q = 0,010 MN/m2

Angle of internal friction of the soil


j = 36,000 °

Unit weight of soil


g = 0,018 MN/m3

Unit weight of reinforced concrete


gc = 0,025 MN/m3

Friction coefficient between rock and concrete


m = 0,750
q

j
t
g h
gc
a b
m

Figure 1.
18. Problem, Solution 126

When the structure is lying on rock, the minimum safety factory against tilting and
sliding is (Finnish standards of foundation engineering, RIL 121-2004 , Chapter
5.1.2.2, p. 58)
n = 1,500

Coefficient of earth pressure at rest


K o = 1-sinj (1)
= 0,412

Resultant of earth pressure due to soil mass and the corresponding distance from
the base of the retaining wall, respectively, (Figure 2)
1
Pg  K 0gh2
2 (2)
= 0,093 MN/m
e g = h /3 (3)
= 1,667 m

Resultant of earth pressure due to superimposed load and the corresponding


distance from the base of the retaining wall, respectively, (Figure 2)
P q = K 0q h (4)
= 0,021 MN/m
eq = h /2 (5)
= 2,500 m

Pq
h

Pg

K0gh K0q

Figure 2.
18. Problem, Solution 127

Overturning moment (Figure 4)


Mk = Pgeg+Pqeq (6)
= 0,206 MNm/m

Weight resultant of the retaining wall and the corresponding distance from the
front corner point, respectively,
G c = g c [t (h -a )+ab ] (7)
= 0,100 MN/m
t b 
g c  t ( h a ) ab
ec  
2 2  (8)
Gc
= 0,625 m

Q
G

h
Gc
a
b

Figure 3.

Weight resultant of soil and the corresponding distance from the front corner
point, respectively, (Figure 3)
G = g (b -t )(h -a ) (9)
= 0,108 MN/m
b t
et
2 (10)
= 1,250 m
18. Problem, Solution 128

It is assumed, that the superimposed load q is not acting above the retaining wall
(dashed line in Figure 1). This is the main loading case. The superimposed load
resultant and the corresponding distance from the front corner point, respectively,
(Figure 3)
Q = 0·qb (11)
= 0,000 MN/m
eQ = b /2 (12)
= 1,000 m

Balancing moment (Figure 4)


M p = G c e c +Ge+Qe Q (13)
= 0,198 MNm/m

Mp Mk

Figure 4.

Safety against tilting


Mp (14)
nt 
Mk
= 0,958 <n= 1,500 Turning over!

The retaining wall is anchored by using vertical rock anchors located as shown in
Figure 5, where distance
c = 0,100 m
18. Problem, Solution 129

Vertical force capacity needed is (from Equation 14)


P (b  c )  M p
n
Mk (15)
nM k  M p
 P
bc (16)
 0,059 MN/m

P Mp Mk
b

Figure 5.

Safety against sliding


mV
ns 
H (17)
m Gc  G  Q 

Pg  Pq (18)
= 1,376 <n= 1,500 Sliding!

The retaining wall has to be anchored by using vertical prestressing force. From
safety against sliding
m (V  P ) (19)
n
H  Qp
Here P is anchor force and Q is shear capacity of the anchor.
18. Problem, Solution 130

If it is assumed that
Qp = 0
then the anchor force required is
nH  mV
P
m (20)
n( Pg  Pq )  m (Gc  G  Q )
P
m (21)
P 0,019 MN/m

ANSWER:

Conditions for tilting and sliding are not satisfied. That is why the retaining wall
has to be anchored by using vertical prestressing force
P = 0,060 MN/m
located at a distance
c = 0,100 m
from the edge of the base slab.
20111007 131
19. PROBLEM

A base slab have a breadth a (Figure 1). Determine the smallest possible
dimension a so that the bearing resistance is adequate! Use design approach 2
of ultimate limit state (GEO) (Chapter 2.4.7.3.4.3) and Annexes A (Chapter. A.3)
and D of Eurocode SFS-EN1997-1!

Characteristic values of loading due to the overhead structure


Vo = 1,500 MN
Ho = 0,130 MN
Mo = 0,240 MNm
Proportion of the permanent load from the total load is
kV = 0,400
kH = 0,000
kM = 0,400
and the rest is variable one. The proportion of the variable load of vertical load V o
and moment M o are caused by effective load and horizontal load H o by wind.

Unit weight of reinforced concrete


gc = 0,025 MN/m3 V0
Unit weight of soil M0
H0
gm = 0,020 MN/m3

Angle of internal friction of the earth h


t
j = 30,000 °

Dimensions
c = 1,500 m gm gc j
c
h = 2,200 m d
d = 0,600 m
a/2 a/2
t = 0,400 m
Figure 1.
Length of the base slab (perpendicular to the figure plane)
L = 4,000 m
In this direction the loading is central.
19. Problem, Solution 132

COMBINATIONS OF ACTIONS

Approach given by Eurocode SFS-EN 1997-1 is used.


Load combination and partial safety factors may be differs in national annex.
This is situation in Finnish national annex.

Combination of sets of partial factors of design approach 2


A1 "+" M1 "+" R2
where
– A refers to loads or actions of loads (Table A.3 of Appendix A),
– M refers to soil parameters (Table A.4),
– R refers to resistance (Table A.5),
– numbers 1 and 2 refer to sets 1 and 2 and
– "+" implies "to be combinated with".

Load combination
qd  g gi g i  g q1qk1  g q 2qk 2 (1)
can be written
1, 35 1,5 1,5 (a)
1, 35 1,5  0  (b)
  
1, 35 0  1,5 (c)
  
1, 35 0  0  (d)
qd   c
( g  g m )   k1
q   qk 2 (2a…h)
1,0  1,5 1,5 (e)
1,0  1,5 0  (f)
  
1,0  0  1,5 (g)
1,0  0  0  (h)

First the load case "a" is studied and after that the others.
19. Problem, Solution 133

LOAD CASE a

Lets select a- value.


a = 2,100 m
The value given here is the final result after the iteration.

Characteristic load values at the surface of the base slab are solved.
The sub indexes are referred to that ones used in Equation 1.

Self weight of the base structure (as function of a )


V gc = g c[(h +c-d )t +ad ]L (3)
= 0,250 MN

Self weight of soil (as function of a )


V gm = g m (a-t )(c-d )L (4)
= 0,122 MN

Permanent vertical load


Vg = kVV0 (5)
= 0,600 MN

Variable vertical imposed load


V q 1 = (1-k V )V 0 (6)
= 0,900 MN

Permanent horizontal load


Hg = kHH0 (7)
= 0,000 MN

Variable horizontal wind load


H q 2 = (1-k H )H 0 (8)
= 0,130 MN
19. Problem, Solution 134

Moment due to permanent loads


M g = k M M 0+k H H 0(c +h ) (9)
= 0,096 MNm

Moment due to imposed load


M q 1 = (1-k M )M 0 (10)
= 0,144 MNm

Moment due to wind load


M q 2 = (1-k H )H 0(c +h ) (11)
= 0,481 MNm

Design values of loads


Vertical load
V d = 1,35(V gm +V g +V gc )+1,5V q 1 (12)
= 2,663 MN
Horizontal load
H d = 1,35H g +1,5H q 2 (13)
= 0,195 MN
Moment
M d = 1,35M g +1,5M q 1+1,5M q 2 (14)
= 1,067 MNm

From equilibrium condition of the moments with respect to the pivot (Figure 2)
M d  Vd eo  0 (15)
the load eccentricity is obtained
Md
eo 
Vd (16)
= 0,401 m
19. Problem, Solution 135

a/2 a/2
Vd
Md

eo
R

b = 2(a/2-eo)
Figure 2.

Effective foundation length in the direction of a


a
b  2(  eo )
2 (17)
= 1,298 m

Effective foundation width (Figure 3)


b
B'  min
L (18)
= 1,298 m

L/2 L/2

L'/2 L'/2
B/2
eB B'/2
R B/2
eL B'/2

Figure 3.
19. Problem, Solution 136

Effective foundation length


b
L'  max
L (19)
= 4,000 m

Effective foundation area (Figure 3)


A' = B'L' (20)
= 5,194 m2

Design value of bottom pressure


V
qd  d
A' (21)
2
= 0,513 MN/m

Design value of cohesion


c' = 0 MN/m2

Unit weight of soil above the foundation level


g = gm (22)
3
= 0,020 MN/m

Unit weight of soil below the foundation level


g' = gm (23)
= 0,020 MN/m3

Effective overburden pressure at the level of the foundation area


q' = c g (24)
= 0,030 MN/m2

Inclination of the foundation base


a = 0°
19. Problem, Solution 137

Partial factor for angle of shearing resistance of the soil (EN 1997-1 ,
Annex A. Table A.4.)
gj = 1,000

Design value of the internal friction angle of the earth


 tanj 
j '  arctan 
 g  (25)
 j 
= 30,000 °

Bearing capacity factors


 j'
N q  tan 2  45   e tan j '
 2 (26)
= 18,401
  j' 
N g  2 tan 2  45  e tan j '  1 tan j '
  2  (27)
= 20,093

Factors for the inclination of the base


bq 
  (1  a tan j ' )
2
bg 
(28a, b)

= 1,000

Shape factors of the base slab


B'
sq  1  sinj '
L' (29)
= 1,162
B'
sg  1  0, 3
L' (30)
= 0,903
19. Problem, Solution 138

Parameter m
m  mB
(31)
B'
2
 L'
B' (32)
1
L'
= 1,755

Inclination factors of the load resultant


m
 Hd 
iq   1   (33)
 Vd  A' c ' cot j ' 

= 0,875
m 1
 Hd 
ig   1   (34)
 Vd  A' c ' cot j ' 

= 0,811

Partial resistance factor for bearing (EN 1997-1 , Appendix A, Table A.5)
gR = 1,4

Bearing capacity
R
qm 
A' (35)
1
 c' N c bc sc ic  q' N q bq sq iq  g ' B' N g bg sg ig
2 (36)
2
= 0,752 MN/m

Design value of the bearing capacity


q
qmd  m
gR (37)
2
= 0,537 MN/m
19. Problem, Solution 139

Ratio between bearing capacity and bottom pressure


n = q md /q d (38)
= 1,048 > 1, OK

Breadth of the base slab is obtained by an iteration process.


a = 2,100 m
The value is rounded up by precision of 0,1 m.

ALL LOAD CASES

Breadths of the base slab (a i ) in all loading cases are presented in Table 1.
There are presented only the quantities, that are function of a and/or partial
factors; all the other quantities are calculated above.

ANSWER: Breadth of the base slab


a = 2,400 m
19. Problem, Solution 140

Table 1.
Quant. Eq. Loading Cases Units
i a b c d e f g h
a 2,055 1,347 2,124 0,784 2,042 1,250 2,358 0,660 m
ext(a ) Max Min
a round 2,100 1,400 2,200 0,800 2,100 1,300 2,400 0,700 m
gg 2 1,35 1,35 1,35 1,35 1,00 1,00 1,00 1,00
g qk 1 2 1,50 1,50 0,00 0,00 1,50 1,50 0,00 0,00
g qk 2 2 1,50 0,00 1,50 0,00 1,50 0,00 1,50 0,00
V gc 3 0,247 0,205 0,251 0,171 0,247 0,199 0,265 0,164 MN
V gm 4 0,119 0,068 0,124 0,028 0,118 0,061 0,141 0,019 MN
Vd 12 2,655 2,529 1,317 1,078 2,315 2,210 1,006 0,782 MN
Hd 13 0,195 0,000 0,195 0,000 0,195 0,000 0,195 0,000 MN
Md 14 1,067 0,346 0,851 0,130 1,034 0,312 0,818 0,096 MNm
e0 16 0,402 0,137 0,646 0,120 0,446 0,141 0,812 0,123 m
b 17 1,251 1,074 0,832 0,544 1,149 0,968 0,733 0,414 m
B' 18 1,251 1,074 0,832 0,544 1,149 0,968 0,733 0,414 m
L' 19 4,000 4,000 4,000 4,000 4,000 4,000 4,000 4,000 m
A' 20 5,005 4,294 3,327 2,174 4,597 3,871 2,934 1,657 m2
qd 21 0,530 0,589 0,396 0,496 0,504 0,571 0,343 0,472 MN/m2
sq 29 1,156 1,134 1,104 1,068 1,144 1,121 1,092 1,052
sg 30 0,906 0,919 0,938 0,959 0,914 0,927 0,945 0,969
m 31 1,762 1,788 1,828 1,880 1,777 1,805 1,845 1,906
iq 32 0,874 1,000 0,746 1,000 0,855 1,000 0,672 1,000
ig 33 0,810 1,000 0,636 1,000 0,783 1,000 0,542 1,000
qm 35 0,743 0,824 0,554 0,694 0,705 0,799 0,480 0,661 MN/m2
q md 36 0,530 0,589 0,396 0,496 0,504 0,571 0,343 0,472 MN/m2
n 37 1,000 1,000 1,000 1,000 1,000 1,000 1,000 1,000
20111007 141
20. PROBLEM

A base slab have a breadth a (Figure 1). Determine the smallest possible
dimension a so that the bearing resistance is adequate! Use design approach 2 of
ultimate limit state (GEO) (Chapter 2.4.7.3.4.3) and Finnish Annexes A
(Chapter. A.3) and D of Eurocode SFS-EN1997-1 !

Characteristic values of loading due to the overhead structure


Vo = 1,500 MN
Ho = 0,130 MN
Mo = 0,240 MNm
Proportion of the permanent load from the total load is
kV = 0,400
kH = 0,000
kM = 0,400
and the rest is variable one. The proportion of the variable load of vertical load V o
and moment M o are caused by effective load and horizontal load H o by wind.

Unit weight of reinforced concrete


gc = 0,025 MN/m3 V0
M0
Unit weight of soil H0
gm = 0,020 MN/m3

Angle of internal friction of the earth h


t
j = 30,000 °

Dimensions
gm gc j
c = 1,500 m c
h = 2,200 m d
d = 0,600 m
a/2 a/2
t = 0,400 m
Figure 1.
Length of the base slab (perpendicular to the figure plane)
L = 4,000 m
In this direction the loading is central.
20. Problem, Solution 142

COMBINATIONS OF ACTIONS

Approach given by Eurocode SFS-EN 1997-1 is used.


Load combination and partial safety factors are from Finnish Annex.
These are different than guide given in Appendix A of SFS-EN 1997-1 .

Combination of sets of partial factors of design approach 2


A1 "+" M1 "+" R2
where
– A refers to loads or actions of loads (Table A.3 of Appendix A),
– M refers to soil parameters (Table A.4),
– R refers to resistance (Table A.5),
– numbers 1 and 2 refer to sets 1 and 2 and
– "+" implies "to be combinated with".

Factor for combination value of imposed load (Finnish Annex for SFS-EN 1990 ,
Table A1.1, Category A)
y 0,imposed = 0,7
Factor for combination value of wind load
y 0,wind = 0,6
thus
1,5y 0,imposed = 1,05
1,5y 0,wind = 0,90

When load factor


K FI = 1
then the load combination is
1,15  1,5 1,5
Fd  Gkj ,sup  0,9Gkj ,inf  Qk1  y 0 i Qki
1, 35 0  0  (1)
20. Problem, Solution 143

which can be written


1,15  1,5  0,9 (a)
1,15  1,5  0  (b)
  
1,15  1,05 1,5  (c)
  
1,15  0  1,5  (d)
1, 35 0  0  (e)
qd  g m   gc   qk 1   qk 2
0,9  1,5  0,9 (f)
(2a…j)
0,9  1,5  0  (g)
  
0,9  1,05 1,5  (h)
0,9  0  1,5  (i)
  
0,9  0  0  (j)

First the load case "a" is studied and after that the others.

LOAD CASE a

Lets select
a = 1,900 m
The value given here is the final result after the iteration.

Characteristic load values at the surface of the base slab are solved. The sub
indexes are referred to that ones used in Equation 1.

Self weight of the base structure and soil (as function of a )


V gc,m = g c[(h +c-d )t +ad ]L+ g m (a-t )(c-d )L (3)
= 0,346 MN

Permanent vertical load


Vg = kVV0 (4)
= 0,600 MN
20. Problem, Solution 144

Variable vertical imposed load


V q 1 = (1-k V )V 0 (5)
= 0,900 MN

Permanent horizontal load


Hg = kHH0 (6)
= 0,000 MN

Variable horizontal wind load


H q 2 = (1-k H )H 0 (7)
= 0,130 MN

Moment due to permanent loads


M g = k M M 0+k H H 0(c +h ) (8)
= 0,096 MNm

Moment due to imposed load


M q 1 = (1-k M )M 0 (9)
= 0,144 MNm

Moment due to wind load


M q 2 = (1-k H )H 0(c +h ) (10)
= 0,481 MNm

Design values of loads: vertical load, horizontal one and moment


V d = 1,15(V gc,m +V g )+1,5V q 1 (11)
= 2,438 MN
Hd = 1,15H g +0,9H q 2 (12)
= 0,117 MN
Md = 1,15M g +1,5M q 1+0,9M q 2 (13)
= 0,759 MNm
20. Problem, Solution 145

From equilibrium condition of the moments with respect to the pivot (Figure 2)
M d  Vd eo  0 (14)
the load eccentricity is obtained
Md
eo 
Vd (15)
= 0,311 m

a/2 a/2
Vd
Md

eo
R

b = 2(a/2-eo)

Figure 2.

Effective foundation length in the direction of a


a
b  2(  eo )
2 (16)
= 1,277 m

Effective foundation width (Figure 3)


b
B'  min
L (17)
= 1,277 m

Effective foundation length


b
L'  max
L (18)
= 4,000 m
20. Problem, Solution 146

L/2 L/2

L'/2 L'/2
B/2
eB B'/2
R B/2
eL B'/2

Figure 3.

Effective foundation area (Figure 3)


A' = B'L' (19)
= 5,108 m2

Design value of bottom pressure


V
qd  d
A' (20)
2
= 0,477 MN/m

Design value of cohesion


c' = 0 MN/m2

Unit weight of soil above the foundation level


g = gm (21)
= 0,020 MN/m3

Unit weight of soil below the foundation level


g' = gm (22)
3
= 0,020 MN/m

Effective overburden pressure at the level of the foundation area


q' = c g (23)
= 0,030 MN/m2
20. Problem, Solution 147

Inclination of the foundation base


a = 0°

Partial factor for angle of shearing resistance of the soil (Finnish annex for
EN 1997-1 , Application A. Table A.4.)
gj = 1,000

Design value of the internal friction angle of the earth


 tanj 
j '  arctan 
 g  (24)
 j 
= 30,000 °

Bearing capacity factors


 j'
N q  tan 2  45   e tan j '
 2 (25)
= 18,401
  j' 
N g  2 tan 2  45  e tan j '  1 tan j '
  2  (26)
= 20,093

Factors for the inclination of the base


bq 
  (1  a tan j ' )
2
bg 
(27a, b)

= 1,000

Shape factors of the base slab


B'
sq  1  sinj '
L' (28)
= 1,160
B'
sg  1  0, 3
L' (29)
= 0,904
20. Problem, Solution 148

Parameter m
m  mB
(30)
B'
2
 L'
B' (31)
1
L'
= 1,758

Inclination factors of the load resultant


m
 Hd 
iq   1   (32)
 Vd  A' c ' cot j ' 

= 0,917
m 1
 Hd 
ig   1   (33)
 Vd  A' c ' cot j ' 

= 0,873

Partial resistance factor for bearing (Finnish annex for EN 1997-1 , Appendix A,
Table A.5)
gR = 1,55

Bearing capacity
R
qm 
A' (34)
1
 c' N c bc sc ic  q' N q bq sq iq  g ' B' N g bg sg ig
2 (35)
2
= 0,790 MN/m

Design value of the bearing capacity


q
qmd  m
gR (36)
2
= 0,510 MN/m
20. Problem, Solution 149

Ratio between bearing capacity and bottom pressure


n = q md /q d (37)
= 1,068 > 1, OK

Breadth of the base slab is obtained by an iteration process.


a = a1 = 1,900 m
The value is rounded up by precision of 0,1 m.

ALL LOAD CASES

Breadths of the base slab (a i ) in all loading cases are presented in Table 1.
There are presented only the quantities, that are function of a and/or partial
factors; all the other quantities are calculated above.

ANSWER: Breadth of the base slab


a = 2,600 m
20. Problem, Solution 150

Table 1.
Quant. Eq. Loading Cases Units
i a b c d e f g h i j
a 1,839 1,377 2,110 2,292 0,834 1,803 1,302 2,128 2,517 0,658 m
ext(a ) Max Min
a round 1,900 1,400 2,200 2,300 0,900 1,900 1,400 2,200 2,600 0,700 m
gg 2 1,15 1,15 1,15 1,15 1,35 0,90 0,90 0,90 0,90 0,90
g qk 1 2 1,50 1,50 1,05 0,00 0,00 1,50 1,50 1,05 0,00 0,00
g qk 2 2 0,90 0,00 1,50 1,50 0,00 0,90 0,00 1,50 1,50 0,00
V gc,m 3 0,338 0,277 0,374 0,398 0,205 0,333 0,267 0,376 0,427 0,182 MN
V d 11 2,429 2,358 2,065 1,147 1,087 2,190 2,130 1,823 0,925 0,704 MN
H d 12 0,117 0,000 0,195 0,195 0,000 0,117 0,000 0,195 0,195 0,000 MN
M d 13 0,759 0,326 0,983 0,832 0,130 0,735 0,302 0,959 0,808 0,086 MNm
e 0 15 0,313 0,138 0,476 0,725 0,119 0,336 0,142 0,526 0,874 0,123 m
b 16 1,213 1,100 1,158 0,842 0,595 1,132 1,018 1,076 0,769 0,413 m
B' 17 1,213 1,100 1,158 0,842 0,595 1,132 1,018 1,076 0,769 0,413 m
L' 18 4,000 4,000 4,000 4,000 4,000 4,000 4,000 4,000 4,000 4,000 m
A' 19 4,853 4,400 4,631 3,369 2,382 4,527 4,071 4,304 3,077 1,651 m2
q d 20 0,500 0,536 0,446 0,341 0,456 0,484 0,523 0,424 0,301 0,426 MN/m2
s q 28 1,152 1,138 1,145 1,105 1,074 1,141 1,127 1,135 1,096 1,052
s g 29 0,909 0,917 0,913 0,937 0,955 0,915 0,924 0,919 0,942 0,969
m 31 1,767 1,784 1,776 1,826 1,870 1,779 1,797 1,788 1,839 1,906
i q 32 0,916 1,000 0,838 0,712 1,000 0,907 1,000 0,817 0,647 1,000
i g 33 0,872 1,000 0,759 0,591 1,000 0,858 1,000 0,730 0,511 1,000
q m 34 0,776 0,831 0,691 0,528 0,707 0,750 0,811 0,657 0,466 0,661 MN/m2
q md 36 0,501 0,536 0,446 0,341 0,456 0,484 0,523 0,424 0,301 0,426 MN/m2
n 37 1,000 1,000 1,000 1,000 1,000 1,000 1,000 1,000 1,000 1,000
20111007 151
21. PROBLEM

A base slab have a breadth a (Figure 1). Determine the smallest possible
dimension a so that the bearing resistance is adequate!
Use design approach DA2* of the ultimate limit state (GEO):
– Finnish Annex for Standard SFS-EN 1997-1 , Chapter 4 and Table A.3.
– SFS-EN 1997-1+AC , Annex D; or Formulary of the course, Chapter 2.1.

Characteristic values of loading due to the overhead structure


Vo = 1,500 MN
Ho = 0,130 MN
Mo = 0,240 MNm
Proportion of the permanent load from the total load is
kV = 0,400
kH = 0,000
kM = 0,400
and the rest is variable one. The proportion of the variable load of vertical load V o
and moment M o are caused by effective load and horizontal load H o by wind.

Unit weight of reinforced concrete V0


M0
gc = 0,025 MN/m3 H0
Unit weight of soil
gm = 0,018 MN/m3
Angle of internal friction of the earth h
t
j = 34,000 °

Dimensions
gm gc j
c = 1,500 m c
h = 2,200 m d
d = 0,600 m a/2 a/2
t = 0,400 m
Figure 1.
Length of the base slab (perpendicular to the figure plane)
L = 4,000 m
In this direction the loading is central.
21. Problem, Solution 152

COMBINATIONS OF ACTIONS

Approach given by Eurocode SFS-EN 1997-1 is used.


Load combination and partial safety factors are from Finnish Annex.
These are different than guide given in Appendix A of SFS-EN 1997-1 .

Combination of sets of partial factors of design approach 2


A1 "+" M1 "+" R2
where
– A refers to loads or actions of loads (Table A.3 of Appendix A),
– M refers to soil parameters (Table A.4),
– R refers to resistance (Table A.5),
– numbers 1 and 2 refer to sets 1 and 2 and
– "+" implies "to be combinated with".

Factor for combination value of imposed load (Finnish Annex for SFS-EN 1990 ,
Table A1.1, Category A)
y 0,imposed = 0,7
Factor for combination value of wind load
y 0,wind = 0,6
thus
1,5y 0,imposed = 1,05
1,5y 0,wind = 0,90

When load factor


K FI = 1
then the load combination is
1,15  1,5 1,5
Fd  Gkj ,sup  0,9Gkj ,inf  Qk1  y 0 i Qki
1, 35 0  0  (1)
21. Problem, Solution 153

which can be written


1,15  1,5  0,9 (a)
1,15  1,5  0  (b)
  
1,15  1,05 1,5  (c)
  
1,15  0  1,5  (d)
1, 35 0  0  (e)
qd  ( g c  g m )  qk1  qk 2
0,9  1,5  0,9 (f)
   (2a…j)
0,9 1,5 0 (g)
  
0,9  1,05 1,5  (h)
0,9  0  1,5  (i)
  
0,9  0  0  (j)

First the load case "a" is studied and after that the others.

LOAD CASE a

Lets select
a = 1,800 m
The value given here is the final result after the iteration.

Characteristic load values at the surface of the base slab are solved. The sub
indexes are referred to that ones used in Equation 1.

Self weight of the base structure and soil (as function of a )


V gc,m = g c[(h +c-d )t +ad ]L+ g m (a-t )(c-d )L (3)
= 0,323 MN

Permanent vertical load


Vg = kVV0 (4)
= 0,600 MN
21. Problem, Solution 154

Variable vertical imposed load


V q 1 = (1-k V )V 0 (5)
= 0,900 MN

Permanent horizontal load


Hg = kHH0 (6)
= 0,000 MN

Variable horizontal wind load


H q 2 = (1-k H )H 0 (7)
= 0,130 MN

Moment due to permanent loads


M g = k M M 0+k H H 0(c +h ) (8)
= 0,096 MNm

Moment due to imposed load


M q 1 = (1-k M )M 0 (9)
= 0,144 MNm

Moment due to wind load


M q 2 = (1-k H )H 0(c +h ) (10)
= 0,481 MNm

Characteristic values of loads: vertical load, horizontal one and moment


V k = V gc,m +V g +V q 1 (11)
= 1,823 MN
Hk = H g +H q 2 (12)
= 0,130 MN
Mk = M g +M q 1+M q 2 (13)
= 0,721 MNm
21. Problem, Solution 155

Design value of the vertical load


V d = 1,15(V gc,m +V g )+1,5V q 1 (14)
= 2,411 MN

From equilibrium condition of the moments with respect to the pivot (Figure 2)
M k -V k e o = 0 (15)
the load eccentricity is obtained
Mk
eo 
Vk (16)
= 0,396 m

a/2 a/2
Vd
Md

eo
R

b = 2(a/2-eo)

Figure 2.

Effective foundation length in the direction of a


a
b  2(  eo )
2 (17)
= 1,009 m

Effective foundation width (Figure 3)


b
B'  min
L (18)
= 1,009 m
21. Problem, Solution 156

L/2 L/2

L'/2 L'/2
B/2
eB B'/2
R B/2
eL B'/2

Figure 3.

Effective foundation length


b
L'  max
L (19)
= 4,000 m

Effective foundation area (Figure 3)


A' = B'L' (20)
= 4,035 m2

Design value of bottom pressure


V
qd  d
A' (21)
2
= 0,597 MN/m

Design value of cohesion


c' = 0 MN/m2

Unit weight of soil above the foundation level


g = gm (22)
= 0,018 MN/m3
21. Problem, Solution 157

Unit weight of soil below the foundation level


g' = gm (23)
3
= 0,018 MN/m

Effective overburden pressure at the level of the foundation area


q' = c g (24)
= 0,027 MN/m2

Inclination of the foundation base


a = 0°

Partial factor for angle of shearing resistance of the soil (Finnish annex for
EN 1997-1 , Application A. Table A.4.)
gj = 1,000

Design value of the internal friction angle of the earth


 tanj 
j '  arctan 
 g  (25)
 j 
= 34,000 °

Bearing capacity factors


 j'
N q  tan 2  45   e tan j '
 2 (26)
= 29,440
  j' 
N g  2 tan 2  45  e tan j '  1 tan j '
  2  (27)
= 38,366

Factors for the inclination of the base


bq 
  (1  a tan j ' )
2
bg 
(28a, b)

= 1,000
21. Problem, Solution 158

Shape factors of the base slab


B'
sq  1  sinj '
L' (29)
= 1,141
B'
sg  1  0, 3
L' (30)
= 0,924

Parameter m
m  mB
(31)
B'
2
 L'
B' (32)
1
L'
= 1,799

Inclination factors of the load resultant


m
 Hk 
iq   1   (33)
 Vk  A' c' cotj ' 

= 0,875
m 1
 Hk  (34)
ig   1  
 Vk  A' c' cotj ' 
= 0,813

Partial resistance factor for bearing (Finnish annex for EN 1997-1 , Appendix A,
Table A.5)
gR = 1,55
21. Problem, Solution 159

Bearing capacity
R
qm 
A' (35)
1
 c' N c bc sc ic  q' N q bq sq iq  g ' B' N g bg sg ig
2 (36)
2
= 1,056 MN/m

Design value of the bearing capacity


q
qmd  m
gR (37)
2
= 0,681 MN/m

Ratio between bearing capacity and bottom pressure


n = q md /q d (38)
= 1,140 > 1, OK

Breadth of the base slab is obtained by an iteration process.


a = 1,800 m
The value is rounded up by precision of 0,1 m.

ALL LOAD CASES

Breadths of the base slab (a i ) in all loading cases are presented in Table 1.
There are presented only the quantities, that are function of a and/or partial
factors; all the other quantities are calculated above.

ANSWER: Breadth of the base slab


a = 1,800 m
21. Problem, Solution 160

Table 1.
Quant. Eq. Loading Cases Units
i a b c d e f g h i j
a 1,707 1,063 1,588 1,794 0,663 1,642 1,007 1,519 1,706 0,540 m
ext(a ) Max Min
a round 1,800 1,100 1,600 1,800 0,700 1,700 1,100 1,600 1,800 0,600 m
g cm 2 1,15 1,15 1,15 1,15 1,35 0,90 0,90 0,90 0,90 0,90
g qk 1 2 1,50 1,50 1,05 0,00 0,00 1,50 1,50 1,05 0,00 0,00
g qk 2 2 0,90 0,00 1,50 1,50 0,00 0,90 0,00 1,50 1,50 0,00
g cmk 2 1 1 1 1 1 1 1 1 1 1
g qk 1k 2 1 1 1 0 0 1 1 1 0 0
g qk 2k 2 1 0 1 1 0 1 0 1 1 0
V gc,m 3 0,311 0,231 0,296 0,322 0,181 0,303 0,224 0,288 0,311 0,165 MN
V k 11 1,811 1,731 1,796 0,922 0,781 1,803 1,724 1,788 0,911 0,765 MN
H k 12 0,130 0,000 0,130 0,130 0,000 0,130 0,000 0,130 0,130 0,000 MN
M k 13 0,721 0,240 0,721 0,577 0,096 0,721 0,240 0,721 0,577 0,096 MNm
V d 14 2,398 2,305 1,976 1,060 1,054 2,163 2,091 1,744 0,820 0,689 MN
e 0 16 0,398 0,139 0,401 0,626 0,123 0,400 0,139 0,403 0,633 0,125 m
b 17 0,911 0,786 0,786 0,542 0,417 0,842 0,728 0,713 0,439 0,289 m
B' 18 0,911 0,786 0,786 0,542 0,417 0,842 0,728 0,713 0,439 0,289 m
L' 19 4,000 4,000 4,000 4,000 4,000 4,000 4,000 4,000 4,000 4,000 m
A' 20 3,643 3,143 3,142 2,167 1,666 3,368 2,913 2,851 1,757 1,155 m2
q d 21 0,658 0,734 0,629 0,489 0,633 0,642 0,718 0,612 0,467 0,596 MN/m2
s q 29 1,127 1,110 1,110 1,076 1,058 1,118 1,102 1,100 1,061 1,040
s g 30 0,932 0,941 0,941 0,959 0,969 0,937 0,945 0,947 0,967 0,978
m 32 1,815 1,836 1,836 1,881 1,906 1,826 1,846 1,849 1,901 1,933
i q 33 0,874 1,000 0,871 0,751 1,000 0,872 1,000 0,870 0,746 1,000
i g 34 0,811 1,000 0,808 0,645 1,000 0,809 1,000 0,806 0,640 1,000
q m 36 1,020 1,137 0,975 0,758 0,981 0,995 1,113 0,948 0,723 0,925 MN/m2
q md 37 0,658 0,734 0,629 0,489 0,633 0,642 0,718 0,612 0,467 0,596 MN/m2
n 38 1,000 1,000 1,000 1,000 1,000 1,000 1,000 1,000 1,000 1,000
20111007 161
22. PROBLEM

A base slab have a breadth a (Figure 1). Determine the smallest possible
dimension a so that the bearing resistance is adequate! Use the equation of bearing
capacity given in RIL 121-2004 ! Use ultimate limit state method given in
RIL 144-2002 !

Characteristic values of loading due to the overhead structure


Vo = 1,500 MN
Ho = 0,130 MN
Mo = 0,240 MNm
Proportion of the permanent load from the total load is
kV = 0,400
kH = 0,000
kM = 0,400
and the rest is variable one. The proportion of the variable load of vertical load V o
and moment M o are caused by effective load and horizontal load H o by wind.

Unit weight of reinforced concrete V0


M0
gc = 0,025 MN/m3 H0
Unit weight of soil
gm = 0,020 MN/m3
h
t
Angle of internal friction of the earth
j = 30,000 °

gm gc j
Dimensions c
c = 1,500 m d
h = 2,200 m a/2 a/2
d = 0,600 m
t = 0,400 m Figure 1.

Length of the base slab (perpendicular to the figure plane)


L = 4,000 m
In this direction the loading is central.
22. Problem, Solution 162

COMBINATIONS OF ACTIONS

Load combination (RIL 144-2002, Chapter 8.21, p. 149)


m n
qd   g gi gi g q1qk1  g q 2qk 2   g qi qki
i 1 i 3 (1)
can be written
1, 2  1,6 1,6 (a)
1, 2  1,6  0  (b)
  
1, 2  0  1,6 (c)
  
1, 2  0  0  (d)
qd  1,0 g c   g  qk1  qk 2
0,9 1,6 1,6 (e) (2a…h)
0,9 1,6 0  (f)
  
0,9 0  1,6 (g)
0,9 0  0  (h)
Partial safety factors are from books
– RIL 144-2002, Table 8.21a, and
– RIL 121-2004 , Chapter 5.1.2.3.

First the load case "a" is studied and after that the others.

LOAD CASE a

Lets select
a = 2,700 m
The value given here is the final result after the iteration.

Characteristic load values at the surface of the base slab are solved. The sub
indexes are referred to that ones used in Equation 1.
22. Problem, Solution 163

Self weight of the base structure and soil (as function of a )


V gc = g c[(h +c-d )t +ad ]L +g m (a-t )(c-d )L (3)
= 0,452 MN

Permanent vertical load


Vg = kVV0 (4)
= 0,600 MN

Variable vertical imposed load


V q 1 = (1-k V )V 0 (5)
= 0,900 MN

Permanent horizontal load


Hg = kHH0 (6)
= 0,000 MN

Variable horizontal wind load


H q 2 = (1-k H )H 0 (7)
= 0,130 MN

Moment due to permanent loads


M g = k M M 0+H g (c +h ) (8)
= 0,096 MNm

Moment due to imposed load


M q 1 = (1-k M )M 0 (9)
= 0,144 MNm

Moment due to wind load


M q 2 = (1-k H )H 0(c +h ) (10)
= 0,481 MNm
22. Problem, Solution 164

Design values of loads: vertical load, horizontal one and moment


V d = 1,0V gc +1,2V g +1,6V q 1 (11)
= 2,612 MN
Hd = 1,2H g +1,6H q 2 (12)
= 0,208 MN
Md = 1,2M g +1,6M q 1+1,6M q 2 (13)
= 1,115 MNm

From equilibrium condition of the moments with respect to the pivot (Figure 2)
M d  Vd eo  0 (14)
the load eccentricity is obtained
Md
eo 
Vd (15)
= 0,427 m
a/2 a/2
Vd
Md

eo
R

b = 2(a/2-eo)

Figure 2.

L/2 L/2

Lt/2 Lt/2
B/2
eB Bt/2
R B/2
eL Bt/2

Figure 3.
22. Problem, Solution 165

Effective foundation length in the direction of a


a
b  2(  eo )
2 (16)
= 1,846 m

Effective foundation width (Figure 3)


b
Bt  min
L (17)
= 1,846 m
Effective foundation length
b
L'  max
L (18)
= 4,000 m
Effective foundation area (Figure 3)
At = BtLt (19)
= 7,384 m2

Design value of bottom pressure


V
qd  d
At (20)
2
= 0,354 MN/m

Embedment depth
D =c (21)
= 1,500 m

Design value of cohesion


cd = 0 MN/m2

Unit weight of soil above the foundation level


g'1 = gm (22)
3
= 0,020 MN/m
22. Problem, Solution 166

Unit weight of soil below the foundation level


g'2 = gm (23)
3
= 0,020 MN/m

Partial safety factor for angle of shearing resistance (RIL 121-2004 , Table 7, p. 60)
gj = 1,250

Design value of the internal friction angle of the earth


 tanj 
j d  arctan   (24)
 gj 
 
= 24,791 °

Bearing capacity factors


j
N D  tan2 (45  d )e tanj d
2 (25)
= 10,431
  j  
N B  1,5tan2  45  d e tanj d  1 tanjd
  2   (26)
= 6,534

Shape factors of the base slab


B 
sB  1  0,4 t 
 Lt  (27)
= 0,815
B 
sD  1  0,2 t 
 Lt  (28)
= 1,092

Inclination factors of the load resultant


2
 Hd 
i D   1  
 Vd  At cd cot j d  (29)
= 0,847
22. Problem, Solution 167

4
 Hd 
i B   1  
 Vd  At cd cot j d  (30)
= 0,718

Design value of the bearing capacity


1
qmd  cd N c sc ic  g 1' DN D s D iD  g 2' Bt N B sB iB
2 (31)
= 0,360 MN/m2

Ratio between bearing capacity and bottom pressure


n = q md /q d (31)
= 1,018 > 1, OK

Breadth of the base slab is obtained by an iteration process.


a = 2,700 m
The value is rounded up by precision of 0,1 m.

ALL LOAD CASES

Breadths of the base slab (a i ) in all loading cases are presented in Table 1.
There are presented only the quantities, that are function of a and/or partial
factors; all the other quantities are calculated above.

ANSWER: Breadth of the base slab


a = 2,800 m
22. Problem, Solution 168

Table 1.
Quant. Eq. Loading Cases Units
i a b c d e f g h
a 2,673 1,763 2,666 0,890 2,630 1,666 2,787 0,755
ext(a ) Max Min
a round 2,700 1,800 2,700 0,900 2,700 1,700 2,800 0,800
gm 2 1,00 1,00 1,00 1,00 1,00 1,00 1,00 1,00
gg 2 1,20 1,20 1,20 1,20 0,90 0,90 0,90 0,90
g qk 1 2 1,60 1,60 0,00 0,00 1,60 1,60 0,00 0,00
g qk 2 2 1,60 0,00 1,60 0,00 1,60 0,00 1,60 0,00
V gc 3 0,448 0,328 0,447 0,213 0,442 0,315 0,463 0,195 MN
Vd 11 2,608 2,488 1,167 0,933 2,422 2,295 1,003 0,735 MN
Hd 12 0,208 0,000 0,208 0,000 0,208 0,000 0,208 0,000 MN
Md 13 1,115 0,346 0,885 0,115 1,086 0,317 0,856 0,086 MNm
e0 15 0,428 0,139 0,758 0,124 0,448 0,138 0,853 0,118 m
b 16 1,818 1,485 1,150 0,643 1,733 1,390 1,081 0,520 m
Bt 17 1,818 1,485 1,150 0,643 1,733 1,390 1,081 0,520 m
Lt 18 4,000 4,000 4,000 4,000 4,000 4,000 4,000 4,000 m
At 19 7,271 5,941 4,599 2,573 6,931 5,559 4,322 2,080 m2
qd 20 0,359 0,419 0,254 0,362 0,349 0,413 0,232 0,353 MN/m2
sB 27 0,818 0,851 0,885 0,936 0,827 0,861 0,892 0,948
sD 28 1,091 1,074 1,057 1,032 1,087 1,069 1,054 1,026
iD 29 0,847 1,000 0,675 1,000 0,836 1,000 0,628 1,000
iB 30 0,717 1,000 0,456 1,000 0,698 1,000 0,395 1,000
q md 31 0,359 0,419 0,254 0,362 0,350 0,413 0,232 0,353 MN/m2
n 32 1,000 1,000 1,000 1,000 1,000 1,000 1,000 1,000
20111110 169
23. PROBLEM

A base slab have a breadth a (Figure 1). Determine the smallest possible
dimension a so that the bearing resistance is adequate! Use the equation of bearing
capacity given in RIL 121-2004 ! Use total safety method when phi factor is
n = 2

Characteristic values of loading due to the overhead structure


Vo = 1,500 MN
Ho = 0,130 MN
Mo = 0,240 MNm
Proportion of the permanent load from the total load is
kV = 0,400
kH = 0,000
kM = 0,400
and the rest is variable one. The proportion of the variable load of vertical load V o
and moment M o are caused by effective load and horizontal load H o by wind.

Unit weight of reinforced concrete


gc = 0,025 MN/m3 V0
M0
H0
Unit weight of soil
gm = 0,020 MN/m3
h
t
Angle of internal friction of the earth
j = 30,000 °

gm gc j
Dimensions b = 1,500 m c
h = 2,200 m d
d = 0,600 m
a/2 a/2
t = 0,400 m
Figure 1.
Length of the base slab (perpendicular to the figure plane)
L = 4,000 m
In this direction the loading is central.
23. Problem, Solution 170

COMBINATIONS OF ACTIONS

Load combination (RIL 144-2002, Chapter 8.21, p. 149)


m n
qd   gi qk1  qk 2   qki
i 1 i 3 (1)
can be written
1 1 (a)

1 0 (b)
qd  g  qk1  qk 2
0 1 (c) (2a…d)
0 0 (d)

First the load case "a" is studied and after that the others.

LOAD CASE a

Lets select
a = 2,300 m
The value given here is the final result after the iteration.

Characteristic load values at the surface of the base slab are solved. The sub
indexes are referred to that ones used in Equation 1.

Self weight of the overhead structure, base structure and soil (as function of a )
V g = k V V 0+g c[(h +c-d )t +ad ]L +g m (a-t )(c-d )L (3)
= 0,999 MN

Variable vertical imposed load


V q 1 = (1-k V )V 0 (4)
= 0,900 MN
23. Problem, Solution 171

Permanent horizontal load


Hg = kHH0 (5)
= 0,000 MN

Variable horizontal wind load


H q 2 = (1-k H )H 0 (6)
= 0,130 MN

Moment due to permanent loads


M g = k M M 0+H g (c +h ) (7)
= 0,096 MNm

Moment due to imposed load


M q 1 = (1-k M )M 0 (8)
= 0,144 MNm

Moment due to wind load


M q 2 = (1-k H )H 0(c +h ) (9)
= 0,481 MNm

Design values of loads: vertical load, horizontal one and moment


V d = V g +V q 1 (10)
= 1,899 MN
Hd = H g +H q 2 (11)
= 0,130 MN
Md = M g +M q 1+M q 2 (12)
= 0,721 MNm

From equilibrium condition of the moments with respect to the pivot (Figure 2)
M d  Vd eo  0 (13)
23. Problem, Solution 172

the load eccentricity is obtained


Md
eo 
Vd (14)
= 0,380 m

a/2 a/2
Vd
Md

R eo

Bt = 2(a/2-eo)

Figure 2.

Effective foundation length in the direction of a


a
b  2(  eo )
2 (15)
= 1,541 m

Effective foundation width (Figure 3)


b
Bt  min
L (16)
= 1,541 m

Effective foundation length


b
L'  max
L (17)
= 4,000 m
23. Problem, Solution 173

L/2 L/2

Lt/2 Lt/2
B/2
eB Bt/2
R B/2
eL Bt/2

Figure 3.

Effective foundation area (Figure 3)


At = BtLt (18)
= 6,162 m2

Design value of bottom pressure


V
qd  d
At (19)
2
= 0,308 MN/m

Embedment depth
D =c (20)
= 1,500 m

Design value of cohesion


cd = 0 MN/m2

Unit weight of soil above the foundation level


g'1 = gm (21)
3
= 0,020 MN/m

Unit weight of soil below the foundation level


g'2 = gm (22)
= 0,020 MN/m3
23. Problem, Solution 174

Design value of the internal friction angle of the earth


jd = j (22)
= 30,000 °

Bearing capacity factors


j
N D  tan2 (45  d )e tanj d
2 (23)
= 18,401
  j  
N B  1,5tan2  45  d e tanj d  1 tanj d
  2   (24)
= 15,070

Shape factors of the base slab


B 
sB  1  0,4 t 
 Lt  (25)
= 0,846
B 
s D  1  0,2 t 
 Lt  (26)
= 1,077

Inclination factors of the load resultant


2
 Hd 
i D   1  
 Vd  At cd cot j d  (27)
= 0,868
4
 Hd 
i B   1  
 Vd  At cd cot j d  (28)
= 0,753

Design value of the bearing capacity


1
qmd  cd N c sc ic  g 1' DN D s D iD  g 2' Bt N B sB iB
2 (29)
2
= 0,354 MN/m
23. Problem, Solution 175

Ratio between bearing capacity and bottom pressure


n = q md /q d (30)
= 2,154 > 2,000 OK

Breadth of the base slab is obtained by an iteration process.


a = 2,300 m
The value is rounded up by precision of 0,1 m.

ALL LOAD CASES

Breadths of the base slab (a i ) in all loading cases are presented in Table 1.
There are presented only the quantities, that are function of a and/or partial
factors; all the other quantities are calculated above.

ANSWER: Breadth of the base slab


a = 2,300 m
23. Problem, Solution 176

Table 1.
Quant. Eq. Loading Cases Unit
i a b c d
a 2,209 1,470 2,141 0,853 m
ext(a ) Max Min
a round 2,300 1,500 2,200 0,900 m
gg 2 1,00 1,00 1,00 1,00
g qk 1 2 1,00 1,00 0,00 0,00
g qk 2 2 1,00 0,00 1,00 0,00
Vg 3 0,987 0,889 0,978 0,808 MN
Vd 10 1,887 1,789 0,978 0,808 MN
Hd 11 0,130 0,000 0,130 0,000 MN
Md 12 0,721 0,240 0,577 0,096 MNm
e0 14 0,382 0,134 0,590 0,119 m
b 15 1,445 1,202 0,961 0,616 m
Bt 16 1,445 1,202 0,961 0,616 m
Lt 17 4,000 4,000 4,000 4,000 m
At 18 5,779 4,807 3,843 2,463 m2
qd 19 0,327 0,372 0,254 0,328 MN/m2
sB 26 0,856 0,880 0,904 0,938
sD 27 1,072 1,060 1,048 1,031
iD 28 0,867 1,000 0,752 1,000
iB 29 0,752 1,000 0,565 1,000
q md 30 0,653 0,745 0,509 0,656 MN/m2
n 31 2,000 2,000 2,000 2,000
20111205 177
24. PROBLEM

A base slab have a breadth a (Figure 1). Determine the smallest possible
dimension a so that the bearing resistance is adequate! (Compare to the
prescriptive method of Eurocode , EN 1997-1:2004 , 6.4(5)P.) The allowed bearing
resistance is
s sall = 0,400 MN/m2

Characteristic values of loading due to the overhead structure


Vo = 1,500 MN
Ho = 0,130 MN
Mo = 0,240 MNm
Proportion of the permanent load from the total load is
kV = 0,400
kH = 0,000
kM = 0,400
and the rest is variable one. The proportion of the variable load of vertical load V o
and moment M o are caused by effective load and horizontal load H o by wind.
V0
M0
Unit weight of reinforced concrete H0
gc = 0,025 MN/m3
Unit weight of soil
gm = 0,020 MN/m3 h
t
Angle of internal friction of the earth
j = 30,000 °

gm gc j
Dimensions c
c = 1,500 m d
h = 2,200 m a/2 a/2
d = 0,600 m
t = 0,400 m Figure 1.

Length of the base slab (perpendicular to the figure plane)


L = 4,000 m
In this direction the loading is central.
24. Problem, Solution 178

COMBINATIONS OF ACTIONS

Load combination
m n
qd   gi qk1  qk 2   qki
i 1 i 3 (1)
can be written
1 1 (a)

1 0 (b)
qd  g  qk1  qk 2
0 1 (c) (2a…d)
0 0 (d)

First the load case "a" is studied and after that the others.

LOAD CASE a

Lets select
a = 2,400 m
The value given here is the final result after the iteration.

Characteristic load values at the surface of the base slab are solved. The sub
indexes are referred to that ones used in Equation 1.

Self weight of the overhead structure, base structure and soil (as function of a )
V g = k V V 0+g c[(h +c-d )t +ad ]L +g m (a-t )(c-d )L (3)
= 1,012 MN

Variable vertical imposed load


V q 1 = (1-k V )V 0 (4)
= 0,900 MN
24. Problem, Solution 179

Moment due to permanent loads


M g = k M M 0+k H H 0(c +h ) (5)
= 0,096 MNm

Moment due to imposed load


M q 1 = (1-k M )M 0 (6)
= 0,144 MNm

Moment due to wind load


M q 2 = (1-k H )H 0(c +h ) (7)
= 0,481 MNm

Design values of loads: vertical load and moment


V d = V g +V q 1 (8)
= 1,912 MN
Md = M g +M q 1+M q 2 (9)
= 0,721 MNm

Stress (Figure 2)
Vd Md
s ( y)   y
A Ix (10)

a/2 a/2
Md Vd
y
x
z smin
smax = ssall

Figure 2.

Area
A = aL (11)
= 9,600 m2
24. Problem, Solution 180

Moment of inertia with respect to x- axis


La 3
Ix 
12 (12)
4
= 4,608 m

Maximum stress (Figure 2)


s max = s (y = a /2) (13)
= 0,387 MN/m2 < s sall

Breadth of the base slab is obtained by an iteration process.


a = 2,400 m
The value is rounded up by precision of 0,1 m.

Minimum stress (Figure 2)


s min = s (y = -a /2) (14)
= 0,011 MN/m2 Compression

ALL LOAD CASES

Breadths of the base slab (a i ) in all loading cases are presented in Table 1.
There are presented only the quantities, that are function of a and/or partial
factors; all the other quantities are calculated above.

LOAD CASE c

In load case c, the tension is obtained as a minimum value of stress.


Tension stress is not possible. Next is examined an effective breadth of
cross-section (Figure 3).
24. Problem, Solution 181

Table 1.
Quantity Eq. Load Case Unit
i a b c d
a 2,343 1,673 1,791 0,907 m
ext(a ) Max Min
a round 2,400 1,700 1,800 1,000 m
gg 2 1,00 1,00 1,00 1,00
g qk 1 2 1,00 1,00 0,00 0,00
g qk 2 2 1,00 0,00 1,00 0,00
Vg 3 1,004 0,916 0,932 0,815 MN
Vd 8 1,904 1,816 0,932 0,815 MN
Md 9 0,721 0,240 0,577 0,096 MNm
A 11 9,370 6,692 7,163 3,626 m2
I x 12 4,285 1,561 1,914 0,248 m4
s max 13 0,400 0,400 0,400 0,400 MN/m2
s min 14 0,006 0,143 -0,140 0,049 MN/m2
ext(s min) Max Min

a/2 a/2
Md Vd
y
x
z smin = 0
smax = ssall R
e/3 2e/3

Figure 3.

Lets select
a = 2,100 m
The value given here is the final result after the iteration.
24. Problem, Solution 182

Self weight of the overhead structure, base structure and soil (Equation 3,
function of a )
Vgc = 0,972 MN

Design values of loads: vertical load and moment


Vdc = Vgc (15)
= 0,972 MN
Mdc = M g +M q 2 (16)
= 0,577 MNm

From the equilibrium condition of the vertical forces (Figure 3)


1
Vdc  eLs sall
2 (17)
breadth of the compressed area is obtained
2Vdc
e
Ls sall (18)
= 1,216 m

Equilibrium condition of the moments with respect to origin


a e
M d  Vdc   
 2 3 (19)
= 0,627 MNm

Ratio between the moment due to bottom stress and external moment
n = M d /M3d (20)
= 1,087 ≥ 1

Breadth of the base slab is obtained by an iteration process.


a = ac = 2,100 m

ANSWER: Breadth of the base slab


a = 2,400 m
20111205 183
25. PROBLEM

Determine the pile forces due to point load


P = 6 MN
of the pile group shown in Figure 1!
All piles have the same axial stiffness
Ci = EiAi
Length of the piles
L1 = 14,000 m
L2 = 10,500 m
L3 = 7,000 m
Number of piles in the rows
n1 = 4
n2 = 3
n3 = 3
Distance between piles
d = 1,000 m

d d

1 2 3

Figure 1.
25. Problem, Solution 184

Relative stiffness of the piles


C L L (1) P
ki  i i  1
C1 L1 Li M
k1 = 1,000
k2 = 1,333 d d
k3 = 2,000
Distances of the rows from the centroid line, r2 r3
respectively, (Figure 2) r1
3
 ni ki zi 1 2 3
r1  i 2
3 (2)
 ni ki
i 1
n2k2d  n3k3  2d

n1k1  n2k2  n3k3 (3)
= 1,143 m
r 2 = r 1-d (4) Figure 2.
= 0,143 m
r 3 = r 1- 2d (5)
= -0,857 m
Moment of the loads with respect to the centroid line
M = Pr 2 (6)
= 0,857 MNm
Pile force at row i
ki ki ri
Ni  P M
 ni ki  i ii
n k r 2 (7)
ki P ki ri M
  (8)
n1k1  n2k2  n3k3 n1k1r12  n2k2r22  n3k3r32

ANSWER: Pile forces are


N1 = 0,529 MN
N2 = 0,588 MN
N3 = 0,706 MN
20111205 185
26. PROBLEM

Determine the minimum and maximum pile loads in the pile group shown below
(Figure 1, Table 1 and 2)!

Loading
Fx = 8 MN
Fz = 2 MN
My = 1 MNm

Table 1. Vertical piles (z Vi on the level of pile truncation).


Variable Row Number Horizontal Relative
of piles location stiffness
Symbol i V n Vi z Vi k Vi
Unit - m -
Numerical 1 10 -3,000 0,600
value 2 9 3,000 0,700

Table 2. Diagonal piles (z Di on the level of pile truncation), inclination


k = 1/tana
= 3,500 .
Variable Row Number Horizontal Relative
of piles location stiffness
Symbol i D n Di z Di k Di Fx
Unit - m - Fz My z
Numerical 1 8 -3,000 0,962 y
value 2 7 2,500 0,962

Relative stiffnesses of a pile


a
E i Ai
Li
ki 
E 0 A0 V1 V2
L0
x D1 D2

Figure 1.
26. Problem, Solution 186

Directional angle of diagonal piles


a = arctan(1/k ) (1)
= 15,945 °
Axial stiffnesses of the vertical and diagonal pile groups, respectively
A V = Sn Vi k Vi (2)
= 12,300
AD = Sn Di k Di (3)
= 14,423

z -coordinates of the centroid lines of vertical and horizontal piles on the level of
pile truncation (Figure 2)
zV   Vi Vi Vi
n k z
AV (4)
= 0,073 m
z D   Di Di Di
n k z
AD (5)
= -0,433 m

zV
zDi
zD
Fx
My
Fz
z
y
a
rDi x0
My0
z’
y’
z0 = zV a
0

x
x’
Figure 2.
26. Problem, Solution 187

Rotation center (Figure 2)


 zV  z D
 x0 
 tana (6a, b)
 z0  zV
 x0 = 1,773 m

 z0 = 0,073 m

Angle of principle direction clockwise (is not needed in this problem)


1   2 AD cosa sina 
0  arctan 
2  AV  AD cos 2a  (7)
= -8,623 °
= -0,150 rad

Lever arms of vertical piles


r Vi = z Vi - z V (8)
rV1 = -3,073 m
rV2 = 2,927 m

Lever arms of diagonal piles


r Di = (z Di - z D)cosa (9)
rD1 = -2,468 m
rD2 = 2,820 m

Moment of inertia with respect to the rotation center

 I   n k r 2
V Vi Vi Vi (10a, b)

 I D   nDi k Di rDi
2
2
 IV  110,634 m
 2
I D 
100,393 m
I = I V +I D (11)
2
= 211,027 m
26. Problem, Solution 188

Loading at the rotation center


Fx0 = Fx (12)
= 8,000 MN
Fz0 = Fz (13)
= 2,000 MN
My0 = M y -F x z 0+F z x 0 (14)
= 3,960 MNm

Load components (Figure 3)


F
FxD  z 0
sina (15)
= 7,280 MN Compression
F
FxV  Fx 0  z 0
tan a (16)
= 1,000 MN Compression

Fx0
Fz0

Fz 0
 Fz 0 F xD 
sina
tan a
Fz 0
FxV  Fx 0 
tan a

Figure 3.
26. Problem, Solution 189

1
Axial forces of vertical piles
F M y0 
NVi  kVi  xV  rVi 
 AV I  (17)
NV1 = 0,014 MN Compression
NV2 = 0,095 MN Compression

Axial forces of diagonal piles


F M y0 
N Di  k Di  xD  rDi 
 AD I  (18)
ND1 = 0,441 MN Compression
ND2 = 0,536 MN Compression

Cosines of directional angles with respect to x - and z -axis


p xVi = cos(0) (19)
= 1,000
p zVi = sin(0) (20)
= 0,000
p xDi = cosa (21)
= 0,962
p zDi = sina (22)
= 0,275

Elements of stiffness matrix


k 11 = Sn Vi k Vi p xVi 2+Sn Di k Di p xDi 2 (23)
= 25,634
k 12 = Sn Vi k Vi p xVi p zVi +Sn Di k Di p xDi p zDi (24)
= 3,810
= Sn Vi k Vi p zVi +Sn Di k Di p zDi
2 2
k 22 (25)
= 1,089

1) Compare to
F M
  y
A I
26. Problem, Solution 190

k 33 = I
2
= 211,027 m

Stiffness matrix
 25,634 3,810 0,000 
 
K = 3,810 1,089 0,000 
 0,000 0,000 211,027 
The stiffness matrix is unitless except the element k 33.

Inverse matrix
 0,081 -0,285 0,000 
K =  
-1
-0,285 1,915 0,000 
 0,000 0,000 0,005 

Elements of force vector {f }


Fx0 = 8,000 MN
Fz0 = 2,000 MN
My0 = 3,960 MNm

Displacement vector, when coordinate system is situated in the rotation center


{d } = [K ]-1{f } (27)
u 0 = 0,081 MN

 w 0 = 1,553 MN
j =
 0,019 MNm

Relative displacements in the original coordinate system


u = u 0-z 0sinj (28)
= 0,080 MN
w = w 0+x 0sinj (29)
= 1,586 MN
The real displacements are obtained, when the real stiffnesses are used.
26. Problem, Solution 191

Axial forces of vertical piles can also be calculated by way of displacements


N Vi = k Vi (p xVi u 0+p zVi w 0+r yVi j ) (30)
NV1 = 0,014 MN
NV2 = 0,095 MN
Respectively for diagonal piles
N Di = k Di (p xDi u 0+p zDi w 0+r yDi j ) (31)
ND1 = 0,441 MN
ND2 = 0,536 MN

ANSWER:

Minimum and maximum pile forces


N min = 0,014 MN Compression V1
N max = 0,536 MN Compression D2

Relative displacements
u = 0,080 MN
w = 1,586 MN
j = 0,019 MNm
26. Problem, Solution 192

Alternative Way to Solv the Pile Forces

Moment components
IV
MV  M y0
IV  I D (32)
= 2,076 MNm
MD = M y0 -M V (33)
= 1,884 MNm

Axial forces of vertical piles


F M r 
NVi  kVi  xV  V Vi 
 AV IV  (34)
NV1 = 0,014 MN Compression
NV2 = 0,095 MN Compression

Axial forces of diagonal piles


F M r 
N Di  k Di  xD  D Di 
 AD ID  (35)
ND1 = 0,441 MN Compression
ND2 = 0,536 MN Compression
20111205 193
27. PROBLEM

A pile group includes piles of wood, reinforced concrete and steel. See Figure 1 and
Table 1.

Are the pile forces acceptable? Determine the principal coordinate system and the
corresponding diagonal main elements of the stiffness matrix!

Table 1. Pile information.


Variable Row Number Slope Cross- Modulus Vertical Horizont.
of piles section of location location
area elasticity
Symbol i ji tana i Ai Ei xi zi
Unit - - - m2 MN/m2 m m
Numerical 1 5 0,000 0,031 5 600 0 -2,000
value 2 5 0,000 0,031 5 600 0 0,000
3 3 0,200 0,090 31 600 0 0,000
4 1 0,250 0,018 210 000 0 2,000

Acceptable stresses according to pile class III (Finnish standard) are as follows:

Wood1
s w, allowable = 5 MN/m2

Reinforced concrete2
s c, allowable = 5 MN/m2

Steel3
s s, allowable = 40 MN/m2

1) r = 100 mm, T40, Time Class A, Humidity Class 1.


2) a = 300 mm, K40.
3) r = 75 mm, Fe52C.
27. Problem 194

Loading
Fx = 2,000 MN
Fz = 0,400 MN
My = 1,000 MNm

Depth
h = 8m

Fx
Fz My
z

h
1 2 3 4

x
Figure 1.
27. Problem, Solution 195

Length (Figure 2)
L i = h i /cosa i (1)

sina

z
1

cosa
a

x
Figure 2.

Stiffness
C
ki  i
Li (2)
E i Ai
 ki 
Li (3)

Cosines of directional angles with respect to x - and z -axis


p x,i = cosa i (4)
p z,i = cos(90°- a i ) (5)
= sina i (6)

Lever arm with respect to origin


r y,i = z i p x,i -x i p z,i (7)

Elements of stiffness matrix for pile row i


k 11,i = n i k i p x,i 2 (8)
k 12,i = k 21,i (9)
= n i k i p x,i p z,i (10)
k 13,i = k 31,i (11)
= n i k i p x,i r y,i (12)
27. Problem, Solution 196

2
k 22,i = n i k i p z,i (13)
k 23,i = k 32,i (14)
= n i k i p z,i r y,i (15)
2
k 33,i = n i k i r y,i (16)

Elements of stiffness matrix for the pile group


max
kmn   kmn,i
i 0 (17)

Table 2. Calculation of the stiffness matrix.


Variable Row Number Vertical Horizont. Slope Directio- Cross-
of piles location location nal angle section
area
Symbol i ji xi zi tana i ai Ai
Unit - - m m - ° m2
Numerical 1 5 0,000 -2,000 0,000 0,000 0,031
value 2 5 0,000 0,000 0,000 0,000 0,031
3 3 0,000 0,000 0,200 11,310 0,090
4 1 0,000 2,000 0,250 14,036 0,018

Variable Row Modulus Length Stiffness Cosines of Lever


of directional angles arm
elasticity
Symbol i Ei Li ki p x,i p z,i r y,i
Equation (1) (3) (4) (6) (7)
2
Unit - MN/m m MN/m - - m
Numerical 1 5 600 8,000 21,700 1,000 0,000 -2,000
value 2 5 600 8,000 21,700 1,000 0,000 0,000
3 31 600 8,158 348,596 0,981 0,196 0,000
4 210 000 8,246 458,392 0,970 0,243 1,940

27. Problem, Solution 197

… Variable Row Elements of the stiffness matrix of pile row i


Symbol i k 11,i k 12,i k 13,i k 22,i k 23,i k 33,i
Equation (8) (10) (12) (13) (15) (16)
Unit - MN/m MN/m MN MN/m MN MNm
Numerical 1 108,500 0,000 -217,000 0,000 0,000 434,000
value 2 108,500 0,000 0,000 0,000 0,000 0,000
3 1005,567 201,113 0,000 40,223 0,000 0,000
4 431,428 107,857 862,856 26,964 215,714 1725,712

Variable Elements of the stiffness matrix of the pile group


Symbol k 11 k 12=k 21 k 13=k 31 k 22 k 23=k 32 k 33
Equation (17) (17) (17) (17) (17) (17)
Unit MN/m MN/m MN MN/m MN MNm
Numerical value 1653,995 308,970 645,856 67,187 215,714 2159,712

Elements of the stiffness matrix for the pile group are calculated in Table 2.
 k11 k12 k13 
K   k21 k22 k23  (18)
 k31 k32 k33 
 1653,995 308,970 645,856 MN/m MN/m MN 
  308,970   MN/m MN/m MN 
67,187 215,714  
 645,856 215,714 2159,712 
 MN MN MNm 
Displacement vector is obtained from equilibrium condition
{f } = [K ]{d } (19)
 {d } = [K ] {f }
-1
(20)
Element of the displacement vector (displacements and rotation of the base slab)
u = 0,001 m
w = 0,003 m
j = 0,000 rad
Pile forces are calculated in Table 3.
Ni = kiDi (21)
= k i (p x,i u +p z,i w +r y,i j ) (22)
N max,i = s p, sall A i (23)
27. Problem, Solution 198

Table 3. Pile forces and the acceptable extreme values.


Symbol i N min,i Ni N max,i Acceptable
Equation (21) (23)
Unit - MN MN MN
Numerical value 1 0 0,015 0,155 OK
2 0 0,013 0,155 OK
3 0 0,435 0,450 OK
4 0 0,595 0,720 OK

Rotation center
 k11 k13

 x   k12 k23 k k k k
  11 23 12 13
 0 k11 k12 2
k11k22  k12

 k12 k22
 (24a, b)
 k12 k13
 k k23 k k  k22k13
 z0   22   12 23
 k11 k12 2
k11k22  k12
 k12 k22

x 0 = -10,0382 m
z 0 = -1,4847 m

Angle of principal direction (anticlockwise)


1  2k12 
f0  arctan 
2 k 
 11 22 
k (25)
= 0,186 rad
= 10,639 °

Principal direction coordinate system (Figure 3)


x ki = (z i -z 0)sinf 0+(x i -x 0)cosf 0 (26)
z ki = (z i -z 0)cosf 0-(x i -x 0)sinf 0 (27)
a ki = a i - f 0 (28)
27. Problem, Solution 199

y z0 z z’
z
z’
(z-z0)sinf0
x0 z-z0
(z-z0)cosf0
x-x0
f0
(x-x0)cosf0
(x-x0)sinf0
x ai
x’ f0
a’ = ai-f0
x’
x
Figure 3.

Table 4. Calculation of the stiffness matrix.


Variable Row Number Coordinates Angle
Symbol i k = i j ki = j i x ki z ki a ki
Equation (26) (27) (28)
Unit - - m m °
Numerical 1 5 9,771 -2,360 -10,639
value 2 5 10,140 -0,394 -10,639
3 3 10,140 -0,394 0,671
4 1 10,509 1,572 3,398

Variable Row Stiffness Cosines Arm


Symbol i k = i k ki = k i p xk,i p zk,i r yk,i
Equation (4) (6) (7)
Unit - MN/m - - m
Numerical 1 21,700 0,983 -0,185 -0,515
value 2 21,700 0,983 -0,185 1,485
3 348,596 1,000 0,012 -0,513
4 458,392 0,998 0,059 0,946 …
27. Problem, Solution 200

… Variable Row Elements of the stiffness matrix for pile row i


Symbol i k = i k 11ki k 12ki k 13ki k 22ki k 23ki k 33ki
Unit - MN/m MN/m MN MN/m MN MNm
Numerical 1 104,802 -19,686 -54,952 3,698 10,322 28,813
value 2 104,802 -19,686 158,318 3,698 -29,739 239,162
3 1045,646 12,253 -536,250 0,144 -6,284 275,011
4 456,782 27,119 432,883 1,610 25,700 410,235
Variable Elements of the stiffness matrix for the pile group
Symbol k 11k k 12k =k 21k k 13k =k 31k k 22k k 23k =k 32k k 33k
Unit MN/m MN/m MN MN/m MN MNm
Numerical value 1712,032 0,000 0,000 9,149 0,000 953,222

Elements of the stiffness matrix for the pile group are calculated in Table 4.
 k11k k12k k13k 
K k   k21k k22k k23k  (29)
 k31k k32k k33k 
 1712,032 0,000 0,000 MN/m MN/m MN 
  0,000 9,149 0,000  MN/m MN/m MN 
  
 0,000 0,000 953,222  MN MN MNm

ANSWER:

Pile forces are acceptable.


Rotation center is
x 0 = -10,0382 m
z 0 = -1,4847 m
Angle of principal direction is
f0 = 0,186 rad
Principle stiffnesses (diagonal main elements of the stiffness matrix) are
k 11k = 1 712 MN/m
k 22k = 9 MN/m
k 33k = 953 MNm
20111205 201
28. PROBLEM

Bending moment
My = 1 MNm
is acting on the top of a column founded on piles (Figure 1 and Table 1). Determine
the horizontal force F z 1-1 and moment M y 1-1 at section 1 - 1!

Table 1. Pile group information.


Variable Row Number Slope Stiffness Vertical Horizont.
of piles location location
Symbol i ji tana i ki xi zi
Unit - - - MN/m m m
Numerical 1 1 -0,250 20,000 0,000 -0,800
value 2 1 0,250 20,000 0,000 -0,800
3 1 -0,250 20,000 0,000 0,800
4 1 0,250 20,000 0,000 0,800

The width of the square column is


a = 0,600 m My

2
E = 30 000 MN/m
L
Dimensions
L = 6m 1 1
d = 1m d y z
h = 10 m

1 3 2 4

Figure 1.
28. Problem, Solution 202

Elements of the stiffness matrix are calculated in Table 2.

Table 2.
Variable Row Number Vertical Horizont. Slope Directio- Stiffness
of piles location location nal angle
Symbol i ji xi zi tana i ai ki
Unit - - m m - ° MN/m
Numerical 1 1 0,000 -0,800 -0,250 -14,036 20,000
value 2 1 0,000 -0,800 0,250 14,036 20,000
3 1 0,000 0,800 -0,250 -14,036 20,000
4 1 0,000 0,800 0,250 14,036 20,000

Variable Row Cosines of Lever


directional angles arm
Symbol i p x,i p z,i r y,i
Unit - - - m
Numerical 1 0,970 -0,243 -0,776
value 2 0,970 0,243 -0,776
3 0,970 -0,243 0,776
4 0,970 0,243 0,776

Variable Row Diagonal main elements of piles Other elements


Symbol i k 11,i k 22,i k 33,i k 12,i =k 21,i k 13,i =k 31,i k 23,i =k 32,i
Unit - MN/m MN/m MNm MN/m MN MN
Numerical 1 18,824 1,176 12,047 -4,706 -15,059 3,765
value 2 18,824 1,176 12,047 4,706 -15,059 -3,765
3 18,824 1,176 12,047 -4,706 15,059 -3,765
4 18,824 1,176 12,047 4,706 15,059 3,765
Variable Main elements of the pile group Other elements
Symbol k 11 k 22 k 33 k 12,i =k 21 k 13,i =k 31 k 23,i =k 32
Unit MN/m MN/m MNm MN/m MN MN
Numerical value 75,294 4,706 48,188 0,000 0,000 0,000
28. Problem, Solution 203

Angle of principal direction


1  2k12 
0 = arctan 
2  k22 - k11  (1)
= 0,000 rad

Rotation center
ì k 11 k 13
ï
ï x = - k 12 k 23 k k - k 12 k 13
= - 11 23
ï 0 k 11 k 12 k 11 k 22 - k 12
2
ï
ï k 12 k 22
í (2a, b)
ï k 12 k 13
ï k k 23 k k - k 22 k 13
ï z 0 = - 22 = - 12 23
ï k 11 k 12 k 11 k 22 - k 12
2
ï k 12 k 22
î
ì x0 = 0,000 m
Þí
î z0 = 0,000 m

Stiffness matrix (Table 2)


é k 11 k 12 k 13 ù
[ K ] = êê k 21 k 22 k 23 úú (3)
êë k 31 k 32 k 33 úû
é 75,294 0,000 0,000 ù éMN/m MN/m MN ù
êMN/m MN úú
= êê 0,000 4,706 0,000
ú
ú ê MN/m
êë 0,000 0,000 48,188 úû êë MN MN MNm úû

Inverse matrix
é 0,013 0,000 0,000 ù
[K ] = êê ú
-1
0,000 0,213 0,000 ú
êë 0,000 0,000 0,021 úû

Equilibrium condition
{f } = [K ]{d } (4)
28. Problem, Solution 204

Displacement vector
{d } = [K ] {f }
-1
(5)

At the top of the column a force is applied


Fz = -1 MN

So the real support force is


H = cF z (6)

Moment at origin due to horizontal force


M y 0,Fz = F z (L +d ) (7)

Elements of the force vector at x = 0 due to moment (M y = 1) and horizontal force


(F z = -1) are shown in Table 3.

Table 3.
i = My i = Fz
F x 0,i 0,000 0,000 MN
F z 0,i 0,000 -1,000 MN
M y 0,i 1,000 -7,000 MNm

Elements of the displacement vector at x = 0 due to moment (M y = 1) and


horizontal force (F z = -1) are shown in Table 4.

Table 4.
i = My i = Fz
u 0,i 0,000 0,000 m vertical displacement
w 0,i 0,000 -0,213 m horizontal displacement
j 0,i 0,021 -0,145 rad rotation
28. Problem, Solution 205

Bending stiffness of the column


a4
D=E
12 (8)
2
= 324 MN/m

Horizontal displacement at the top of the column due to moment M y (Figure 2)


M y L2
w My = w0, My  ( L  d )j 0, My 
2D (9)
= 0,201 m

My
MyL2/2D

(L+d)j0,My

MyL/D

L+d

j0,My

Figure 2.

Horizontal displacement at the top of the column due to horizontal force F z


(Figure 3)
Fz L3
wFz = w0, Fz  ( L  d )j0, Fz 
3D (10)
= -1,452 m
28. Problem, Solution 206

Compatibility condition at the top of the column


w = w My  cw Fz = 0 (11)
- w My
Þc = (12)
w Fz

= 0,138

FzL3/3D (L+d)j0,Fz Fz

w0,Fz

L+d

j0,Fz

Figure 3.

Horizontal force and moment at section 1 - 1


F z 1-1 = H = cF z (13)
= -0,138 MN
M y 1-1 = M y +HL (14)
= 0,170 MNm

ANSWER:

Horizontal force and moment at section 1 - 1 are, respectively,


F z 1-1 = -0,138 MN
M y 1-1 = 0,170 MNm
28. Problem, Solution 207

DETERMINING OF THE DEFLECTION AND ROTATION


OF CANTILEVER BEAM BY USING MOHR'S METHOD

In Figure A it is presented, how the deflection and rotation of a cantilever beam


having constant stiffness due to point load F and point moment M 0 acting at the
end of the cantilever are determined.

L
F
D = EI
v
j

-FL/D
x -
M/D

-FL/D R = -FL2/2D
MM/D = v = FL3/3D
2L/3
QM/D =  = -FL2/2D

L
j
v
M0
D = EI

x
+
M0/D
M/D

M0/D
R = M0L/D
MM/D = v = -M0L2/2D
L/2
QM/D =  = M0L/D

Figure A.
28. Problem, Solution 208

Mohr's method

1) The bending moments M (x ) are divided by the bending stiffnesses D (x )


(M/D -distribution curve).

2) The boundary conditions of the structure are modified so that the deflections
of the original structure correspond to the bending moments of the modified
one and, respectively, rotations correspond to shear forces. The modified
structure is subjected to M/D -distribution curve.

3) Bending moment caused by M/D -load M M/D (x ) corresponds to deflection


v (x ) and, respectively, shear force Q M/D (x ) corresponds to rotation j (x ).

Some examples in Finnish: Arvo Ylinen: Kimmo ja lujuusoppi I . WSOY.


2 edition. Porvoo 1965. P. 268 - 273.
20131108 209
29. PROBLEM

Determine the area, where vertical force


V = 1 MN
can be situated so that all piles are compressed (core figure)! See Figure 1.

Use system of equilibrium conditions of space pile group!

All piles are in vertical direction and have similar properties (cross-section, same
material).

Dimensions are
d = 1,500 m
L = 5,000 m

Table 1. Pile group information.


Symbol i xi yi zi Li
Unit - m m m m
Numerical 1 0,000 0,750 -1,500 10,000
value 2 0,000 0,750 0,000 15,000
3 0,000 0,750 1,500 20,000
4 0,000 -0,750 -1,500 10,000
5 0,000 -0,750 0,000 15,000
29. Problem 210

1 2 3
d/2 y
z
d/2
4 5
d d
V

x
L

Figure 1.
29. Problem, Solution 211

Because of the fact that now


p y,i = cosa i,y = 0 (1)
p z,i = cosa i,z = 0 (2)
r x,i = y i p z,i -z i p y,i = 0 (3)
we have a pile group in plane. Thus,
1 2 3
px p y pz rx ry rz
é å kp2 0 0 0 å kpx ry å kpx rz ù p x 1
x
ê ú
ê 0 0 0 0 0 0 ú py
ê 0 0 0 0 0 0 úp
[K ] = ê ú z (4)
ê 0 0 0 0 0 0 ú rx
ê 2 ú
êå kry p x 0 0 0 å kry å kry rz ú r y 2
ê å kr p 0 0 0 å krz ry å krz2 úû rz 3
ë z x

Relative stiffnesses of the piles


Ei Ai
Li
ki =
E3 A3 (5)
L3
L
Þ ki = 3
Li
k1 = 2,000
k2 = 1,333
k3 = 1,000

Centroid

y0 = å i i
k y
å ki (6)
= 0,098 m

z0 = å i i
kz
å ki (7)
= -0,587 m
29. Problem, Solution 212

Correspondingly,
p x,i = cosa i,x (8)
r y,i = z i p x,i -x i p z,i (9)
r z,i = x i p y,i -y i p x,i (10)

Elements of stiffness matrix are presented in Table 2.

Table 2.
Variable Pile Location Angle Stiffness
Symbol i xi yi zi ai ki
Unit m m m ° -
Numerical 1 0,000 0,652 -0,913 0,000 2,000
value 2 0,000 0,652 0,587 0,000 1,333
3 0,000 0,652 2,087 0,000 1,000
4 0,000 -0,848 -0,913 0,000 2,000
5 0,000 -0,848 0,587 0,000 1,333

Variable Pile Cosines of directional angles Lever arms


Symbol i p x,i p y,i p z,i r x,i r y,i r z,i
Unit - - - m m m
Numerical 1 1,000 0,000 0,000 0,000 -0,913 -0,652
value 2 1,000 0,000 0,000 0,000 0,587 -0,652
3 1,000 0,000 0,000 0,000 2,087 -0,652
4 1,000 0,000 0,000 0,000 -0,913 0,848
5 1,000 0,000 0,000 0,000 0,587 0,848

29. Problem, Solution 213


Variable Pile Elements of the stiffness matrix of pile row i
Symbol i k 11,i k 12,i k 13,i k 22,i k 23,i k 33,i
Numerical 1 2,000 -1,826 -1,304 1,667 1,191 0,851
value 2 1,333 0,783 -0,870 0,459 -0,510 0,567
3 1,000 2,087 -0,652 4,355 -1,361 0,425
4 2,000 -1,826 1,696 1,667 -1,548 1,438
5 1,333 0,783 1,130 0,459 0,664 0,958
Unit MN/m MN MN MNm MNm MNm
Numerical value 7,667 0,000 0,000 8,609 -1,565 4,239
Symbol k 11 k 12=k 21 k 13=k 31 k 22 k 23=k 32 k 33
Variable Elements of the stiffness matrix of the pile group

Stiffness matrix
é 7,667 0,000 0,000 ù éMN / m MN / m MN ù
[K ] = êê 0,000 8,609 -1,565
ú
ú
êMN / m MN / m MN ú
ê ú
êë 0,000 -1,565 4,239 úû êë MN MN MNmúû
Inverse matrix
é 0,130 0,000 0,000 ù ém / N m / N 1 / N ù
[K ] = êê ú êm / N m / N 1 / N ú
-1
0,000 0,125 0,046 ú ê ú
êë 0,000 0,046 0,253 úû êë 1 / N 1 / N 1 / Nm úû

From system of equilibrium condition


{F } = [K ]{d } (11)
the deflection vector
-1
{d } = [K ] {F } (12)
ìuü ì 1 ü (x -axial vertical force [MN])
ï ï -1 ï ï
Þ íj ý = [K ] í z ý (Moment with respect to y -axis [MNm]) (13)
ïq ï ï- y ï
î þ î þ (Moment with respect to z -axis [MNm])
ì u = 0,000 y + 0,000 z + 0,130
ï
Þí j = -0,046 y + 0,125 z + 0,000
ï q = -0,253 y + 0,046 z + 0,000
î
is obtained.
29. Problem, Solution 214

Pile forces
Ni = kiD i (14)
= k i (p x,i u +r y,i j +r z,i q ) (15)
N1 = 0,414 y + -0,287 z + 0,261 =0
N2 = 0,184 y + 0,057 z + 0,174 =0
N3 = 0,069 y + 0,230 z + 0,130 =0
N4 = -0,345 y + -0,149 z + 0,261 =0
N5 = -0,322 y + 0,149 z + 0,174 =0

ANSWER:

Boundary lines of the core figure are shown in Figure 2.

1,5 y [m] 1
1,0 2
0,5
3
0,0 z [m]
-3 -2 -1 0 1 2 3 4
-0,5
5
-1,0
Slab
-1,5

Figure 2.
20111205 215
30. PROBLEM

Determine the pile forces shown in Figure 1.

n:1
1 4
a
Mx
y
x
2 5 y
x Fy Fx
a
My
Fz
3 6
n:1

a a
z z

Mz
y
z
n:1 n:1

x
Figure 1.

Elements of the force vector


Fx = 0,000 MN
Fy = 0,000 MN
Fz = 0,000 MN
Mx = 1,000 MNm
My = 0,000 MNm
Mz = 0,000 MNm
Slope of diagonal piles
n = 4
All piles have the same relative stiffness EA /L .
30. Problem, Solution 216

System of equilibrium condition


{F } = [K ]{d } (1)
where force vector
{F } = {F x F y F z M x M y M z }T (2)
and displacement vector
{d } = {u v w w j q }
T
(3)

xi
yi

z
i
zi i

i

x
Figure 2.
30. Problem, Solution 217

and stiffness matrix of the system is sum of the stiffness matrixes of the piles
 p x2 , i p x , i p y, i p x , i pz , i p x , i rx , i p x , i ry , i p x , i rz , i 
 
 p y, i p x , i p 2y, i p y, i pz , i p y, i rx , i p y , i ry , i p y, i rz , i 
 
n pz2, i
K    ki  z,i x,i
p p pz , i p y, i pz , i rx , i p z , i ry , i pz , i rz , i 
(4)
i 1  rx , i p x , i rx , i p y, i rx , i pz , i rx2, i rx , i ry, i rx , i rz , i 

 ry , i p x , i ry , i p y , i ry , i p z , i ry, i rx , i ry2, i ry, i rz , i 
 
 rz , i p x , i rz , i p y, i rz , i pz , i rz , i rx , i rz , i ry, i rz2,i 

where
p x,i = cos i (5)
p y,i = cos i (6)
p z,i = cos i (7)
r x,i = y i p z,i -z i p y,i (8)
r y,i = z i p x,i -x i p z,i (9)
r z,i = x i p y,i -y i p x,i (10)
Angles  i ,  i and  i are shown in Figure 2.

The elements of the stiffness matrix are presented in Table 1.

Table 1.
Variable Pile Number Vertical Horizontal Stiffness
of piles location location
Symbol i ji xi yi zi ki
Unit - - m m m MN/m
Numerical 1 1 0,000 -2,000 -2,000 1,000
value 2 1 0,000 -2,000 0,000 1,000
3 1 0,000 -2,000 2,000 1,000
4 1 0,000 2,000 -2,000 1,000
5 1 0,000 2,000 0,000 1,000
6 1 0,000 2,000 2,000 1,000

30. Problem, Solution 218


Variable Pile Slope and directional angles
Symbol i tan i i tan i i tan i i
Unit - - ° - ° °
Numerical 1 -0,250 -14,036 -4,000 104,036  90,000
value 2 0,250 14,036 4,000 75,964  90,000
3 0,250 14,036  90,000 4,000 75,964
4 -0,250 -14,036  90,000 -4,000 104,036
5 0,250 14,036 4,000 75,964  90,000
6 -0,250 -14,036 -4,000 104,036  90,000

Variable Pile Cosines of directional angles Lever arms


Symbol i p x,i p y,i p z,i r x,i r y,i r z,i
Unit - - - - m m m
Numerical 1 0,970 -0,243 0,000 -0,485 -1,940 1,940
value 2 0,970 0,243 0,000 0,000 0,000 1,940
3 0,970 0,000 0,243 -0,485 1,940 1,940
4 0,970 0,000 -0,243 -0,485 -1,940 -1,940
5 0,970 0,243 0,000 0,000 0,000 -1,940
6 0,970 -0,243 0,000 0,485 1,940 -1,940

Variable i K 11,i K 12,i K 13,i K 14,i K 15,i K 16,i


Numerical 1 0,941 -0,235 0,000 -0,471 -1,882 1,882
value 2 0,941 0,235 0,000 0,000 0,000 1,882
3 0,941 0,000 0,235 -0,471 1,882 1,882
4 0,941 0,000 -0,235 -0,471 -1,882 -1,882
5 0,941 0,235 0,000 0,000 0,000 -1,882
6 0,941 -0,235 0,000 0,471 1,882 -1,882
Unit MN/m MN/m MN/m MN MN MN
Numerical value 5,647 0,000 0,000 -0,941 0,000 0,000
Symbol K 11 K 12=K 21 K 13=K 31 K 14=K 41 K 15=K 51 K 16=K 61

30. Problem, Solution 219


Symbol i K 22,i K 23,i K 24,i K 25,i K 26,i
Numerical 1 0,059 0,000 0,118 0,471 -0,471
value 2 0,059 0,000 0,000 0,000 0,471
3 0,000 0,000 0,000 0,000 0,000
4 0,000 0,000 0,000 0,000 0,000
5 0,059 0,000 0,000 0,000 -0,471
6 0,059 0,000 -0,118 -0,471 0,471
Unit MN/m MN/m MN MN MN
Numerical value 0,235 0,000 0,000 0,000 0,000
Symbol K 22 K 23=K 32 K 24=K 42 K 25=K 52 K 26=K 62

Symbol i K 33,i K 34,i K 35,i K 36,i


Numerical 1 0,000 0,000 0,000 0,000
value 2 0,000 0,000 0,000 0,000
3 0,059 -0,118 0,471 0,471
4 0,059 0,118 0,471 0,471
5 0,000 0,000 0,000 0,000
6 0,000 0,000 0,000 0,000
Unit MN/m MN MN MN
Numerical value 0,118 0,000 0,941 0,941
Symbol K 33 K 34=K 43 K 35=K 53 K 36=K 63

Symbol i K 44,i K 45,i K 46,i K 55,i K 56,i K 66,i


Numerical 1 0,235 0,941 -0,941 3,765 -3,765 3,765
value 2 0,000 0,000 0,000 0,000 0,000 3,765
3 0,235 -0,941 -0,941 3,765 3,765 3,765
4 0,235 0,941 0,941 3,765 3,765 3,765
5 0,000 0,000 0,000 0,000 0,000 3,765
6 0,235 0,941 -0,941 3,765 -3,765 3,765
Unit MNm MNm MNm MNm MNm MNm
Numerical value 0,941 1,882 -1,882 15,059 0,000 22,588
Symbol K 44 K 45=K 54 K 46=K 64 K 55 K 56=K 65 K 66
30. Problem, Solution 220

Stiffness matrix
5,647 0,000 0,000 -0,941 0,000 0,000
0,000 0,235 0,000 0,000 0,000 0,000
K = 0,000 0,000 0,118 0,000 0,941 0,941
-0,941 0,000 0,000 0,941 1,882 -1,882
0,000 0,000 0,941 1,882 15,059 0,000
0,000 0,000 0,941 -1,882 0,000 22,588

Elements of the displacement vector


u = 0,531 m
v = 0,000 m
w = 6,375 m
w = 3,188 rad
j = -0,797 rad
q = 0,000 rad

Pile forces
Ni = kiDi (11)
= k i (p x,i u +p y,i v +p z,i w +r x,i w +r y,i j +r z,i q ) (12)

ANSWER:

Pile forces are


N1 = 0,515 MN
N2 = 0,515 MN
N3 = -1,031 MN
N4 = -1,031 MN
N5 = 0,515 MN
N6 = 0,515 MN
20120823 221
31. PROBLEM

Determine the shear force and bending moment distribution, respectively, and the
deflection curve of the massive pile (large diameter), shown in Figure 1, by using
three beam elements!
x

Diameter of the pile M H


d = 0,700 m
Mean value of cylinder compressive strength of concrete cb a3
f cm = 38 MN/m2
Element lengths
a1 = 3,500 m a2
d
a2 = 3,500 m ca
a3 = 3,500 m
Loads a1
A A
H = 1,500 MN
M = 0,200 MNm y
Foundation coefficient A-A
ca = 40 MN/m3
cb = 5 MN/m3 Figure 1.

Equilibrium condition of a beam element (positive directions are shown in


Figure 2)
é 12 D 6D 12 D 6 D ù
ê 3 2
- 3 ú
ê L L L L2 ú
ê 6D 4D 6D 2 D ú ì v1 ü ì Q1 ü
- ï ï ï ï
ê L2 L L2 L ú ïj1 ï ï M1 ï
=
ê 12 D 6 D 12 D 6 D ú í v2 ý í Q2 ý
ê- 3 - 2 3
- 2 úï ï ï ï
ê L L L L ú ïîj 2 ïþ ïî M 2 ïþ
ê 6 D 2 D 6 D 4 D ú
-
êë L2 L L2 L úû

M1 Q1 Q2 M2
j1 v1 D = EI v2 j2

Figure 2.
31. Problem, Solution 222

Spring coefficient at the top of element i (Figure 3)


a c (
ki = d × i i -1,i
) (
+ 3ci ,i -1 + ai + 1 3ci ,i +1 + ci +1,i )
8 (1)
k 01 = ¥
» 1,0E+100 MN/m
k 02 = ¥
» 1,0E+100 MN/m
k1 = 98,000 MN/m
k2 = 55,125 MN/m
k3 = 6,125 MN/m

ci+1,i i+1 ki+1

ai+1 A A
ci,i+1 ki A-A
ci,i-1 i d
ai
ci-1,i ki-1
i-1
Figure 3.

Constant
ko = 22 000 MN/m2
f cmo = 10 MN/m2
Modulus of elasticity of concrete
0, 3
æ f ö (2)
E = ko çç cm ÷÷
è fcmo ø
=32 837 MN/m2
Moment of inertia for a circle cross-section
4
I = p (d /2) /4 (3)
4
= 0,011786 m
Bending stiffness of the pile
D = EI (4)
2
= 387 MNm
31. Problem, Solution 223

Stiffness matrix of element i


é 108,317 189,555 -108,317 189,555ù
ê ú
189,555 442,295 -189,555 221,147ú
Ki = ê
e
ê -108,317 -189,555 108,317 -189,555ú
ê ú
ë 189,555 221,147 -189,555 442,295û
Corresponding units
éMN/m MN MN/m MN ù
ê MN MNm úú
[ ]
K ie = ê
êMN/m
MNm
MN MN/m
MN
MN ú
ê ú
ë MN MNm MN MNm û

kn
2n+1
2n+2

n an

k3
7
8
3
3 a3
4
k2
5
e
6
1
2 a2
2
k1
3
4

1 a1

k02 k01
1
2

Figure 4.
31. Problem, Solution 224

From the equilibrium condition of the system


{F } = [K ]{d } (5)
the displacement vector
-1
{d } = [K ] {F } (6)
is obtained.

System stiffness matrix (Figure 4)


é k11 k12 k13 k14 k15 k16 k17 k18 ù
êk k 28 úú
ê 21 k 22 k 23 k 24 k 25 k 26 k 27
ê k 31 k 32 k 33 k 34 k 35 k 36 k 37 k 38 ú
ê ú
k k k 43 k44 k 45 k46 k 47 k 48 ú
K= ê 41 42 (7)
ê k51 k52 k 53 k54 k 55 k56 k 57 k 58 ú
ê ú
ê k61 k62 k 63 k64 k 65 k66 k 67 k 68 ú
êk k k 73 k74 k 75 k76 k 77 k 78 ú
ê 71 72 ú
êë k81 k82 k 83 k84 k 85 k86 k 87 k 88 úû
ék1 + k 1
k12 1
k13 1
k14 0 0 0 0 ù
ê 11 01 1 1 1 ú
ê k 21= k12 k 22 + k02 k 23 k 24 0 0 0 0 ú
êk = k k 32= k 23 1
k 33 2
+ k11 + k1 1
k 34 2
+ k12 2
k13 2
k14 0 0 ú
ê 31 13 ú
1 2 2 2
ê k41= k14 k 42= k 24 k 43= k 34 k 44 + k 22 k 23 k 24 0 0 ú
= ê 2 3 2 3 3 3 ú
ê k51= k15 k 52= k 25 k 53= k 35 k54= k 45 k 33 + k11 + k2 k 34 + k12 k13 k14 ú
ê k61= k16 k 62= k 26 k 63= k 36 k64= k 46 k 65= k56 2
k 44 3
+ k 22 3
k 23 3 ú
k 24
ê 3 3 ú
ê k71= k18 k72= k 27 k73= k 37 k 74= k 47 k75= k57 k 76= k 67 k 33 + k3 k 34 ú
êk = k k 82= k 28 k 83= k 38 k84= k 48 k 85= k58 k 86= k 68 k 87= k78 3 ú
k 44
ë 81 18 û
(8)
Numerical values
1,0E+100 189,555 -108,317 189,555 0,000 0,000 0,000 0,000
189,555 1,0E+100 -189,555 221,147 0,000 0,000 0,000 0,000
-108,317 -189,555 314,634 0,000 -108,317 189,555 0,000 0,000
189,555 221,147 0,000 884,589 -189,555 221,147 0,000 0,000
0,000 0,000 -108,317 -189,555 271,759 0,000 -108,317 189,555
0,000 0,000 189,555 221,147 0,000 884,589 -189,555 221,147
0,000 0,000 0,000 0,000 -108,317 -189,555 114,442 -189,555
0,000 0,000 0,000 0,000 189,555 221,147 -189,555 442,295
31. Problem, Solution 225

Corresponding units
MN/m MN MN/m MN MN/m MN MN/m MN
MN MNm MN MNm MN MNm MN MNm
MN/m MN MN/m MN MN/m MN MN/m MN
MN MNm MN MNm MN MNm MN MNm
MN/m MN MN/m MN MN/m MN MN/m MN
MN MNm MN MNm MN MNm MN MNm
MN/m MN MN/m MN MN/m MN MN/m MN
MN MNm MN MNm MN MNm MN MNm

Inverse matrix [K ]-1


0,000 0,000 0,000 0,000 0,000 0,000 0,000 0,000
0,000 0,000 0,000 0,000 0,000 0,000 0,000 0,000
0,000 0,000 0,005 0,001 0,003 -0,001 -0,001 -0,001
0,000 0,000 0,001 0,002 0,002 0,000 0,001 0,000
0,000 0,000 0,003 0,002 0,013 0,002 0,017 0,001
0,000 0,000 -0,001 0,000 0,002 0,003 0,011 0,002
0,000 0,000 -0,001 0,001 0,017 0,011 0,076 0,020
0,000 0,000 -0,001 0,000 0,001 0,002 0,020 0,009

Corresponding units
m/MN 1/MN m/MN 1/MN m/MN 1/MN m/MN 1/MN
1/MN 1/MNm 1/MN 1/MNm 1/MN 1/MNm 1/MN 1/MNm
m/MN 1/MN m/MN 1/MN m/MN 1/MN m/MN 1/MN
1/MN 1/MNm 1/MN 1/MNm 1/MN 1/MNm 1/MN 1/MNm
m/MN 1/MN m/MN 1/MN m/MN 1/MN m/MN 1/MN
1/MN 1/MNm 1/MN 1/MNm 1/MN 1/MNm 1/MN 1/MNm
m/MN 1/MN m/MN 1/MN m/MN 1/MN m/MN 1/MN
1/MN 1/MNm 1/MN 1/MNm 1/MN 1/MNm 1/MN 1/MNm
31. Problem, Solution 226

Loads at the node points


F0 = 0,000 MN
M0 = 0,000 MNm
F1 = 0,000 MN
M1 = 0,000 MNm
F2 = 0,000 MN
M2 = 0,000 MNm
F3 = 1,500 MN
M3 = 0,200 MNm
Node displacements and rotations
w0 = 0,000 m
j0 = 0,000 rad
w1 = -0,002 m
j1 = 0,001 rad
w2 = 0,025 m
j2 = 0,018 rad
w3 = 0,119 m
j3 = 0,032 rad
Force vector of element i
Fie = Kied ie (9)
1. element
ì Q1 ü ì 0,000 m ü ì 0,399 MN ü
ïM ï ï 0,000 ï ï ï
ï 1ï e ï ï ï 0,596 MNm ï
í ý = K1 × í ý = í ý
ï Q2 ï ï -0,002 m ï ï -0,399 MN ï
ïî M 2 ïþ ïî 0,001 ïþ ïî 0,801 MNm ïþ
2. element
ì Q1 ü ì -0,002 m ü ì 0,602 MN ü
ïM ï ï 0,001 ï ï ï
ï 1ï e ï ï ï -0,801 MNm ï
í ý = K ×
2 í ý = í ý
ï Q2 ï ï 0,025 m ï ï -0,602 MN ï
ïî M 2 ïþ ïî 0,018 ïþ ïî 2,907 MNm ïþ
31. Problem, Solution 227

3. element
ì Q1 ü ì 0,025 m ü ì -0,774 MN ü
ïM ï ï 0,018 ï ï -2,907 MNm ï
ï 1ï e ï ï ï ï
í ý = K ×
3 í ý = í ý
ï Q2 ï ï 0,119 m ï ï 0,774 MN ï
ïî M 2 ïþ ïî 0,032 ïþ ïî 0,200 MNm ïþ

ANSWER:

The shear force and bending moment distribution, respectively, and the
deflections at the node points are shown in Table 1. The corresponding curves
are shown in Figures 5, 6 and 7, respectively. Positive directions are shown in
Figure 8.

Table 1.
xi Qi Mi wi
[m] [MN] [MNm] [m]
0,000 -0,399 0,596 0,000
3,500 -0,399 -0,801 -0,002
3,500 -0,602 -0,801 -0,002
7,000 -0,602 -2,907 0,025
7,000 0,774 -2,907 0,025
10,500 0,774 -0,200 0,119
10,500 0 0

-1,0

-0,5
0 2 4 6 8 10 12
0,0

0,5 x [m]

1,0 Q [MN]

Figure 5. Shear force distribution curve.


31. Problem, Solution 228

-4,0
-3,0
-2,0
-1,0
0 2 4 6 8 10 12
0,0 M x [m]
1,0 [MNm]

Figure 6. Bending moment distribution curve.

-0,05
0 2 4 6 8 10 12
0,00
x [m]
0,05

0,10

0,15 w [m]

Figure 7. Deflection curve.

M1 M2
D = EI
j1 j2
Q1 Q2
v1 v2

Figure 8. Positive directions (compare to Figure 2).


20120823 229
32. PROBLEM

Determine the shear force and bending moment distribution, respectively, and the
deflection curve of the massive pile (large diameter) and column, shown in Figure
1, by using in the system stiffness matrix the three uppermost beam elements out
of four! x
M
Diameter of the pile H
d = 0,700 m
a4
Mean value of cylinder compressive strength of concrete
f cm = 38 MN/m2
Element lengths
cb a3
a1 = 3,500 m
a2 = 3,500 m
a3 = 3,500 m
a2
a4 = 3,500 m d
Loads ca
H = 1,500 MN
a1
M = -5,050 MNm A A
Foundation coefficient
y
ca = 40 MN/m3
A-A
cb = 5 MN/m3
Figure 1.
Equilibrium condition of a beam element (positive directions are shown in Fig. 2)
é 12 D 6D 12 D 6D ù
ê 3 - ú
ê L L2 L3 L2 ú
ê 6D 4D 6D 2 D ú ì v1 ü ì Q1 ü
- ï ï ï ï
ê L2 L L2 L ú ïj1 ï ï M1 ï
=
ê 12 D 6D 12 D 6 D ú í v 2 ý í Q2 ý
ê- 3 - 2 - 2 úï ï ï ï
ê L L L3 L ú ïîj 2 ïþ ïî M 2 ïþ
ê 6D 2D
-
6D 4D ú
êë L2 L L2 L úû

M1 Q1 Q2 M2
j1 v1 D = EI v2 j2

Figure 2.
32. Problem, Solution 230

Spring coefficient at the top of element i (Figure 3)


a c (
ki = d × i i -1,i
) (
+ 3ci ,i -1 + ai + 1 3ci , i +1 + ci +1,i )
8 (1)
k1 = 98,000 MN/m
k2 = 55,125 MN/m
k3 = 6,125 MN/m
k4 = 0 MN/m

ci+1,i i+1 ki+1

ai+1 A A
ci,i+1 ki A-A
ci,i-1 i d
ai
ci-1,i ki-1
i-1

Figure 3.

Constant
ko = 22 000 MN/m2
f cmo = 10 MN/m2
Modulus of elasticity of concrete
0, 3
æ fcm ö (2)
E = ko çç ÷
è fcmo ÷ø
32 837 MN/m2
=
Moment of inertia for a circle cross-section
p ( d / 2) 4
I=
4 (3)
4
= 0,011786 m
Bending stiffness of the pile
D = EI (4)
2
= 387,008 MNm
32. Problem, Solution 231

Stiffness matrix of element i


é 108,317 189,555 -108,317 189,555 ù
ê 189,555 442,295 -189,555 221,147 úú
K ie = ê
ê -108,317 -189,555 108,317 -189,555 ú
ê ú
ë 189,555 221,147 -189,555 442,295 û

2n+1 kn

2n+2
3
n an
4
k3
7
e
8
1
3 a3
2
k2
5 6

2 a2

k1
3
4

1 a1
k02
k01
1
2
Figure 4.
32. Problem, Solution 232

From the equilibrium condition of the system


{F } = [K ]{d } (5)
the displacement vector
-1
{d } = [K ] {F } (6)
is obtained.

System stiffness matrix (Figure 4)


é k11 k12 k13 k14 k15 k18 ù
k16 k17
ê ú
ê k 21 k 22 k 23 k 24 k 25 k 26 k 27 k 28 ú
ê k 31 k 32 k 33 k 34 k 35 k 36 k 37 k 38 ú
ê ú
k k42 k 43 k44 k45 k 46 k47 k48 ú
K= ê 41 (7)
ê k 51 k52 k 53 k54 k55 k 56 k57 k58 ú
ê ú
ê k 61 k62 k 63 k64 k65 k 66 k67 k68 ú
êk k72 k 73 k74 k75 k 76 k77 k78 ú
ê 71 ú
êë k 81 k82 k 83 k84 k85 k 86 k87 k88 úû
ék1 + k 2 + k 1
k 34 2
+ k12 2
k13 2
k14 0 0 0 0 ù
ê 33 11 1
1 2 2 2 ú
ê k 21= k12 k44 + k22 k 23 k 24 0 0 0 0 ú
ê k = k k 32= k 23 2
k 33 3
+ k11 + k2 2
k34 3
+ k12 3
k13 3
k14 0 0 ú
ê 31 13 ú
ê k41= k14 2 3 3 3
k42= k 24 k 43= k 34 k44 + k 22 k 23 k 24 0 0 ú
= ê 3 4 3 4 4 4 ú
ê k51= k15 k52= k 25 k 53= k 35 k 54= k45 k 33 + k11 + k3 k 34 + k12 k13 k14 ú
ê k61= k16 3 4 4 4 ú
k62= k 26 k 63= k 36 k 64= k46 k 65= k 56 k44 + k 22 k 23 k 24
ê 4 4 ú
ê k71= k18 k 72= k 27 k 73= k 37 k74= k47 k 75= k 57 k76= k67 k 33 + k4 k 34 ú
ê k = k k82= k 28 k 83= k 38 k 84= k48 k 85= k 58 k86= k68 k87= k 78 4 ú
k 44
ë 81 18 û
(8)

Numerical values
314,634 0,000 -108,317 189,555 0,000 0,000 0,000 0,000
0,000 884,589 -189,555 221,147 0,000 0,000 0,000 0,000
-108,317 -189,555 271,759 0,000 -108,317 189,555 0,000 0,000
189,555 221,147 0,000 884,589 -189,555 221,147 0,000 0,000
0,000 0,000 -108,317 -189,555 222,759 0,000 -108,317 189,555
0,000 0,000 189,555 221,147 0,000 884,589 -189,555 221,147
0,000 0,000 0,000 0,000 -108,317 -189,555 108,317 -189,555
0,000 0,000 0,000 0,000 189,555 221,147 -189,555 442,295
32. Problem, Solution 233

Corresponding units
MN/m MN MN/m MN MN/m MN MN/m MN
MN MNm MN MNm MN MNm MN MNm
MN/m MN MN/m MN MN/m MN MN/m MN
MN MNm MN MNm MN MNm MN MNm
MN/m MN MN/m MN MN/m MN MN/m MN
MN MNm MN MNm MN MNm MN MNm
MN/m MN MN/m MN MN/m MN MN/m MN
MN MNm MN MNm MN MNm MN MNm

Inverse matrix [K ]-1


0,005 0,001 0,003 -0,001 -0,001 -0,001 -0,005 -0,001
0,001 0,002 0,002 0,000 0,001 0,000 -0,001 0,000
0,003 0,002 0,013 0,002 0,017 0,001 0,019 0,001
-0,001 0,000 0,002 0,003 0,011 0,002 0,019 0,002
-0,001 0,001 0,017 0,011 0,076 0,020 0,146 0,020
-0,001 0,000 0,001 0,002 0,020 0,009 0,053 0,009
-0,005 -0,001 0,019 0,019 0,146 0,053 0,368 0,069
-0,001 0,000 0,001 0,002 0,020 0,009 0,069 0,018

Corresponding units
m/MN 1/MN m/MN 1/MN m/MN 1/MN m/MN 1/MN
1/MN 1/MNm 1/MN 1/MNm 1/MN 1/MNm 1/MN 1/MNm
m/MN 1/MN m/MN 1/MN m/MN 1/MN m/MN 1/MN
1/MN 1/MNm 1/MN 1/MNm 1/MN 1/MNm 1/MN 1/MNm
m/MN 1/MN m/MN 1/MN m/MN 1/MN m/MN 1/MN
1/MN 1/MNm 1/MN 1/MNm 1/MN 1/MNm 1/MN 1/MNm
m/MN 1/MN m/MN 1/MN m/MN 1/MN m/MN 1/MN
1/MN 1/MNm 1/MN 1/MNm 1/MN 1/MNm 1/MN 1/MNm
32. Problem, Solution 234

Loads at the node points


F1 = 0,000 MN
M1 = 0,000 MNm
F2 = 0,000 MN
M2 = 0,000 MNm
F3 = 0,000 MN
M3 = 0,000 MNm
F4 = 1,500 MN
M4 = -5,050 MNm
Node displacements and rotations
w1 = -0,002 m
j1 = 0,001 rad
w2 = 0,025 m
j2 = 0,018 rad
w3 = 0,119 m
j3 = 0,032 rad
w4 = 0,205 m
j4 = 0,010 rad
Force vector of element i
Fie = Kied ie (9)
1. element
ì Q1 ü ì 0m ü ì 0,399 MN ü
ïM ï ï 0 ï ï ï
ï 1ï e ï ï ï 0,596 MNm ï
í ý = K1 × í ý = í ý
ï Q2 ï ï -0,002 m ï ï -0,399 MN ï
ïî M 2 ïþ ïî 0,001 ïþ ïî 0,801 MNm ïþ
2. element
ì Q1 ü ì -0,002 m ü ì 0,602 MN ü
ïM ï ï 0,001 ï ï ï
ï 1ï e ï ï ï -0,801 MNm ï
í ý = K ×
2 í ý = í ý
ï Q2 ï ï 0,025 m ï ï -0,602 MN ï
ïî M 2 ïþ ïî 0,018 ïþ ïî 2,907 MNm ïþ
32. Problem, Solution 235

3. element
ì Q1 ü ì 0,025 m ü ì -0,774 MN ü
ïM ï ï 0,018 ï ï -2,907 MNm ï
ï 1ï e ï ï ï ï
í ý = K ×
3 í ý = í ý
ï Q2 ï ï 0,119 m ï ï 0,774 MN ï
ïî M 2 ïþ ïî 0,032 ïþ ïî 0,200 MNm ïþ
4. element
ì Q1 ü ì 0,119 m ü ì -1,500 MN ü
ïM ï ï ï ï -0,200 ï
ï 1ï e ï 0,032 ï ï MNm ï
í ý = K4 × í ý = í ý
ï Q2 ï ï 0,205 m ï ï 1,500 MN ï
ïî M 2 ïþ ïî 0,010 ïþ ïî -5,050 MNm ïþ

ANSWER:

The shear force and bending moment distribution, respectively, and the
deflections at the node points are shown in Table 1. The corresponding curves
are shown in Figures 5, 6 and 7, respectively. Positive directions are shown in
Figure 8.

Table 1.
xi Qi Mi wi
[m] [MN] [MNm] [m]
0,000 -0,399 0,596 0,000
3,500 -0,399 -0,801 -0,002
3,500 -0,602 -0,801 -0,002
7,000 -0,602 -2,907 0,025
7,000 0,774 -2,907 0,025
10,500 0,774 -0,200 0,119
10,500 1,500 -0,200 0,119
14,000 1,500 5,050 0,205
14,000 0 0
32. Problem, Solution 236

-1,0
-0,5 0 2 4 6 8 10 12 14 16
0,0
0,5 x [m]
1,0
1,5
2,0 Q [MN]

Figure 5. Shear force distribution curve.

-5,0
0 5 10 15
0,0
x [m]
5,0
M
10,0 [MNm]

Figure 6. Bending moment distribution curve.

-0,10
0 5 10 15
0,00
x [m]
0,10
0,20
0,30 w [m]

Figure 7. Deflection curve.

M1 M2
D = EI
j1 j2
Q1 Q2
v1 v2

Figure 8. Positive directions (compare to Figure 2).


20111220 237
33. PROBLEM

A horizontal load caused by braking of a vehicle is acting on the top of an


intermediate column-type support. The size of the load according to the Finnish
Standard is
J = 0,500 MN
The intermediate support consists of a massive concrete pile with circular cross-
section (1) and an equal concrete column (2) shown in Figure 1. What is
displacement a at the top of the column caused by the braking load?

Use force method to solve the problem!

Elasticity of the soil is approximated by


a single spring, whose spring coefficient D=
k = 120 MN/m J J
The diameters of the pile and the column,
a
respectively, are
D2 h2
d1 = 1,200 m
d2 = 0,800 m
d2
Height
h1 = 10 m b
h2 = 8m

Modulus of elasticity of concrete


2 A A X = kb
E = 30 000 MN/m

Bending stiffness of the superstructure D1 h1

is assumed to be infinity.
D = d1

The bottom of the pile can be


assumed to act as a hinge.
A-A

Figure 1.
33. Problem, Solution 238

Bending stiffnesses
4
  di 
Di  E  
4 2  (1)
2
D1 = 3 054 MNm
2
D2 = 603 MNm

J 1 J
+ =
D1 D2
X1 X1
h1 h2 h1 h2 h1 h2

x x d=b x
+ =
a
v
d10 v
X1d11 d11 v

Figure 2.

Equilibrium condition (Figure 2)


d 10-X 1d 11 = d (2)
where
X
d 1
k (3)

Bending moment due to point load P = 1 (Figure 3)


 x , x  0...h1
M1 ( x )  
h1 , x  h1 ...h1  h2  (4)

Bending moment due to braking load (Figure 4)


M 0(x ) = Jx (5)
33. Problem, Solution 239

h1 h2

x = [0, h1]

M(x) = Ax
A=1

x = [h1, h1+h2]

h1 1 M(x) = Ax-(x-h1)

A=1

x
M(x) h1

Figure 3.

h1 h2

x = [h1, h1+h2]

M(x) = Jx
J
x
M(x) J(h1+h2)

Figure 4.
33. Problem, Solution 240

Displacements
M1 M 0
d 10   dx
D (6)
h1 h1  h2
Jx 2 h1Jx
  dx   dx
D 1 D2 (7)
0 h1
 h3 h h 2h  h  
 J 1  1 2 1 2 
 3 D1 2 D2  (8)
= 0,983 m
M1 M1
d 11   dx (9)
D
h1 h h
x2 1 2 h2
  dx   1 dx
(10)
0 D1 h1 D2

h13 h12h2 (11)


 
3 D1 D2
= 1,435 m/MN
Reaction force from Equations 2, 3, 8 and 11
d 10
X1 
1 (12)
 d 11
k
= 0,681 MN

Bending moment of the structure (Figure 5)


M  M 0  X 1 M1 (13)
J  X 1  x , x  0... h1 

 Jx  X 1h1 , x  h1 ... h1  h2 
(14)
33. Problem, Solution 241

J
D1 D2

X1
J-X1
h1 h2

x
M(x) Jh2+X1h1
(J+X1)h1

Figure 5.

Bending moment due to load P = 1 acting at the top of the column (Figure 6)
M C (x ) = x (15)

h1 h2

x = [h1, h1+h2]

M(x) = x
1
x
M(x)
h1+h2

Figure 6.
33. Problem, Solution 242

Displacement at the top of the column


a  d1 (16)
MM C
 dx
D (17)

 
h1
J  X1 x 2 dx  h1  h2 Jx 2  X1h1 x dx
D1
 D2 (18)
0 h1

J  X 1 h13 Jh2 3h12  3h1h2  h22  X 1h1h2 2h1  h2 


   (19)
3 D1 3 D2 2 D2
= 0,051 m

ANSWER: Displacement at the top of the column


a = 0,051 m
20111122 243
34. PROBLEM

A horizontal load caused by braking of a vehicle is acting on the top of an


intermediate column-type support. The size of the load according to the Finnish
Standard is
J = 0,500 MN
The intermediate support consists of a massive concrete pile with circular cross-
section (1) and an equal concrete column (2) shown in Figure 1. What is
displacement a at the top of the column caused by the braking load?

Solv the problem by using the expression of the deflections of the cantilever!

Elasticity of the soil is approximated by


a single spring, whose spring coefficient D=
k = 120 MN/m J J
The diameters of the pile and the column, a
respectively, are D2 h2
d1 = 1,200 m
d2 = 0,800 m d2
Height
h1 = 10 m b
h2 = 8m

Modulus of elasticity of concrete A A X = kb


2
E = 30 000 MN/m
D1 h1
Bending stiffness of the superstructure
is assumed to be infinity. d1
D =

The bottom of the pile can be


assumed to act as a hinge. A-A

Figure 1.
34. Problem, Solution 244

Bending stiffnesses
4
 d 
Di  E  i 
4 2  (1)
2
D1 = 3 054 MNm
2
D2 = 603 MNm

Equations given in the Formulary are applied.

X J-X
D2 D1

a
J b
h2 h1

J
Jh2+(J-X)h1
(J-X)h1
J
J-X
J-X
J
(J-X)h1
v2
2
2h1

J-X

v1

Figure 2.
34. Problem, Solution 245

Deflection at the spring (1) (Figure 2)


b  v1  2h1 (2)


J  X h13  Jh22 J  X h1h2 
 
  1
h
(3)
3 D1  2 D2 D2 

where the braking load


X = kb (4)

From Equations 21 and 22


h13 h1h22 h12h2
 
3 D1 2 D2 D2
bJ
3 2
kh kh h (5)
1 1  1 2
3 D1 D2
= 0,006 m

Displacement at the top of the column is sum of spring deflection b and deflection
v 2 at the end of cantilever (2)
a  b  v2 (6)
Jh23 J  X h1h22
b 
3 D2 2 D2 (7)
= 0,051 m

ANSWER: Displacement at the top of the column


a = 0,051 m
34. Problem, Solution 246

Extra Study

Deflection curve, when the origin is at the bottom of the pile (dashed line in Fig. 3)
 3 3
 J  X h1  2  3 x   x   
 6 D1  h1  h1  
  
 2
  Jh2  J  X h1h2  h1  x   a  b, x  [0, h1 ]

v ( x )    2 D2 D2  (8)

 Jh3    x  h1   J  X h1
3
 2 2  3 x h1      h2  x  h1 2 ,
 6 D2  h2  h2   2 D2
 
 x  [h1 , h2 ]
where the braking load from Equation 22 is
X = 0,681 MN

The deflection curve is mirrored so, that the deflection in origin is zero (continuous
line in Fig. 8)
v p (x ) = a-v (x ) (9)

0,06 v [m]
0,05
0,04
0,03
0,02
0,01 x [m]
0,00
18 -0,01 16 14 12 10 8 6 4 2 0

Figure 3.
20111220 247
35. PROBLEM

A horizontal load caused by braking of a vehicle is acting on the top of an


intermediate column-type support. The size of the load according to the Finnish
Standard is
J = 0,500 MN
The intermediate support consists of a massive concrete pile with circular cross-
section (1) and an equal concrete column (2) shown in Figure 1. What is
displacement a at the top of the column caused by the braking load?

Solv the problem by using Mohr's method!

Elasticity of the soil is approximated by


a single spring, whose spring coefficient D=
k = 120 MN/m J J
The diameters of the pile and the column, a
respectively, are D2 h2
d1 = 1,200 m
d2 = 0,800 m d2
Height
h1 = 10 m b
h2 = 8m

Modulus of elasticity of concrete A A X = kb


2
E = 30 000 MN/m
D1 h1
Bending stiffness of the superstructure
is assumed to be infinity. d1
D =

The bottom of the pile can be


assumed to act as a hinge. A-A

Figure 1.
35. Problem, Solution 248

Bending stiffnesses
4
 d 
Di  E  i 
4 2  (1)
2
D1 = 3 054 MNm
2
D2 = 603 MNm

X = kb
D2 D1
a
b kb-J
J
h2 h1

-kbh2/D2
x

(kb-J)h1/D1
(kb-J)(h1+h2)/D2 (kb-J)h1/D2
M/D
R1
R2 vB R4 vA
R3 e2B e4A
e2A
e1B = e3B
e1A = e3A

Figure 2.
35. Problem, Solution 249

Deflection v at the spring (B ) and at the bottom of the pile (A ) (Figure 2)


vB  a  b   R1e1B  R2e2 B  R3e3 B

v A  a   R1e1 A  R2e2 A  R3e3 A  R4e4 A (28a, b)

By summation
-b = R 1(e 1A -e 1B )+R 2(e 2A -e 2B )+R 3(e 3A -e 3B )+R 4e 4A (29)

Load resultants
kb  J 2
R1  h2
2 D2 (30)
kb  J
R2  h1h2
D2 (31)
 kb 2
R3  h2
2 D2 (32)
kb  J 2
R4  h1
2 D1 (33)

Distances
e iA -e iB = h 1 (34)
e 4A = 2h 1/3 (35)

Spring deflection from Equation 29


h1h22 h12h2 h13
 
2 D2 D2 3 D1
bJ
kh1 h2 kh13
2 (36)
1 
D2 3 D1
= 0,006 m

Displacement at the top of the column from Equation 28a


Jh23 kb  J
a b  h1h22
3 D2 2 D2 (37)
= 0,051 m
35. Problem, Solution 250

ANSWER:

Displacement at the top of the column


a = 0,051 m
20111220 251
36. PROBLEM

A horizontal load caused by braking of a vehicle is acting on the top of an


intermediate column-type support. The size of the load according to the Finnish
Standard is
J = 0,500 MN
The intermediate support consists of a massive concrete pile with circular cross-
section (1) and an equal concrete column (2) shown in Figure 1. What is
displacement a at the top of the column caused by the braking load?

Solv the problem by using Finite Element Method (FEM)!

Elasticity of the soil is approximated by


a single spring, whose spring coefficient D=
k = 120 MN/m J J
The diameters of the pile and the column,
a
respectively, are
D2 h2
d1 = 1,200 m
d2 = 0,800 m d2
Height
h1 = 10 m b
h2 = 8m

Modulus of elasticity of concrete


2 A A X = kb
E = 30 000 MN/m

D1 h1
Bending stiffness of the superstructure
is assumed to be infinity.
D = d1

The bottom of the pile can be


assumed to act as a hinge.
A-A

Figure 1.
36. Problem, Solution 252

Bending stiffnesses
4
 d 
Di = E  i 
4 2  (1)
2
D1 = 3 054 MNm
2
D2 = 603 MNm

Spring coefficient
k 01 =  MN/m
= 1,0E+100 MN/m (at calculation)
k1 = 120 MN/m
k2 = 0 MN/m

Equilibrium condition of a beam element (positive directions are shown in Fig. 2)


é 12 D 6D 12 D 6 D ù
ê 3  ú
2
ê L L L3 L2 ú
ê 6D 4D 6D 2 D ú  v1   Q1 
    
ê L2 L L2 L ú 1   M1 
=
6 D ú  v2   Q2 
ê 12 D
(38)
6 D 12 D
ê 3  2  ú    
ê L L L3 L2 ú  2   M 2 
ê 6D 2D

6D 4D ú
êë L2 L L2 L úû

M1 Q1 Q2 M2
1 v1 D = EI v2 2

Figure 2.

Stiffness matrix of element 1


é 36,644 183,218 -36,644 183,218ù
ê ú
Ki =
e ê 183,218 1 221,451 -183,218 610,726ú
ê -36,644 -183,218 36,644 -183,218ú
ê 183,218 610,726 -183,218 1 221,451ú
ë û
36. Problem, Solution 253

Stiffness matrix of element 2


é 14,137 56,549 -14,137 56,549ù
ê ú
K ie = ê
56,549 301,593 -56,549 150,796ú
ê -14,137 -56,549 14,137 -56,549ú
ê 56,549 150,796 -56,549 301,593úû
ë

System stiffness matrix (Figure 3)


é k1  k 1
k12 1
k13 1
k14 0 0 ù
ê 11 01 1 1 1 ú
ê k21 = k12 k22 k23 k24 0 0 ú
êk = k k32 = k23 1
k33 2
 k11 1
 k1 k34 2
 k12 2
k13 2 ú
k14
K = ê 31 13 ú
ê k41 = k14 k42 = k24 k43 = k34 1
k44 2
 k22 2
k23 2
k24 ú
ê 2 2 ú
ê k51 = k15 k52 = k25 k53 = k35 k54 = k45 k33 k34 ú
êë k61 = k16 k62 = k26 k63 = k36 2 ú
k64 = k46 k65 = k56 k44 û

(39)
1,0E+100 183,218 -36,644 183,218 0,000 0,000
183,218 1221,451 -183,218 610,726 0,000 0,000
= -36,644 -183,218 170,781 -126,669 -14,137 56,549
183,218 610,726 -126,669 1523,044 -56,549 150,796
0,000 0,000 -14,137 -56,549 14,137 -56,549
0,000 0,000 56,549 150,796 -56,549 301,593

Loads at the node points


F1 = 0,000 MN
M1 = 0,000 MNm
F2 = 0,000 MN
M2 = 0,000 MNm
F3 = 0,500 MN
M3 = -2,192 MNm
36. Problem, Solution 254

k2
5
3 6
4
2 h2

e k1
3
1 4
2
1 h1

k02 k01
1
2

Figure 3.

Node displacements and rotations


w1 = 0,000 m
1 = 0,000 rad
w2 = 0,006 m
2 = 0,003 rad
w3 = 0,051 m
3 = 0,000 rad

ANSWER:

By finding such a value of M 3, that rotation


3 = 0
the unknown displacement
a = w3 (40)
= 0,051 m
is obtained.
36. Problem, Solution 255

Extra Study

Force vector of element i


Fi = Ki di
e e e
(41)
1. element
é Q1 ù é 0,000 m ù é 0,181 MN ù
êM ú ê ú ê ú
ê 1ú = Ke  ê 0,000 ú ê 0,000 MNmú
1 =
ê Q2 ú ê 0,006 m ú ê -0,181 MN ú
ê ú ê ú ê ú
ë M2 û ë 0,003 û ë 1,808 MNmû
2. element
é Q1 ù é 0,006 m ù é -0,500 MN ù
êM ú ê ú ê ú
ê 1ú = Ke  ê 0,003 ú ê -1,808 MNmú
2 =
ê Q2 ú ê 0,051 m ú ê 0,500 MN ú
ê ú ê ú ê ú
ë M2 û ë 0,000 û ë -2,192 MNmû
The shear force and bending moment distribution, respectively, and the deflections
at the node points are shown in Table 1. The corresponding curves are shown in
Figures 5, 6 and 7, respectively. Positive directions are shown in Figure 4.

Table 1.
xi Qi Mi wi
[m] [MN] [MNm] [m]
0,000 -0,181 0,000 0,000
10,000 -0,181 -1,808 0,006
10,000 0,500 -1,808 0,006
18,000 0,500 2,192 0,051
18,000 0 0

M1 M2
D = EI
1 2
Q1 Q2
v1 v2

Figure 4. Positive directions (compare to Figure 2).


36. Problem, Solution 256

0,6 Q [MN]
0,4
0,2 x [m]
0,0
0 2 4 6 8 10 12 14 16 18 20
-0,2
-0,4

Figure 5. Shear force distribution curve.

-3,0
-2,0
-1,0
0 5 10 15 20
0,0
x [m]
1,0
2,0 M
3,0 [MNm]

Figure 6. Bending moment distribution curve.

0 5 10 15 20
0,00
x [m]
0,01
0,02
0,03
0,04
0,05
0,06 w [m]

Figure 7. Deflection curve.


20111205 257
37. PROBLEM

The base slab shown in Figure 1 is partly supported on non-yielding rock


(support B) and partly on continuous (cases A and B) or non-continuous
(case C) elastic (Winkler) foundation, whose foundation coefficient is
3
c = 10 MN/m

Length of the base slab


L = 6m

A) Determine the area (distance e ) so that load


F = 1 MN/m
does not cause any tension at support B!

B) Determine the displacement of the other edge of the base slab (point A) when
load F is located so that the reaction force at support B is equal to zero?

C) If the foundation is approximated by


n = 7
equal springs located as shown in Figure 1, how big are the spring forces when
load F is located so that the reaction force at support B is equal to zero?

F
e

D = EI = 
A
B
n=7 6 5 4 3 2 1
L/7 L/7 L/7
L/7 L/7 L/7 L/7

Figure 1.
37. Problem, Solution 258

A)

Stress (Figure 2)
F M
 ( y)   y
A Iz (1)
a
e
F
M z
y
B
R
L/2 L/2

+

max min = 0

Figure 2.

Base area
A = bL (2)
where b is the width of the base slab.

Moment of inertia with respect to z -axis


bL3
Iz 
12 (3)

Stress at support B
F aF L
B   0
A Iz 2 (4)
F  6a 
 0 1  
bL  L (5)
37. Problem, Solution 259

L
a
6 (6)
a = 1,000 m

Distance
L
e a
2 (7)
 4,000 m

Stress at support B as function of load location e is shown in Figure 3.

 [MN/m2] 1,0

0,5
e [m]
0,0
6 5 4 3 2 1 0
-0,5

Figure 3.

ANSWER:

Distance
e  4,000 m

B)

Pressure under the base slab is the product of foundation coefficient c and
deflection v
p = cv (8)
Resultant of the pressure (Figure 4)
1
R  pL
2 (9)
37. Problem, Solution 260

1
 R  cvL
2 (10)

F
e

y
B
y
v

R
p 2L/3

Figure 4.

Bending moment equilibrium condition with respect to support B


MB  0 (11)
2
 0  eF  LR
3 (12)
2 1
 0  eF  L cvL
3 2 (13)
3F
 v (e )  e
cL2 (14)
= 0,008 e

Deflection of the left edge as function of load location e is shown in Figure 5.

When
e = 4,000 m
displacement is
v = 0,033 m
37. Problem, Solution 261

6 5 4 3 2 1 0
0,00
e [m]
0,02

0,04
v [m] 0,06

Figure 5.

ANSWER:

Deflection of the left edge


v (e ) = 0,008 e

C)

If spring force at the last spring is T n , then the force at spring i is (Figure 6)
i
Ti  Tn
n (15)

n n-1 3 2 1
Tn-1 ...
Tn

Figure 6.
37. Problem, Solution 262

Equilibrium condition
n i
 Tn  F (16)
i 1n
nF
 Tn 
n (17)
i
i 1

Force at spring i
iF
Ti 
n
(18)
i
i 1

ANSWER:

Spring forces are shown in Figure 7.


F1 = 0,036 MN/m
F2 = 0,071 MN/m
F3 = 0,107 MN/m
F4 = 0,143 MN/m
F5 = 0,179 MN/m
F6 = 0,214 MN/m
F7 = 0,250 MN/m
Checking
SFi = 1,000 MN/m

F 0,30
[MN/m]
0,20

0,10

0,00
7 6 5 4 3 2 1 Spring

Figure 7.
20120823 263
38. PROBLEM

A concrete slab shown in Figure 1, height h 1 and mean value of cylinder


compressive strength
h1 = 0,100 m
f cm = 38 MN/m2
is founded on a polystyrene layer, height h 2 and modulus of elasticity E 2
h2 = 0,100 m
2
E2 = 15 MN/m
on a gravel layer, height h 3 and modulus of elasticity E 3
h3 = 0,200 m
2
E3 = 40 MN/m
and on earth, foundation coefficient
3
c4 = 5 MN/m
Determine the deflection curve and the distribution curves of bottom pressure,
bending moment and shear force, respectively, of the floor!
To solve the problem, divide the structure by using five node points as shown in
Fig. 1 and suppose, that the deflection curve is linear between two adjacent nodes.
Solve the bending moments at the nodes by using two different ways:
1) differential equation of bending and
2) concentrating the pressure under the structure at the nodes!
Dimension a and loading per length
a = 2,500 m
F1 = 0,050 MN/m
F2 = 0,025 MN/m
F1 F2
1 2 3 4 5
K h1
E2 h2
E3 h3

c4
a a a a

Figure 1.
38. Problem, Solution 264

Resultant of the bottom pressure at node point R i is obtained from the respective
areas of the bottom pressure distribution curve shown in Figure 2.

ì dx 1 s i +1 - s i dx
ïs i +
2 2 2 2
, i =1
ï
ï dx 1 s i - s i -1 dx dx 1 s i +1 - s i dx ìi Î {2Kn - 1}
Ri = ís i - +si + , í
ï 2 2 2 2 2 2 2 2 îi Î N
ï dx 1 s i - s i -1 dx
ïs i - , i=n
î 2 2 2 2

(1)
q

D = EI
c1 c2 c3 cn-1 cn
dx/2 dx/2 dx/2 dx/2 dx/2
1 2 3 n-1 n
si = ciwi sn
s1 s3
s2
R1 R2 R3 Rn
Figure 2.

Because pressure s i is a product of foundation coefficient c i and deflection w i ,


then
si = ciwi (2)
Thus the reaction force at node i is
ì cdx
ï 8 ( 3wi + wi + 1 ), i = 1
ï
ï cdx
Ri = í ( wi -1 + 6wi + wi +1 ), i Î {2K n - 1}, i Î N
ï 8 (3)
ï cdx
ï 8 ( wi -1 + 3wi ), i = n
î
when foundation coefficient c is uniform.
38. Problem, Solution 265

Reaction forces (Figure 3)


ca (4)
R1 = ( 3w1 + w2 )
8
ca (5)
R2 = ( w1 + 6w 2 + w3 )
8
ca
R3 = ( w2 + 6w3 + w4 ) (6)
8
ca
R4 = ( w3 + 6w4 + w5 ) (7)
8
ca
R5 = ( w4 + 3w5 )
8 (8)

F1 F2

1 2 3 4 5

R1 R2 R3 R4 R5

a a a a

Figure 3.

Derivation of deflection between point i and its adjacent points


w i - w i -1
w'i -1 / 2 =
dx (9)
w - wi
w'i +1 / 2 = i +1
dx (10)
Second derivation of deflection
w i' +1 / 2 - wi' -1 / 2
wi'' =
dx (11)
Bending moment
M i = –EI i w i '' (12)
38. Problem, Solution 266

wi' + 1 / 2 - w'i -1 / 2
Þ M i = - EI i
dx (13)
wi + 1 - w i - wi + wi -1
Þ M i = - EI i
dx 2 (14)
EI i
Þ Mi = (- w i -1 + 2wi - wi +1 )
dx 2 (15)
Bending moment at point 2, 3 and 4, respectively,
EI
M 2 = R1a = - 2 ( w1 - 2 w2 + w3 )
a (16)
EI
M 3 = 2 R5a + R4a = - ( w 2 - 2 w 3 + w4 )
a2 (17)
EI
M 4 = R5a = - ( w 3 - 2 w4 + w5 )
a2 (18)
Equilibrium condition of vertical forces (Figure 3)
F 1+F 2 = R 1+R 2+R 3+R 4+R 5 (19)
Equilibrium condition of moment with respect to point 3
2aR 1+aR 2 = aR 4+2aR 5+aF 1 (20)
System of equilibrium conditions (from Equations 16…20)
ì ca 2 EI EI EI
ï ( 3 w1 + w2 ) = - 2 w1 + 2 2 w2 - 2 w3
ï 8 a a a
ï 2 2
ï2 ca ( w4 + 3 w5 ) + ca ( w3 + 6 w4 + w5 ) = - EI w2 + 2 EI w3 - EI w4
ï 8 8 a2 a2 a2
ï 2
ï ca EI EI EI
ï 8 ( w 4 + 3 w 5 ) = - w 3 + 2 w 4 - w
2 2 2 5
ï a a a
ï
í F1 + F2 = ca (3 w1 + w2 + w1 + 6 w2 + w 3 + w2 + 6 w3 + w4 +
ï 8 (21a…e)
ï + w 3 + 6 w4 + w5 + w4 + 3 w 5 )
ï
ï
ï 2 2 2
ï0 = 2 ca ( 3 w + w ) + ca ( w + 6 w + w ) - ca ( w + 6 w + w ) +
ï 1 2 1 2 3 3 4 5
8 8 8
ï
ï ca 2
ïî - 2 ( w4 + 3 w5 ) - aF1
8
38. Problem, Solution 267

ìæ ca 2 EI ö æ ca 2 EI ö EI
ç
ï 3 + ÷ w1 + ç - 2 ÷ w2 + w3 =0
ç
ïè 8 2 ÷ ç 2 ÷ a2
a ø è 8 a ø
ï
ï EI æ ca 2 EI ö÷ æ ca 2 EI ö ca 2
w
ï 2 2 ç+ ç - 2 w + ç 8 + ÷ w + 7 w5 = 0
8 2÷ 3 ç 8 2÷ 4 8
ï a è a ø è a ø
ï
ï EI æ ca 2 EI ö æ ca 2 EI ö
Þ í 2 w3 + ç - 2 2 ÷ w4 + ç 3 + 2 ÷ w5 =0
a ç 8 a ÷ ç 8 a ÷
ï è ø è ø
ï ca (22a…e)
ï4 w1 + 8 ca w2 + 8 ca w3 + 8 ca w4 + 4 ca w5 = F1 + F2
ï 8 8 8 8 8
ï ca ca ca ca
ï7 w1 + 8 w2 - 8 w4 - 7 w5 = F1
ï 8 8 8 8
ï
î

Constant
ko = 22 000 MN/m2
f cmo = 10 MN/m2
Modulus of elasticity of concrete
0, 3
æ fcm ö (23)
E = ko çç ÷
è fcmo ÷ø
= 32 837 MN/m2

Moment of inertia of the slab per length


3
I = h 1 /12 (24)
4
= 0,000083 m /m
Bending stiffness
D = EI (25)
2
= 2,736 MNm

Foundation coefficient (Figure 4; Finnish concrete floor standard BY 31, p. 39)


1
c=
h2 h3 1 (26)
+ +
E 2 E 3 c4
3
= 4,724 MN/m
38. Problem, Solution 268

E2 h2
E3 h3
c4

Figure 4.

Matrix
11,511 2,815 0,438 0,000 0,000
0,000 0,438 2,815 29,965 25,837
[K ] = 0,000 0,000 0,438 2,815 11,511
5,906 11,811 11,811 11,811 5,906
10,335 11,811 0,000 -11,811 -10,335

Inverse matrix
0,109 -0,008 -0,001 -0,002 -0,024
-0,098 0,048 -0,002 -0,008 0,114
[K ]-1 = 0,046 -0,088 0,046 0,102 -0,110
-0,002 0,048 -0,098 -0,008 0,006
-0,001 -0,008 0,109 -0,002 0,003

Elements of force vector (sub-indexes refer to Equations 22)


Fa = 0,000 MNm/m
Fb = 0,000 MNm/m
Fc = 0,000 MNm/m
Fd = 0,075 MN/m
Fe = 0,050 MN/m

System of equilibrium conditions


{w } = [K ]-1{f } (27)
38. Problem, Solution 269

Elements of displacement vector


w 1 = -0,001335 m
w 2 = 0,005123 m
w 3 = 0,002166 m
w 4 = -0,000263 m
w 5 = -0,000018 m

Shear force
M i +1 - M i
Qi =
a (28)

ANSWER:

Deflections, bottom pressures, bending moments and shear forces, respectively, at


the node points are shown in Table 1 and the corresponding curves in Figures 5…8,
respectively.

Table 1.
xi wi si Ri Mi Qi
(2) (3) (15) (28)
2
[m] [m] [MN/m ] [MN] [MNm] [MN]
0,000 -0,00134 -0,00631 0,00165 0,00000 0,00165
2,500 0,00512 0,02420 0,04661 0,00412 0,00165
2,500 0,00512 0,02420 0,04661 0,00412 -0,00174
5,000 0,00217 0,01023 0,02637 -0,00023 -0,00174
5,000 0,00217 0,01023 0,02637 -0,00023 -0,00038
7,500 -0,00026 -0,00124 0,00084 -0,00117 -0,00038
7,500 -0,00026 -0,00124 0,00084 -0,00117 0,00047
10,000 -0,00002 -0,00009 -0,00047 0,00000 0,00047
10,000 0 0 0
38. Problem, Solution 270

-0,002
0 2 4 6 8 10
0,000 x [m]
0,002
0,004
0,006 v [m]

Figure 5. Deflection curve.

-0,02
0 2 4 6 8 10
0,00 x [m]

0,02

0,04 s [MN/m2]

Figure 6. Bottom pressure distribution curve.

-0,002 0 2 4 6 8 10
0,000
x [m]
0,002
0,004
0,006 M [MNm]

Figure 7. Bending moment distribution curve.

-0,002
0 2 4 6 8 10
0,000
x [m]

0,002 Q [MN]

Figure 8. Shear force distribution curve.


20111205 271
39. PROBLEM

Due to temperature variation, the top of a foundation wall shown in Figure 1 is


moving laterally
d = 0,020 m

Determine the bottom pressure as function of the foundation coefficient, when the
latter one varies between
c min = 10 MN/m3
and
c max = 300 MN/m3

Determine the bottom pressure also in the case when


c =  MN/m3

Vertical force
Fx = 0,300 MN/m

Modulus of elasticity of the wall


2
E = 15 000 MN/m

Dimensions
Fx
d = 0,500 m d
h = 3,000 m
b = 1,000 m
t = 0,200 m
h
t
y
z
d

b
x
Figure 1.
39. Problem, Solution 272

Resultant of bottom pressure (Figure 2)


R   s ( z )dz (1)

My
z
u(z) y
x s
R
b

Figure 2.

Bottom pressure
s (z ) = cu (z ) (2)

Deflection
u (z ) = y z (3)
where y is the rotation angle.

Moment with respect to y -axis


M y   s ( z )zdz (4)

 cy  z 2dz
(5)

Moment of inertia of the bottom cross-section (l ) with respect to y -axis

I y ,l   z 2dz (6)

Hence moment
M y (c ) = c y I y,l (7)
39. Problem, Solution 273

On the other hand, moment as function of virtual horizontal force H (Figure 3)


M y = H (h +d ) (8)

d
w
H 

d
y

Figure 3.

From Equations 7 and 8


H (h  d )
y
cI y, l (9)

Deflection due to rotation angle at the top of the wall


  ( h  d )y (10)
2
H (h  d )

cI y ,l (11)

Deflection due to horizontal force at the top of the wall (m )


Hh 3
w
3 D y ,m (12)
39. Problem, Solution 274

Total deflection at the top of the wall


d (c )    w (13)
2 3
H (h  d ) Hh
 
cI y , l 3 D y ,m (14)

Horizontal force
d
H (c ) 
( h  d )2 h3 (15)

cI y ,l 3 D y ,m

Bottom pressure
Fx M y
s (z)   z
Al I y ,l (16)
Fx H ( h  d )
  z
Al I y ,l (17)
F d (h  d )
 s (c , z )  x  z
Al  ( h  d )2 h 3  (18)
I y ,l   
 cI y ,l 3 EI 
 y ,m 
F d (h  d )
 x z (19)
Al ( h  d )2 h3 I y ,l

c 3 EI y ,m

Bottom area per unit length


Al = b (20)

Moment of inertia of the base slab and the wall with respect to y -axis per unit
length, respectively,
b3
I y ,l 
12 (21)
3
t
I y ,m 
12 (22)
39. Problem, Solution 275

Bottom pressure
Fx d (h  d )
s (c , z )   z
b ( h  d ) 2 h 3b 3 (23)

c 3 Et 3

When the foundation coefficient approaches infinity


Fx 3 Et 3d (h  d )
s (z)   z
b h3b3 (24)

ANSWER:

The extreme values of the bottom pressure are shown in Table 1 and the
corresponding distribution curves are shown in Figure 4. Correspondingly, the
stresses at the edges of the slab are shown in Figure 5.

Table 1. Extreme values of bottom pressure.


s min s max
z = -b /2 z = b /2
[MN/m ] [MN/m2]
2

c min = 10 MN/m3 0,273 0,327


c max = 300 MN/m3 -0,002 0,602
c =  MN/m3 -0,167 0,767

minimum 1,0 s [MN/m2]


maximum 0,8
infinite 0,6
0,4
0,2
z [m]
0,0
-0,5 -0,3 -0,1
-0,2 0,1 0,3 0,5
-0,4

Figure 4. The extreme values of the bottom pressure.


39. Problem, Solution 276

0,7 s [MN/m2]
0,6
0,5
0,4 z = b/2
z = -b/2
0,3
0,2
0,1 c [MN/m3]
0,0
0 50 100 150 200 250 300 350
-0,1

Figure 5. The stresses at the edges of the slab as function of foundation coefficient.
20121102 277
40. PROBLEM

A bridge abutment is shown in Figure 1.


Examine is the load carrying capacity adequate by using Eurocode.
y
Dimensions
t
a = 2,000 m G/2+Q/4
b = 1,900 m
J/2+K/2
B = 4,500 m c
q C z
c = 2,700 m x
L
C = 8,000 m c
d = 1,500 m J/2+K/2
D = 1,000 m A t
G/2+3Q/4 A
h0 = 2,000 m
H = 3,200 m
A-A
L = 10,000 m
r = 0,500 m q
G+Q
s = 0,600 m u r d
t = 0,400 m J+K
a 45°
u = 0,600 m 1:k 2×s/2
H
v = 0,500 m
b
gm1, j1 z
Inclination of the embankment v
h0
k = 1,25 w
gm2, j2 gc y
D
x
gm3, j3 B
Figure 1.

Unit weight (of reinforced concrete, water and soil layers)


gc = 0,025 MN/m3
gw = 0,010 MN/m3
g m1 = 0,021 MN/m3
g m2 = 0,011 MN/m3 (Ground water level is taken account.)
g m3 = 0,012 MN/m3 (Ground water level is taken account.)
20121102 278
40. PROBLEM

Angle of internal friction of the earth


j1 = 38 °
j2 = 38 °
j3 = 36 °
Compressing: vibrating slab, 400 kg; 4 times; layer height 0,350 m
(RIL 207-2009, Ch. 9.5.5.).
Loading
Weight of the bridge deck on the bearings: permanent vertical action
G = 2,370 MN
Traffic load from the deck on the bearings: Tandem load
and uniformly distributed load (variable vertical action)
QT = 0,940 MN (Comp. EN 1991-2, Ch. 4.3.2.)
Q UD = 0,345 MN
Traffic load on the embankment (variable vertical action)
q = 0,020 MN/m2 (NCCI 1, Ch. B.4.8.)
Brake load (variable horizontal action)
J = 0,387 MN (NCCI 1, Eq. B.1.)
Bearing friction (variable horizontal action)
K = kG
where
k = 0,060 (NCCI 1, Ch. H.3.)
Water level is assumed to be constant.
Finnish Standards:
- Liikennevirasto: Eurokoodin soveltamisohje, Siltojen kuormat ja
suunnitteluperusteet - NCCI 1. 20/2011. Helsinki 2011.
http://portal.liikennevirasto.fi/sivu/www/f/liikennevirasto/julkaisutoiminta/
- Liikennevirasto: Eurokoodin soveltamisohje, Geotekninen
suunnittelu - NCCI 7. 12/2011. Helsinki 2011.
http://portal.liikennevirasto.fi/sivu/www/f/liikennevirasto/julkaisutoiminta/
- SFS-EN 1991-2. Eurokoodi 1. Rakenteiden kuormat. Osa 2: Siltojen
liikennekuormat.
- Suomen Rakennusinsinöörien Liitto RIL ry: RIL 207-2009. Geotekninen
suunnittelu. Eurokoodin EN 1997-1 suunnitteluohje.
40. Problem, Solution 279

LOADS

Self Weight

Foundation slab
Horizontal load
G c 1 = g c BDL (1)
= 1,125 MN
Distance from origin
e c 1 = v +b +s /2-B /2 (2)
= 0,450 m
Moment
M c 1 = G c 1e c 1 (3)
= 0,506 MNm

Backwall
G c 2A = g c [rH (C- 2t )] (4)
= 0,288 MN
e c 2A = r /2+s /2 (5)
= 0,550 m
M c 2 = G c 2A e c 2A (6)
= 0,158 MNm

Breast wall (center of gravity in origin)


G c 2B = g c [s (H-d )C ] (7)
= 0,204 MN

Side walls (2 walls)


G c 3 = 2g c bHt (8)
= 0,122 MN
e c 3 = b /2+s /2 (9)
= 1,250 m
40. Problem, Solution 280

M c 3 = G c 3e c 3 (10)
= 0,152 MNm

Wing wall (2 walls)


Rectangle
G c 4A = 2g c aut (11)
= 0,024 MN
e c 4A = a /2+b +s /2 (12)
= 3,200 m
M c 4A = G c 4A e c 4A (13)
= 0,077 MNm
Triangle
2
G c 4B = 2g c ta /2 (14)
= 0,040 MN
e c 4B = a /3+b +s /2 (15)
= 2,867 m
M c 4B = G c 4B e c 4B (16)
= 0,115 MNm
Sum
Gc4 = G c 4A +G c 4B (17)
= 0,064 MN
Mc4 = M c 4A +M c 4B (18)
= 0,191 MNm

Sum of self weight


G c = G c 1+G c 2A+Gc 2B+G c 3+G c 4 (19)
= 1,599 MN
M c = M c 1+M c 2+M c 3+M c 4 (20)
= 1,008 MNm
40. Problem, Solution 281

Weight of Filling

Height of embankment from top of the base slab (Figure 2)

(21)
- x /k
ì h slab
Rear edge of the base
hi = miní 0 i
x A = -(v î+b
H +s /2) (22)
= -2,700 m
hA = 3,200 m
Rear edge of the side wall
x B = -(b +s /2) (23)
= -2,200 m
hB = 3,200 m
Front edge of the breast wall
x C = s /2 (24)
= 0,300 m
hC = 1,760 m
Front edge of the base slab
x D = B- (s /2+b+v ) (25)
= 1,800 m
hD = 0,560 m e z
Distance u
e = (H-h D )k r d (26)
= 3,300 m
1:k 2×s/2
H
hB
b
hA
v hD
h0 hC

D
y x
B

Figure 2.
40. Problem, Solution 282

Filling between side walls (Figure 3)


G m 1 = g m 1(v +b-r )(C- 2t )H (27)
= 0,919 MN
e m 1 = (v +b-r )/2+r +s /2 (28)
= 1,750 m
M m 1 = G m 1e m 1 (29)
= 1,609 MNm
y
m3
Filling in front of breast wall t m4
Rectangular prism
G m 2A = g m 1C (B-s-b-v )h D (30)
c
= 0,141 MN x
q L C z
e m 2A = -[(B-s-b-v )/2+s /2] (31)
= -1,050 m m1 c m2
M m 2A = G m 2A e m 2A (32)
A A
= -0,148 MNm t m4
Trilateral prism m3
G m 2B = g m 1C (B-s-b-v )(h C -h D )/2 A-A (33)
= 0,151 MN
z
e m 2B = -[(B-s-b-v )/3+s /2] (34)
m1
= -0,800 m r d
M m 2B = G m 2B e m 2B gm1 a+u
(35)
j 1:k
2×s/2
= -0,121 MNm 1 v m4B
Sum v
H b
G m 2 = G m 2A +G m 2B (36)
m4A
= 0,292 MN m3B m2B
M m 2 = M m 2A +M m 2B m3A m2A (37)
= -0,269 MNm D x
y
B
Figure 3.
40. Problem, Solution 283

Filling out of the breast walls, the both sides


Rectangular prism
G m 3A = g m 1B (L-C )h D (38)
= 0,106 MN
e m 3A = v +b +s /2-B /2 (39)
= 0,450 m
M m 3A = G m 3A e m 3A (40)
= 0,048 MNm
Trilateral prism
G m 3B = g m 1B (L-C )*(h A -h D )/2 (41)
= 0,249 MN
e m 3B = v +b +s /2-B /3 (42)
= 1,200 m
M m 3B = G m 3B e m 3B (43)
= 0,299 MNm
Trilateral prism is deducted, if h A >H
ì g m1 ( B - e )( L - C )( H - hA )
ï , hA > H
Gm 3C = í 2 (44)
ïî0, hA £ H
= 0,000 MN
e m 3C = v +b +s /2- (B-e )/3 (45)
= 2,300 m
M m 3C = G m 3C e m 3C (46)
= 0,000 MNm
Sum
G m 3 = G m 3A +G m 3B +G m3C (47)
= 0,355 MN
M m 3 = M m 3A +M m 3B +M m 3C (48)
= 0,347 MNm
40. Problem, Solution 284

Filling behind of the breast walls, the both sides


Rectangular prism
G m 4A = 2g m 1vt (H-a-u ) (49)
= 0,005 MN
e m 4A = v /2+b +s /2 (50)
= 2,450 m
M m 4A = G m 4A e m 4A (51)
= 0,012 MNm
Trilateral prism
G m 4B = 2g m 1tv 2/2 (52)
= 0,002 MN
e m 4B = 2v /3+b +s /2 (53)
= 2,533 m
M m 4B = G m 4B e m 4B (54)
= 0,005 MNm
Sum
G m 4 = G m 4A +G m 4B (55)
= 0,007 MN
M m 4 = M m 4A +M m 4B (56)
= 0,018 MNm

Sum of filling
G m = G m 1+G m 2+G m 3+G m 4 (57)
= 1,574 MN
M m = M m 1+M m 2+M m 3+M m 4 (58)
= 1,704 MNm
40. Problem, Solution 285

Permanent Earth Pressure

Coefficient for earth pressure in rest


K on = 1-sinj n (59)
Ko1 = 0,384
Ko2 = 0,384

Earth pressure at rest at the top (y ) and bottom (a ) of layer n , respectively


ì ì K onq, n = 1, [n Î N ]
ï ï n-1
ï pony = í
ï ï K on å g i hi , n ³ 2
í î i =1 (60)
ï n
ïp = K
ï ona on å g i hi , n ³ 1
î i =1
p o1y = 0,000 MN/m2
p o 1a = K o 1g m 1H (61)
= 0,026 MN/m2
p o2y = K o 2g m 1H (62)
= 0,026 MN/m2
p o2a = K o 2(g m 1H +g m 2D ) (63)
= 0,030 MN/m2

Resultant of earth pressure at rest at layer n and the q


total sum of resultants P o (Figure 4)

(64)
P o 1 = [p o 1y H +0,5(p o 1a -p o 1y )H ]C (65)
Po1
= 0,331 MN gm1, j1
P o 2 = é[p o 2y D +0,5(p o 2a -p o 2y )Dù]L

1
( )
Pon = ê ponyhn + pona - pony hn ú L eo1
0,2792 MN
(66)
û w
P o = P o 1+P o 2 (67)
P gm2, j2
= 0,610 MN/m eo2 o2

Figure 4.
40. Problem, Solution 286

Corresponding distances from the bottom of the base slab wall

+ ( pona - pony ) n ì max h , n = [1,n ]


hn 1 h n
pony
eon = 2 2 3 +ïå i max
í i = n+1
pony + ( pona - pony )
1 ï0, n = n (68)
2 î max

e o 1 = [p o1y H /2+0,5(p o1a -p o1y )H /3]/[p o1y +0,5(p o1a -p o1y )]+ (69)
= 2,067 m
e o 2 = [p o2y D /2+0,5(p o2a -p o2y )D /3]/[p o2y +0,5(p o2a -p o2y )] (70)
= 0,487 m
q
Moments z Pl1
M o 1 = P o 1e o 1 zo Pl2 (71)
= 0,683 MNm
M o 2 = P o 2e o 2 (72)
= 0,136 MNm el1 gm1, j1
M o = M o 1+M o 2 el2 (73)
= 0,819 MNm
w

Compression of filling (Figure 5)


Pressure (RIL 207-2009 , Fig. 9.1S.) Figure 5.
p = 0,016 MN/m2
Depth of the fold of the earth pressure distribution curve
z = 0,500 m
Depth of the lower edge of the additional pressure distribution curve
p
zo =
K o1g m1 (74)
= 1,982 m
<H

Additional earth pressure at the fold, i.e. at depth z ,


p l = p -K o 1g m 1z (75)
2
= 0,012 MN/m
40. Problem, Solution 287

Resultant force due to additional pressure above z


1
Pl 1 = p l zC
2 (76)
= 0,024 MN
2
e l1 = H + D - z
3 (77)
= 3,867 m
Ml1 = P l 1e l 1 (78)
0,093 MNm

Resultant force due to additional pressure under z


1
Pl 2 = p l ( z o - z )C
2 (79)
= 0,071 MN/m
1
e 2l = H + D - z - ( z o - z )
3 (80)
= 3,206 m
Ml2 = P l 2e l 2 (81)
0,227 MNm

Resultant force due to additional pressure


P l = P l 1+P l 2 (82)
= 0,095 MN
M l = M l 1+M l 2 (83)
= 0,320 MNm

Sum of the earth pressure


P P = P o +P l (84)
= 0,705 MN
M P = M o +M l (85)
= 1,139 MNm
40. Problem, Solution 288

Buoyancy

Permanent buoyancy due to base slab


N = g w BDL (86)
= 0,450 MN
e N = v +b +s /2-B /2 (87)
= 0,450 m
M N = Ne N (88)
= 0,203 MNm

Traffic Load from the Deck

One-sided variable load


Q a,T = (3-1)Q T /4 (89)
= 0,470 MN
Q a,UD = (3-1)Q UD /4 (90)
= 0,173 MN
M x,QT = Q a,T c (91)
= 1,269 MNm
M x,QUD = Q a,UD c (92)
= 0,466 MNm

Traffic Load on the Embankment

Variable vertical action


Q p = q (v +b )*(C- 2t ) (93)
= 0,346 MN
e Qp = (v +b )/2+s /2 (94)
= 1,500 m
M y,Qp = Q p e Qp (95)
= 0,518 MNm
40. Problem, Solution 289

Variable Earth Pressure due to Traffic Load

Earth pressure
p oq 1 = K o 1q (96)
= 0,008 MN/m2
p oq 2 = K o 2q (97)
= 0,008 MN/m2

Resultant, distance from origin and moment at the layers


P oq 1 = p oq 1H (C- 2t ) (98)
= 0,177 MN
P oq 2 = p oq 2D (C- 2t ) (99)
= 0,055 MN
e oq 1 = D +H /2 (100)
= 2,600 m
e oq 2 = D /2 (101)
= 0,500 m
M oq 1 = P oq 1e oq 1 (102)
= 0,460 MNm
M oq 2 = P oq 2e oq 2 (103)
= 0,028 MNm

Sum
P oq = P oq 1+P oq 2 (104)
= 0,232 MN
M oq = M oq 1+M oq 2 (105)
= 0,488 MNm
40. Problem, Solution 290

Brake load

Variable horizontal action


J = 0,387 MN
e J = H-d+D (106)
= 2,700 m
M J = Je J (107)
= 1,045 MNm

Bearing friction

Variable horizontal action


K = kG (108)
= 0,142 MN
e K = H-d+D (109)
= 2,700 m
M K = Ke K (110)
= 0,384 MNm
40. Problem, Solution 291

Summary of Loads

Summary is presented in Table 1.

Table 1.
Vertical Horizontal Moment Moment Combin. Note
Force Force value
Vz Hx My Mx Y0
[MN] [MN] [MNm] [MNm] -
Permanent
1a) Self weight (-G c ) -1,599 - -1,008 -
1b) Filling (-G m ) -1,574 - -1,704 -
1c) Deck (-G ) -2,370 - - -
1d) E. pressure (-P P - 0,705 1,139 -
1e) Buoyancy (N ) 0,450 - 0,203 -
V z,permanent H x,permanentM y,permanentM x,permanent
S -5,093 0,705 -1,371 0,000
Variable
2. Traffic
a) From deck (-Q T ) -0,940 - - 1,269 0,75
b) From deck (-Q UD -0,345 - - 0,466 0,40
c) From bank (-Q p ) -0,346 - -0,518 - 0,40
d) E. pressure (p oq ) - 0,232 0,488 - 0,75 1
e) E. pressure (p oq ) - 0,232 0,488 - 0,40 2
f) Brake (J ) - 0,387 1,045 -
3. Bearing friction (K - 0,142 0,384 - 0,60

1) Is used, when there is not tandem load on the deck. (NCCI 1, Table. G1.)
2) Is used, when there is tandem load on the deck.
40. Problem, Solution 292

Combinations of Actions

In the case of bearing capacity and sliding Ultimate Limit State (ULS)
GEO/STR Design Approach (DA)2* is used. (NCCI 7, Ch. 5.1.2)

Combination of sets of partial factors of design approach 2


A1 "+" M1 "+" R2
where
– A refers to loads or actions of loads (Table A.3 of Appendix A),
– M refers to soil parameters (Table A.4),
– R refers to resistance (Table A.5),
– numbers 1 and 2 refer to sets 1 and 2 and
– "+" implies "to be combined with".

Load combination
ìï1, 35 K FI Gkj , sup + 0,9Gkj , inf
Fd = í
ïî1,15 K FI Gkj , sup + 0,9Gkj , inf + g Q1 K FI Qk1 + K FI S (g Qiy 0 i Qki )
(111)
where index sup refers to upper and inf to lower characteristic value of permanent
action.

Structure belongs to reliability class RC2 (NA SIS-EN 1990 , Table


B1), thus factor of actions (SFS-EN 1990 , Table B3)
K FI = 1,0

Partial factor of the leading variable action Q k 1 and accompanying variable


action Q ki
g Q1 üï ì1, 35 traffic load
ý= í
g Qi ïþ î1,50 accompanyi ng variable load

Combination values are given in Table 1.

One loading case is studied first and after that the others in Table 2.
40. Problem, Solution 293

Characteristic values of loads: vertical load, horizontal one and moment


V k = g Gj,sup,k (G c +G m +G )- g Gj,inf,k N +g Q,T,k Q T +g Q,UD,k Q UD +…
... +g Q,P,k Q p (112)
= 5,093 MN
H k = g Gj,inf,k P P +g Q,P,k P oq +g Q,J,k J +g Q,K,k K (113)
= 0,705 MN
M yk = - g Gj,sup,k (M c +M m )+g Gj,inf,k (M P +M N )- g Q,P,k M yQp +…
…+g Q,poq,k M oq +g Q,J,k M J +g Q,K,k M K (114)
= -1,371 MNm
M xk = g Q,T,k M xQT +g Q,UD,k M xQUD (115)
= 0,000 MNm

Design value of the vertical load


V d = g Gj,sup (G c +G m +G )- g Gj,inf N (116)
= 7,078 MN

Load eccentricities (x and y values)


M yk
ex =
Vk (117)
= -0,269 m
M xk
ey =
Vk (118)
= 0,000 m

Effective foundation area (Figure 6)


æ s ö
Bt = 2ç B - v - b - - e x ÷
è 2 ø (119)
= 4,138 m
æL ö
Lt = 2ç - e y ÷
è2 ø (120)
= 10,000 m
40. Problem, Solution 294

ìB
B' = miní t
î Lt (121)
= 4,138 m
ì Bt
L' = max í
î Lt (122)
= 10,000 m
A' = B'L' (123)
= 41,382 m2
L/2 L/2

B/2
Bt/2 eB

R B/2
Bt/2 eL

Lt/2 Lt/2
Figure 6.

Design value of bottom pressure


V
qd = d
A' (124)
= 0,171 MN/m2

Design value of cohesion


c' = 0 MN/m2

Unit weight of soil above the foundation level


g = g m2 (125)
3
= 0,011 MN/m

Unit weight of soil below the foundation level


g ' = g m3 (126)
= 0,012 MN/m3
40. Problem, Solution 295

Effective overburden pressure at the level of the foundation area


q' = h d g (127)
= 0,006 MN/m2

Inclination of the foundation base


a = 0°

Partial factor for angle of shearing resistance of the soil (Finnish annex for
EN 1997-1 , Application A. Table A.4.)
gj = 1,000

Design value of the internal friction angle of the earth


æ tanj 3 ö
j ' = arctan ç ÷ (128)
ç gj ÷
è ø
= 36,000 °

Bearing capacity factors


æ j'ö
N q = tan 2 ç 45° + ÷ e p tan j '
è 2ø (129)
= 37,752
é æ j' ö ù
N g = 2 ê tan 2 ç 45° + ÷ep tan j ' - 1ú tan j '
ë è 2ø û (130)
= 53,405

Factors for the inclination of the base


bq üï 2
ý = (1 - a tan j ' ) (131a, b)
bg ïþ

= 1,000
40. Problem, Solution 296

Shape factors of the base slab


B'
sq = 1 + sinj '
L' (132)
= 1,243
B'
sg = 1 - 0, 3
L' (133)
= 0,876

Parameter m
m = mB (134)
B'
2+
= L'
(135)
B'
1+
L'
= 1,707

Inclination factors of the load resultant


m
æ Hk ö
iq = çç 1 - ÷÷ (136)
è Vk + A' c ' cot j ' ø

= 0,775
m +1
æ Hk ö (137)
ig = çç 1 - ÷÷
è Vk + A' c' cotj ' ø
= 0,668

Partial resistance factor for bearing (Finnish annex for EN 1997-1 , Appendix A,
Table A.5)
gR = 1,55
40. Problem, Solution 297

Bearing capacity
R
qm =
A' (138)
1
= c' N c bc sc ic + q' N q bq sq iq + g ' B' N g bg sg ig
2 (139)
= 1,000 MN/m2

Design value of the bearing capacity


q
qmd = m
gR (140)
2
= 0,645 MN/m

Ratio between bearing capacity and bottom pressure


q
n = md
qd (141)
= 3,773 > 1, OK

ANSWER:

Bearing capacity in other loading cases is calculated in Table 2.


Part of all possible loading cases is presented.
Bearing capacity is adequate.
40. Problem, Solution 298

Table 2.
a b c d e f g h
g Gj,sup 1,35 1,15 1,15 1,15 1,15 1,15 1,15 1,15
g Gj,inf 0,90 0,90 0,90 0,90 0,90 0,90 0,90 0,90
g Q,T 0 1,35 0,00 1,35 1,01 0,00 1,01 0,00
g Q,UD 0 1,35 1,35 0,00 0,54 0,54 0,00 0,00
g Q,P 0 0,54 0,54 0,54 1,35 1,35 1,35 1,35
g Gj,sup,k 1 1 1 1 1 1 1 1
g Gj,inf,k 1 1 1 1 1 1 1 1
g Q,T,k 0 1 0 1 0,75 0 0,75 0
g Q,UD,k 0 1 1 0 0,40 0,40 0 0
g Q,P,k 0 0,40 0,40 0,40 1 1 1 1
g Q,poq,k 0 0,40 0,75 0,40 1 1 1 1
g Q,J,k 0 1 1 1 1 1 1 1
g Q,K,k 0 0,60 0,60 0,60 0,60 0,60 0,60 0,60
V k 6,022 7,464 6,524 7,119 7,258 6,553 7,120 6,415
H k 1,439 2,063 2,195 2,063 2,290 2,290 2,290 2,290
M yk -0,675 1,668 2,065 1,668 1,995 1,995 1,995 1,995
M xk 0,000 1,735 0,466 1,269 1,138 0,186 0,952 0,000
V d 8,332 7,037 7,037 7,037 7,037 7,037 7,037 7,037
e x -0,112 0,223 0,317 0,234 0,275 0,304 0,280 0,311
e y 0,000 0,232 0,071 0,178 0,157 0,028 0,134 0,000
B' 3,824 3,153 2,967 3,131 3,050 2,991 3,040 2,978
L' 10,00 9,54 9,86 9,64 9,69 9,94 9,73 10,00
A' 38,24 30,06 29,25 30,20 29,55 29,74 29,58 29,78
q d 0,218 0,234 0,241 0,233 0,238 0,237 0,238 0,236
s q 1,225 1,194 1,177 1,191 1,185 1,177 1,184 1,175
s g 0,885 0,901 0,910 0,903 0,906 0,910 0,906 0,911
m 1,723 1,751 1,769 1,755 1,761 1,769 1,762 1,771
i q 0,625 0,567 0,484 0,549 0,513 0,467 0,505 0,458
i g 0,475 0,411 0,321 0,390 0,351 0,304 0,342 0,294
q md 0,447 0,343 0,265 0,326 0,292 0,254 0,285 0,246
n 2,053 1,464 1,100 1,397 1,225 1,072 1,197 1,040
MIN

Das könnte Ihnen auch gefallen